You are on page 1of 55

Aznar vs. Garcia, G.R. No. L-16749, Jan.

3, 1963 Gibbs and his wife were American nationals, domiciled in


California. They acquired lands in the Philippines. The wife
FACTS: died in California. Gibbs was appointed administrator of
the intestate proceedings instituted in Manila. Gibbs asked
Edward Christensen, who at his death was a US citizen but
the court to adjudicate to him lands acquired in the
domiciled in the Philippines, left a will, devising unto Maria
Philippines not under our laws on succession but because
Helen a certain amount of money and giving the rest of his
in accordance with the law of California, the community
estate to Maria Lucy. Helen opposed the partition on the
property of spouses who are citizens of California, upon
ground that she is deprived of her legitime. Her contention
the death of the wife previous to that of the husband,
is that the law of California directs that the law of the
belongs absolutely to the surviving husband without
domicile (Philippines) should govern the will.
administration. CFI granted such to Gibbs upon proof of
California law. The register of deeds refused to transfer
ISSUE: Whether or not the national law or the domiciliary
such properties on the ground of non-payment of
law should apply
inheritance tax. Gibbs argued that the conjugal right of a
HELD: California wife in a community property is a personal right
and even if this was a case of succession, California law
The intrinsic validity of wills is governed by the national would still apply.
law of the decedent. In the present case, the national law
of Edward is the laws of California. However, there were Issue: is Gibbs exempt from inheritance tax?
two conflicting California laws regarding succession. One is
Held:
enunciated in In Re Kaufman (which does not provide for
legitimes) and another is Art. 946 of the California Civil
The appellee contends that the law of California should
Code (which provides that the law of the domicile
determine the nature and extent of the title, if any, that
applies). SC held that the national law is Art. 946, which is
was vested in Eva Johnson Gibbs citing article 9 of the
the conflict of laws rule of California. The reason is that In
Civil Code. But that, even if the nature and extent of her
Re Kaufman applies only to residents while Art. 946 is
title under said certificates be governed by the law of the
specific to non-residents. Thus, since Art. 946 contains a
Philippine Islands, the laws of California govern the
refer-back to Philippine laws (the law of the domicile), then
succession to such title, citing the second paragraph of
Maria Helen is entitled to her legitime.
article 10 of the Civil Code. It is argued that the conjugal
right of the California wife in community real estate in the
Bellis vs. Bellis, G.R. No. L-23678, June 6, 1967
Philippine Islands is a personal right and must, therefore,
FACTS: be settled by the law governing her personal status, that is,
the law of California. But our attention has not been called
Amos Bellis, a US citizen, died a resident of Texas. He left to any law of California that incapacitates a married
two wills -- one devising a certain amount of money to his woman from acquiring or holding land in a foreign
first wife and three illegitimate children and another, jurisdiction in accordance with the lex rei sitae.
leaving the rest of his estate to his seven legitimate
children. Before partition, the illegitimate children who are The trial court found that under the law of California,
Filipinos opposed on the ground that they are deprived of upon the death of the wife, the entire community
their legitimes. property without administration belongs to the surviving
husband; that he is the absolute owner of all the
ISSUE: Whether the applicable law is Texas law or community property from the moment of the death of
Philippine laws his wife, not by virtue of succession or by virtue of her
death, but by virtue of the fact that when the death of
HELD: the wife precedes that of the husband he acquires the
community property, not as an heir or as the beneficiary
Applying the nationality rule, the law of Texas should
of his deceased wife, but because she never had more
govern the intrinsic validity of the will and therefore
than an inchoate interest or expentancy which is
answer the question on entitlement to legitimes. But since
extinguished upon her death. However, the argument of
the law of Texas was never proven, the doctrine of
the appellee apparently leads to this dilemma: If he takes
processual presumption was applied. Hence, SC assumed
nothing by succession from his deceased wife, how can the
that Texas law is the same as Philippine laws, which
second paragraph of article 10 be invoked? Can the
upholds the nationality rule.
appellee be heard to say that there is a legal succession
Renvoi doctrine is not applicable because there is no under the law of the Philippine Islands and no legal
conflict as to the nationality and domicile of Bellis. He is succession under the law of California? It seems clear that
both a citizen and a resident of Texas. So even if assuming the second paragraph of article 10 applies only when a
the law of Texas applies the domiciliary rule, it is still legal or testamentary succession has taken place in the
Texas law that governs because his domicile is Texas. Philippines and in accordance with the law of the
Philippine Islands; and the foreign law is consulted only in
Gibbs v. Government of Philippine Islands regard to the order of succession or the extent of the
successional rights; in other words, the second paragraph
Facts:

Private International Law Page 1


of article 10 can be invoked only when the deceased was Yes. The laws of the foreign jurisdicti on do not
vested with a descendible interest in property within the prove themselves in our courts. The courts of the
jurisdiction of the Philippine Islands. Philippine Islands are not authorized to take judicial
notice of the laws of the various states of the
However, the court held that it is principle firmly American Union. Such laws must be proved as
facts.
established that to the law of the state in which the land
is situated we must look for the rules which govern its Here the requirements of the law were not met.
descent, alienation, and transfer, and for the effect and
construction of wills and other conveyances. This There was no showing that the book from
fundamental principle is stated in the first paragraph of which an extract was taken was printed or
article 10 of our Civil Code as follows: "Personal property published under the authority of the state of
is subject to the laws of the nation of the owner thereof; West Virginia, as provided in Sec 30 of the Code
of Civil Procedure.
real property to the laws of the country in which it is
Nor was the extract from the law attested by the
situated. certificate of the offi cer having charge of the
original, under the seal of the State of West
Under the provisions of the Civil Code and the Virginia as provided in Sec 301.
jurisprudence prevailing here, the wife, upon the No evidence was introduced showing that the
acquisition of any conjugal property, becomes extract from the laws of West Virginia was in
immediately vested with an interest and title therein force at the ti me the alleged will was executed.
equal to that of her husband, subject to the power of Due execution of the will was not established: only
showed testimony of the petitioner
management and disposition which the law vests in the
husband. It results that the wife of the appellee was, by
The court therefore did not err in denying the
the law of the Philippine Islands, vested of a descendible probate of the will. The existence of such law in
interest, equal to that of her husband and the descendible West
interest of Eva Johnson Gibbs in the lands aforesaid was Virginia must be proved.
transmitted to her heirs by virtue of inheritance and this
transmission plainly falls within the language of section
1536 of Article XI of Chapter 40 of the Administrative Miciano v. Brimo
Code which levies a tax on inheritances.
Decedent is a Turkish citizen. He left a will stating that he
Fleumer vs. Hix wishes to dispose of his properties in accordance with
54 Phil 610 Philippine Laws. His brother, Andre Brimo opposed the
partition. The appellant's opposition is based on the fact
Facts: that the partition in question puts into effect the
provisions of Joseph G. Brimo's will which are not in
The petitioner is a special administrator of the estate accordance with the laws of his Turkish nationality, for
of Edward Hix. He alleged that the latters will was
which reason they are void as being in violation or article
executed in Elkins, West Virginia on November 3,
1925 by Hix who had his residence in that 10 of the Civil Code which states that testamentary
jurisdicti on, and that the laws of that state govern. successions shall be governed by the national law of the
deceased. He was therefore excluded from the inheritance
To this end, the peti ti oner submitted a copy of pursuant to a clause in the decedents will that anyone
Secti on 3868 of Acts 1882, c.84 as found in West who would oppose the decedents wish to dispose his
Virginia Code, annotated by Hogg, Charles E., vol.2 estate under Philippine law shall have his share annul or
1914, p. 1690 and as certi fi ed to by the Director of
cancelled.
Nati onal Library.

The Judge of the First Instance however denied the Issue:


probate of the will on the grounds that the will did not
show the following: Whether or not the declaration that Turkish laws are
acknowledgment by Hix in the presence of 2 impertinent to this case;
competent witnesses
Witnesses subscribed to will in presence of the Whether or not the appellants exclusion from the will is
testator, and of each other valid?

Hence, this appeal. Held:

Issue: 1st issue

Is it necessary to prove in this jurisdicti on the The oppositor did not prove that said testamentary
existence of such law in West Virginia as a dispositions are not in accordance with the Turkish laws,
prerequisite to the allowance and recording of said
inasmuch as he did not present any evidence showing
will?
what the Turkish laws are on the matter, and in the
Held: absence of evidence on such laws, they are presumed to
be the same as those of the Philippines.

Private International Law Page 2


The refusal to give the oppositor another opportunity to death of Hodges and not in any other
prove such laws does not constitute an error. It is time.
discretionary with the trial court, and, taking into
consideration that the oppositor was granted ample The Supreme Court held that for what the Texas law is on
opportunity to introduce competent evidence, the Court the matter, is a question of fact to be resolved by the
finds no abuse of discretion on the part of the lower court evidence that would be presented in the probate court.
in this particular. There is, therefore, no evidence in the Texas law at the time of her death (and not said law at
record that the national law of the testator Joseph G. any other time).
Brimo was violated in the testamentary dispositions in
question which, not being contrary to our laws in force, Article 16 of the Civil Code provides that the national
must be complied with and executed. law of the person whose succession is under
consideration, whatever may be the nature of the
2nd issue property and regardless of the country wherein said
property may be found, shall prevail. However, the
The institution of legatees in this will is conditional, and Conflict of Law of Texas, which is the national law of the
the condition is that the instituted legatees must respect testatrix, Linnie Jane Hodges, provide that the domiciliary
the testator's will to distribute his property, not in law (Philippine law see paragraph 2, supra) should
accordance with the laws of his nationality, but in govern the testamentary dispositions and successional
accordance with the laws of the Philippines. rights over movables (personal properties), and the law of
the situs of the property (also Philippine law as to
properties located in the Philippines) with regards
The fact is, however, that the said condition is void, being immovable (real properties). Thus applying the Renvoi
contrary to law, for article 792 of the civil Code provides Doctrine, as approved and applied by our Supreme Court
the following: in the case of In The Matter Of The Testate Estate of
Eduard E. Christensen, G.R. No. L-16749, promulgated
Impossible conditions and those contrary to law January 31, 1963, Philippine law should apply to the Will
or good morals shall be considered as not of Linnie Jane Hodges and to the successional rights to
imposed and shall not prejudice the heir or her estate insofar as her movable and immovable assets
legatee in any manner whatsoever, even should in the Philippines are concerned. We shall not, at this
the testator otherwise provide. stage, discuss what law should govern the assets of Linnie
Jane Hodges located in Oklahoma and Texas, because the
And said condition is contrary to law because it expressly only assets in issue in this motion are those within the
ignores the testator's national law when, according to jurisdiction of this motion Court in the two above-
article 10 of the civil Code above quoted, such national captioned Special Proceedings.
law of the testator is the one to govern his testamentary
dispositions. Nota bene: When can foreign law be given judicial notice

PCIB VS. ESCOLIN 1. If the court of the forum is familiar of the law
56 SCRA 266 2. Or it is within the actual knowledge of the court

FACTS: Suntay v. Suntay

Linnie Jane Hodges died giving her testamentary provisions Jose B. Suntay died intestate leaving properties in the
to her husband. At the time of her Philippines and a house in China. He is survived by children
from the 1st marriage and a child and his widow from the
death, she was citizen of Texas but, was, however
2nd. Intestate proceedings were instituted. Thereafter the
domiciled in the Philippines. To see whether widow filed a petition for a probate of a will but was later
the testamentary provisions are valid, it is apparent and denied when the will was lost after the filing of said
necessary to know what law should be petition. On appeal, the petition was granted since there
applied. was sufficiency to prove the loss of the will. In spite of the
fact that a commission from the probate court was issued
on 24 April 1937 for the taking of the deposition of Go Toh,
ISSUE:
an attesting witness to the will, on 7 February 1938 the
probate court denied a motion for continuance of the
Whether or not laws of Texas is applicable.
hearing sent by cablegram from China by the surviving
widow and dismissed the petition. In the meantime the
RULING: Pacific War supervened. After liberation, Silvino claimed to
have found a will by his father which was filed, recorded
Prior evidence already presented to prove the existence of and probated in the Amoy district court, Province of
Fookien, China and thus filed a petition in the intestate
Texas Law.
proceedings praying for the probate of the will.

It is necessary that the Texas law be ascertained. Here it


Issue:
must be proven whether a renvoi will
happen or whether Texas law makes the testamentary May a will filed, recorded, and probated in China be
provisions valid. In line with Texas law, reprobated in the Philippines?
that which should be proven is the law enforced during the

Private International Law Page 3


Held: testimony of two attesting witnesses to the will and that
the order of the municipal district court of Amoy does not
As to the will claimed to have been executed on 4 January purport to probate the will. In the absence of proof that
1931 in Amoy, China, the law on the point in Rule 78. the municipal district court of Amoy is a probate court and
on the Chinese law of procedure in probate matters, it may
be presumed that the proceedings in the matter of
Section 1 of the rule provides:
probating or allowing a will in the Chinese courts are the a
deposition or to a perpetuation of testimony, and even if it
Wills proved and allowed in a foreign country, were so it does not measure same as those provided for in
according to the laws of such country, may be our laws on the subject. It is a proceedings in rem and for
allowed, filed, and recorded by the proper the validity of such proceedings personal notice or by
Court of First Instance in the Philippines. publication or both to all interested parties must be
made. The interested parties in the case were known to
Section 2 provides: reside in the Philippines. The evidence shows that no
such notice was received by the interested parties
When a copy of such will and the allowance residing in the Philippines. The proceedings had in the
thereof, duly authenticated, is filed with a municipal district court of Amoy, China, may be likened to
petition for allowance in the Philippines, by the or come up to the standard of such proceedings in the
executor or other person interested, in the court Philippines for lack of notice to all interested parties and
having jurisdiction, such court shall fix a time and the proceedings were held at the back of such interested
place for the hearing, and cause notice thereof parties.
to be given as in case of an original will
presented for allowance. The decree appealed from is affirmed, without
pronouncement as to costs.
Section 3 provides:
In re: Johnson
If it appears at the hearing that the will should
be allowed in the Philippines, the court shall so Emil H. Johnson, a native of Sweden and a naturalized
allow it, and a certificate of its allowance, citizen of the United States, died in the city of Manila,
signed by the Judge, and attested by the seal of leaving a holographic will and is signed by himself and two
the courts, to which shall be attached a copy of witnesses only, instead of three witnesses required by
the will, shall be filed and recorded by the clerk,
section 618 of the Code of Civil Procedure. However, a
and the will shall have the same effect as if
originally proved and allowed in such court. petition was presented in the Court of First Instance of the
city of Manila for the probate of this will, on the ground
The fact that the municipal district court of Amoy, China, that Johnson was at the time of his death a citizen of the
is a probate court must be proved. The law of China on State of Illinois, United States of America; that the will was
procedure in the probate or allowance of wills must also duly executed in accordance with the laws of that State;
be proved. The legal requirements for the execution of a and hence could properly be probated here. Petitioner
valid will in China in 1931 should also be established by contends that the decedent is not a citizen of Illinois and
competent evidence. There is no proof on these points.
prays to annul the decree of probate and put the estate
into intestate administration, thus preparing the way for
The unverified answers to the questions propounded by
the establishment of the claim of the petitioner as the sole
counsel for the appellant to the Consul General of the
Republic of China objected to by counsel for the appellee, legitimate heir of her father.
are inadmissible, because apart from the fact that the
office of Consul General does not qualify and make the Issue:
person who holds it an expert on the Chinese law on
procedure in probate matters, if the same be admitted, Whether or not judgment from which the petitioner seeks
the adverse party would be deprived of his right to relief should be set aside because the testator was not a
confront and cross-examine the witness. Consuls are resident of the State of Illinois and the will was not in
appointed to attend to trade matters. conformity with the laws of that State.

The order of the municipal district court of Amoy, China, Held:


does not purport to probate or allow the will which was
the subject of the proceedings. In view thereof, the will Emil Johnson was a citizen of the State of Illinois. In the
and the alleged probate thereof cannot be said to have absence of clear proof to the contrary it should be
been done in accordance with the accepted basic and presumed that a person naturalized in a court of a certain
fundamental concepts and principles followed in the
State thereby becomes a citizen of that State as well as of
probate and allowance of wills. Consequently, the
authenticated transcript of proceedings held in the the United States.
municipal district court of Amoy, China, cannot be deemed
and accepted as proceedings leading to the probate or In Section 625 of the Code of Civil Procedure it is declared
allowance of a will and, therefore, the will referred to that "the allowance by the court of a will of real or
therein cannot be allowed, filed and recorded by a personal property shall be conclusive as to its due
competent court of this country. execution."

Likewise, the proceedings had in the municipal district The due execution of a will involves conditions relating to a
court of Amoy were for the purpose of taking the number of matters, such as the age and mental capacity of

Private International Law Page 4


the testator, the signing of the document by the testator, United States. Acting upon said return, the Collector of
or by someone in his behalf, and the acknowledgment of Internal Revenue accepted the valuation of the personal
the instrument by him in the presence of the required properties declared therein, but increased the appraisal of
number of witnesses who affix their signatures to the will the two parcels of land located in Baguio City by fixing
to attest the act. The proof of all these requisites is their fair market value. After allowing the deductions
involved in the probate; and as to each and all of them claimed by the ancillary administrator for funeral expenses
the probate is conclusive. Our reported cases do not in the amount of P2,000.00 and for judicial and
contain the slightest intimation that a will which has been administration expenses in the sum of P5,500.00, the
probated according to law, and without fraud, can be Collector assessed the state the amount of P5,147.98 for
annulled, in any other proceeding whatever, on account of estate tax and P10,875,26 or inheritance tax, or a total of
any supposed irregularity or defect in the execution of the P16,023.23. Both of these assessments were paid by the
will or on account of any error in the action of the court estate.
upon the proof adduced before it. The ancillary administrator filed in amended estate and
inheritance tax return in pursuance of his reservation
We are not unmindful of the fact that when a citizen of made at the time of filing of the preliminary return and
one State leaves it and takes up his abode in another State for the purpose of availing of the right granted by section
with no intention of returning, he immediately acquires 91 of the National Internal Revenue Code.
citizenship in the State of his new domicile. This is in Beatrice Mauricia Stevenson assigned all her rights and
accordance with that provision of the Fourteenth interests in the estate to the spouses, Douglas and Bettina
Amendment to the Constitution of the United States which Fisher, respondents herein.
says that every citizen of the United States is a citizen of The ancillary administrator filed a second amended estate
the State where in he resides. The effect of this provision and inheritance tax return. This return declared the same
necessarily is that a person transferring his domicile from assets of the estate stated in the amended return of
one State to another loses his citizenship in the State of September 22, 1952, except that it contained new claims
his original above upon acquiring citizenship in the State for additional exemption and deduction to wit: (1)
of his new abode. The acquisition of the new State deduction in the amount of P4,000.00 from the gross
citizenship extinguishes the old. That situation, in our estate of the decedent as provided for in Section 861 (4)
opinion, has no analogy to that which arises when a of the U.S. Federal Internal Revenue Code which the
citizen of an American State comes to reside in the ancillary administrator averred was allowable by way of
Philippine Islands. Here he cannot acquire a new the reciprocity granted by Section 122 of the National
citizenship; nor by the mere change of domicile does he Internal Revenue Code, as then held by the Board of Tax
lose that which he brought with him. Appeals in case No. 71 entitled "Housman vs. Collector,"
August 14, 1952; and (2) exemption from the imposition
of estate and inheritance taxes on the 210,000 shares of
Proper rule in taking judicial notice: The proper rule is to
stock in the Mindanao Mother Lode Mines, Inc. also
require proof of the statutes of the States of the American
pursuant to the reciprocity proviso of Section 122 of the
Union whenever their provisions are determinative of the
National Internal Revenue Code. In this last return, the
issues in any action litigated in the Philippine courts.
estate claimed that it was liable only for the amount of
P525.34 for estate tax and P238.06 for inheritance tax and
Collector vs. Fisher that, as a consequence, it had overpaid the government.
The refund of the amount of P15,259.83, allegedly
Facts: overpaid, was accordingly requested by the estate. The
Collector denied the claim. For this reason, action was
This case relates to the determination and settlement of commenced in the Court of First Instance of Manila by
the hereditary estate left by the deceased Walter G. respondents, as assignees of Beatrice Mauricia Stevenson,
Stevenson, and the laws applicable thereto. Walter G. for the recovery of said amount. Pursuant to Republic Act
Stevenson (born in the Philippines on August 9, 1874 of No. 1125, the case was forwarded to the Court of Tax
British parents and married in the City of Manila on Appeals which court, after hearing, rendered decision :
January 23, 1909 to Beatrice Mauricia Stevenson another that: (a) the one-half () share of the surviving spouse in
British subject) died on February 22, 1951 in San Francisco, the conjugal partnership property as diminished by the
California, U.S.A. whereto he and his wife moved and obligations properly chargeable to such property should be
established their permanent residence since May 10, deducted from the net estate of the deceased Walter G.
1945. In his will executed in San Francisco on May 22, Stevenson, pursuant to Section 89-C of the National
1947, and which was duly probated in the Superior Court Internal Revenue Code; (b) the intangible personal
of California on April 11, 1951, Stevenson instituted his property belonging to the estate of said Stevenson is
wife Beatrice as his sole heiress to the following real and exempt from inheritance tax, pursuant to the provision of
personal properties acquired by the spouses while section 122 of the National Internal Revenue Code in
residing in the Philippines. relation to the California Inheritance Tax Law but
Ancillary administration proceedings were instituted in the decedent's estate is not entitled to an exemption of
Court of First Instance of Manila for the settlement of the P4,000.00 in the computation of the estate tax; (c) for
estate in the Philippines. In due time Stevenson's will was purposes of estate and inheritance taxation the Baguio real
duly admitted to probate by our court and Ian Murray Statt estate of the spouses should be valued at P52,200.00, and
was appointed ancillary administrator of the estate, filed a 210,000 shares of stock in the Mindanao Mother Lode
preliminary estate and inheritance tax return with the Mines, Inc. should be appraised at P0.38 per share; and (d)
reservation of having the properties declared therein the estate shall be entitled to a deduction of P2,000.00 for
finally appraised at their values six months after the death funeral expenses and judicial expenses of P8,604.39.
of Stevenson. Preliminary return was made by the ancillary
administrator in order to secure the waiver of the Collector Issue: whether or not foreign law needs to be proved in
of Internal Revenue on the inheritance tax due on the our jurisdiction?
210,000 shares of stock in the Mindanao Mother Lode
Mines Inc. which the estate then desired to dispose in the Ruling:

Private International Law Page 5


1922; that sometime in 1925, Magdalena C. Bohanan
It is well-settled that foreign laws do not prove married Carl Aaron and this marriage was subsisting at
themselves in our jurisdiction and our courts are not the time of the death of the testator. Since no right to
authorized to take judicial notice of them. Like any other share in the inheritance in favor of a divorced wife exists
fact, they must be alleged and proved. Section 41, Rule in the State of Nevada and since the court below had
123 of our Rules of Court prescribes the manner of proving already found that there was no conjugal property
foreign laws before our tribunals. However, although we between the testator and Magdalena C. Bohanan, the
believe it desirable that these laws be proved in latter can now have no longer claim to pay portion of the
accordance with said rule, we held in the case estate left by the testator.
of Willamette Iron and Steel Works v. Muzzal, 61 Phil. 471,
that "a reading of sections 300 and 301 of our Code of Civil Issue:
Procedure (now section 41, Rule 123) will convince one
that these sections do not exclude the presentation of whether the estementary dispositions, especially those for
other competent evidence to prove the existence of a the children which are short of the legitime given them by
foreign law." In that case, we considered the testimony of the Civil Code of the Philippines, are valid?
an attorney-at-law of San Francisco, California who
quoted verbatim a section of California Civil Code and Ruling:
who stated that the same was in force at the time the
obligations were contracted, as sufficient evidence to The old Civil Code, which is applicable to this case
establish the existence of said law. In line with this view, because the testator died in 1944, expressly provides that
we find no error, therefore, on the part of the Tax Court in successional rights to personal property are to be earned
considering the pertinent California law as proved by by the national law of the person whose succession is in
respondents' witness. question. Says the law on this point:
Nevertheless, legal and testamentary successions, in
Phil Trust Company vs. Bohanan respect to the order of succession as well as to the extent
of the successional rights and the intrinsic validity of their
Facts: provisions, shall be regulated by the national law of the
person whose succession is in question, whatever may be
Appeal against an order of the Court of First Instance of the nature of the property and the country in which it is
Manila, Hon. Ramon San Jose, presiding, dismissing the found. (par. 2, Art. 10, old Civil Code, which is the same as
objections filed by Magdalena C. Bohanan, Mary Bohanan par. 2 Art. 16, new Civil Code.)
and Edward Bohanan to the project of partition submitted In the proceedings for the probate of the will, it was found
by the executor and approving the said project. out and it was decided that the testator was a citizen of
The Court of First Instance of Manila, Hon. Rafael Amparo, the State of Nevada because he had selected this as his
presiding, admitted to probate a last will and testament of domicile and his permanent residence. It is not disputed
C. O. Bohanan, executed by him in Manila. In the said that the laws of Nevada allow a testator to dispose of all
order, the court made the following findings: his properties by will. It does not appear that at time of the
According to the evidence of the opponents the testator hearing of the project of partition, the above-quoted
was born in Nebraska and therefore a citizen of that state, provision was introduced in evidence, as it was the
or at least a citizen of California where some of his executor's duly to do. The law of Nevada, being a foreign
properties are located. This contention is untenable. law can only be proved in our courts in the form and
Notwithstanding the long residence of the decedent in the manner provided for by our Rules, which are as follows:
Philippines, his stay here was merely temporary, and he SEC. 41. Proof of public or official record. An official
continued and remained to be a citizen of the United record or an entry therein, when admissible for any
States and of the state of his pertinent residence to spend purpose, may be evidenced by an official publication
the rest of his days in that state. His permanent residence thereof or by a copy attested by the officer having the
or domicile in the United States depended upon his legal custody of the record, or by his deputy, and
personal intent or desire, and he selected Nevada as his accompanied, if the record is not kept in the Philippines,
homicide and therefore at the time of his death, he was a with a certificate that such officer has the custody. . . .
citizen of that state. Nobody can choose his domicile or (Rule 123).
permanent residence for him. That is his exclusive
personal right. We have, however, consulted the records of the case in the
Wherefore, the court finds that the testator C. O. Bohanan court below and we have found that during the hearing on
was at the time of his death a citizen of the United States October 4, 1954 of the motion of Magdalena C. Bohanan
and of the State of Nevada and declares that his will and for withdrawal of P20,000 as her share, the foreign law,
testament, is fully in accordance with the laws of the state especially Section 9905, Compiled Nevada Laws was
of Nevada and admits the same to probate. Accordingly, introduced in evidence by appellant's counsel. Again said
the Philippine Trust Company, named as the executor of laws presented by the counsel for the executor and
the will, is hereby appointed to such executor and upon admitted by the Court during the hearing of the case on
the filing of a bond in the sum of P10,000.00. before Judge Rafael Amparo.
The executor filed a project of partition dated January 24, In addition, the other appellants, children of the testator,
1956, making adjudications, in accordance with the do not dispute the above-quoted provision of the laws of
provisions of the will. the State of Nevada. Under all the above circumstances,
The wife Magadalena C. Bohanan and her two children we are constrained to hold that the pertinent law of
question the validity of the testamentary provisions Nevada, especially Section 9905 of the Compiled Nevada
disposing of the estate in the manner above indicated, Laws of 1925, can be taken judicial notice of by us,
claiming that they have been deprived of the legitime without proof of such law having been offered at the
that the laws of the forum concede to them. hearing of the project of partition.
Moreover, the court below had found that the testator and As in accordance with Article 10 of the old Civil Code, the
Magdalena C. Bohanan were married on January 30, validity of testamentary dispositions are to be governed by
1909, and that divorce was granted to him on May 20, the national law of the testator, and as it has been decided

Private International Law Page 6


and it is not disputed that the national law of the testator an official publication thereof or by a copy attested by the
is that of the State of Nevada, already indicated above, officer having the legal custody of the record, or by his
which allows a testator to dispose of all his property deputy, and accompanied with a certificate that such
according to his will, as in the case at bar, the order of the officer has custody. The certificate may be made by a
court approving the project of partition made in secretary of an embassy or legation, consul general,
accordance with the testamentary provisions, must be, as consul, vice-consul, or consular agent or by any officer in
it is hereby affirmed, with costs against appellants. the foreign service of the Philippines stationed in the
foreign country in which the record is kept, and
Zalamea vs. CA authenticated by the seal of his office. 7
Respondent TWA relied solely on the statement of Ms.
Facts: Gwendolyn Lather, its customer service agent, in her
deposition dated January 27, 1986 that the Code of
Petitioners-spouses Cesar C. Zalamea and Suthira Zalamea, Federal Regulations of the Civil Aeronautics Board allows
and their daughter, Liana Zalamea, purchased three (3) overbooking. Aside from said statement, no official
airline tickets from the Manila agent of respondent publication of said code was presented as evidence. Thus,
TransWorld Airlines, Inc. for a flight to New York to Los respondent court's finding that overbooking is specifically
Angeles. allowed by the US Code of Federal Regulations has no basis
The tickets of petitioners-spouses were purchased at a in fact.
discount of 75% while that of their daughter was a full fare Even if the claimed U.S. Code of Federal Regulations does
ticket. All three tickets represented confirmed exist, the same is not applicable to the case at bar in
reservations. accordance with the principle of lex loci contractus which
Petitioners received notice of the reconfirmation of their require that the law of the place where the airline ticket
reservations for said flight. On the appointed date, was issued should be applied by the court where the
however, petitioners checked in at 10:00 a.m., an hour passengers are residents and nationals of the forum and
earlier than the scheduled flight at 11:00 a.m. but were the ticket is issued in such State by the defendant
placed on the wait-list because the number of passengers airline. 8 Since the tickets were sold and issued in the
who had checked in before them had already taken all the Philippines, the applicable law in this case would be
seats available on the flight. Liana Zalamea appeared as Philippine law.
the No. 13 on the wait-list while the two other Zalameas Existing jurisprudence explicitly states that overbooking
were listed as "No. 34, showing a party of two." Out of the amounts to bad faith, entitling the passengers concerned
42 names on the wait list, the first 22 names were to an award of moral damages. In Alitalia Airways v. Court
eventually allowed to board the flight to Los Angeles, of Appeals, 9 where passengers with confirmed bookings
including petitioner Cesar Zalamea. The two others, on the were refused carriage on the last minute, this Court held
other hand, at No. 34, being ranked lower than 22, were that when an airline issues a ticket to a passenger
not able to fly. As it were, those holding full-fare tickets confirmed on a particular flight, on a certain date, a
were given first priority among the wait-listed passengers. contract of carriage arises, and the passenger has every
Mr. Zalamea, who was holding the full-fare ticket of his right to expect that he would fly on that flight and on that
daughter, was allowed to board the plane; while his wife date. If he does not, then the carrier opens itself to a suit
and daughter, who presented the discounted tickets were for breach of contract of carriage. Where an airline had
denied boarding. deliberately overbooked, it took the risk of having to
Even in the next TWA flight to Los Angeles Mrs. Zalamea deprive some passengers of their seats in case all of them
and her daughter, could not be accommodated because it would show up for the check in. For the indignity and
was also fully booked. Thus, they were constrained to book inconvenience of being refused a confirmed seat on the
in another flight and purchased two tickets from American last minute, said passenger is entitled to an award of moral
Airlines at a cost of Nine Hundred Eighteen ($918.00) damages.
Dollars.
Upon their arrival in the Philippines, petitioners filed an Wild Valley Shipping Co. Vs. CA
action for damages based on breach of contract of air
carriage before the Regional Trial Court of Makati. As Facts:
aforesaid, the lower court ruled in favor of petitioners.
On appeal, the respondent Court of Appeals held that The Philippine Roxas, a vessel owned by Philippine
moral damages are recoverable in a damage suit President Lines, Inc., private respondent herein, arrived in
predicated upon a breach of contract of Puerto Ordaz, Venezuela, to load iron ore.
carriage only where there is fraud or bad faith. Since it is a Upon the completion of the loading and when the vessel
matter of record that overbooking of flights is a common was ready to leave port, an official pilot of Venezuela, was
and accepted practice of airlines in the United States and designated by the harbour authorities in Puerto Ordaz to
is specifically allowed under the Code of Federal navigate the Philippine Roxas through the Orinoco River.
Regulations by the Civil Aeronautics Board, no fraud nor The Philippine Roxas experienced some vibrations when it
bad faith could be imputed on respondent TransWorld entered the San Roque Channel. The vessel proceeded on
Airlines. its way, with the pilot assuring the watch officer that the
vibration was a result of the shallowness of the channel.
Ruling: The master (captain) checked the position of the vessel
and verified that it was in the centre of the channel.
That there was fraud or bad faith on the part of The Philippine Roxas ran around in the Orinoco River, thus
respondent airline when it did not allow petitioners to obstructing the ingress and egress of vessels.
board their flight for Los Angeles in spite of confirmed As a result of the blockage, the Malandrinon, a vessel
tickets cannot be disputed. The U.S. law or regulation owned by herein petitioner Wildvalley Shipping Company,
allegedly authorizing overbooking has never been proved. Ltd., was unable to sail out of Puerto Ordaz on that day.
Foreign laws do not prove themselves nor can the courts Subsequently, Wildvalley Shipping Company, Ltd. filed a
take judicial notice of them. Like any other fact, they must suit with the Regional Trial Court of Manila, Branch III
be alleged and proved. Written law may be evidenced by against Philippine President Lines, Inc. and Pioneer

Private International Law Page 7


Insurance Company (the underwriter/insurer of Philippine Board of Commissioners. Acting commissioner issued an
Roxas) for damages in the form of unearned profits, and order affirming the decision of the Board of Special
interest thereon amounting to US $400,000.00 plus Inquiry.
attorney's fees, costs, and expenses of litigation.
On August 15, 1990, the Commission on Immigration and
Issue: whether or not Venezuelan law is applicable to the Deportatiion ordered the arrest of William and was
case at bar? released upon posting P 200,000 cash bond. Thus on the
29th of the same month, he filed a petition for certiorari
Ruling: and prohibition before the RTC of Manila. A motion to
dismiss was filed but denied.
It is well-settled that foreign laws do not prove
themselves in our jurisdiction and our courts are not Issue:
authorized to take judicial notice of them. Like any other
fact, they must be alleged and proved. Whether or not William Gatchalian is to be declared as a
For a copy of a foreign public document to be admissible, Filipino citizen
the following requisites are mandatory: (1) It must be
attested by the officer having legal custody of the records Held:
or by his deputy; and (2) It must be accompanied by a
certificate by a secretary of the embassy or legation, William Gatchalian is declared as a Filipino Citizen. Having
consul general, consul, vice consular or consular agent or declared the assailed marriage as valid, respondent
foreign service officer, and with the seal of his office. The William Gatchalian follows the citizenship of his father, a
latter requirement is not a mere technicality but is Filipino as legitimate child. Respondent belongs to a class
intended to justify the giving of full faith and credit to the of Filipinos who are citizens of the Philippines at the time
genuineness of a document in a foreign country. of the adoption of the constitution.
With respect to proof of written laws, parol proof is
objectionable, for the written law itself is the best In Moy Ya Lim vs. Commissioner of Immigration (41 SCRA
evidence. According to the weight of authority, when a 292 [1971]) and in Lee vs. Commissioner of Immigration
foreign statute is involved, the best evidence rule requires (supra), this Court declared that:
that it be proved by a duly authenticated copy of the
statute. (e)verytime the citizenship of a person is material or
At this juncture, we have to point out that the Venezuelan indispensable in a judicial or administrative case, whatever
law was not pleaded before the lower court. the corresponding court or administrative authority
A foreign law is considered to be pleaded if there is an decides therein as to such citizenship is generally not
allegation in the pleading about the existence of the considered as res adjudicata, hence it has to be threshed
foreign law, its import and legal consequence on the out again and again as the occasion may demand.
event or transaction in issue.
A review of the Complaint revealed that it was never An exception to the above rule was laid by this Court in
alleged or invoked despite the fact that the grounding of Burca vs. Republic (51 SCRA 248 [1973]), viz:
the M/V Philippine Roxas occurred within the territorial
jurisdiction of Venezuela. We declare it to be a sound rule that where the citizenship
We reiterate that under the rules of private international of a party in a case is definitely resolved by a court or by an
law, a foreign law must be properly pleaded and proved as administrative agency, as a material issue in the
a fact. In the absence of pleading and proof, the laws of a controversy, after a full-blown hearing with the active
foreign country, or state, will be presumed to be the same participation of the Solicitor General or his authorized
as our own local or domestic law and this is known as representative, and this finding or the citizenship of the
processual presumption. party is affirmed by this Court, the decision on the matter
shall constitute conclusive proof of such party's citizenship
Board of Commissioners v. Dela Rosa in any other case or proceeding. But it is made clear that in
no instance will a decision on the question of citizenship in
Facts: such cases be considered conclusive or binding in any
other case or proceeding, unless obtained in accordance
On July 12, 1960, Santiago Gatchalian, grandfather of with the procedure herein stated.
William Gatchalian, was recognized by the Bureau of
Immigration as a native born Filipino citizen following the Thus, in order that the doctrine of res judicata may be
citizenship of natural mother Mariana Gatchalian. On June applied in cases of citizenship, the following must be
27, 1961, Willian, then twelve years old, arrives in Manila present: 1) a person's citizenship must be raised as a
from Hongkong together with a daughter and a son of material issue in a controversy where said person is a
Santiago. They had with them certificate of registration party; 2) the Solicitor General or his authorized
and identity issued by the Philippine consulate in representative took active part in the resolution thereof,
Hongkong based on a cablegram bearing the signature of and 3) the finding or citizenship is affirmed by this Court.
the secretary of foreign affairs, Felixberto Serrano, and
sought admission as Filipino citizens. Doctrine of processual presumption

On July 6, 1961, the board of special inquiry admitted the Civil Code
Gatchalians as Filipino citizens and issued an identification
certificate to William. The board of commissioners was Art. 15. Laws relating to family rights and
directed by the Secretary of Justice to Review all cases duties, or to the status, condition and
where entry was granted on the ground that the entrant legal capacity of persons are binding
was a Filipino citizen such included the case of William. As upon citizens of the Philippines, even
a result of the decision of the board of special inquiry though living abroad. (9a)
which recommended for the reversal of the decision of the

Private International Law Page 8


Art. 16. Real property as well as personal Art. 815. When a Filipino is in a foreign country,
property is subject to the law of the he is authorized to make a will in any of
country where it is stipulated. the forms established by the law of the
country in which he may be. Such will
However, intestate and testamentary may be probated in the Philippines. (n)
successions, both with respect to the
order of succession and to the amount Art. 816. The will of an alien who is abroad
of successional rights and to the produces effect in the Philippines if
intrinsic validity of testamentary made with the formalities prescribed
provisions, shall be regulated by the by the law of the place in which he
national law of the person whose resides, or according to the formalities
succession is under consideration, observed in his country, or in
whatever may be the nature of the conformity with those which this Code
property and regardless of the country prescribes. (n)
wherein said property may be found.
(10a) Art. 818. Two or more persons cannot make a
will jointly, or in the same instrument,
Art. 17. The forms and solemnities of contracts, either for their reciprocal benefit or for
wills, and other public instruments the benefit of a third person. (669)
shall be governed by the laws of the
country in which they are executed. Art. 819. Wills, prohibited by the preceding
article, executed by Filipinos in a
When the acts referred to are executed foreign country shall not be valid in the
before the diplomatic or consular Philippines, even though authorized by
officials of the Republic of the the laws of the country where they
Philippines in a foreign country, the may have been executed. (733a)
solemnities established by Philippine
laws shall be observed in their Art. 829. A revocation done outside the
execution. Philippines, by a person who does not
have his domicile in this country, is
Prohibitive laws concerning persons, valid when it is done according to the
their acts or property, and those which law of the place where the will was
have, for their object, public order, made, or according to the law of the
public policy and good customs shall place in which the testator had his
not be rendered ineffective by laws or domicile at the time; and if the
judgments promulgated, or by revocation takes place in this country,
determinations or conventions agreed when it is in accordance with the
upon in a foreign country. (11a) provisions of this Code. (n)

Art. 71. All marriages performed outside the Art. 1039. Capacity to succeed is governed by the
Philippines in accordance with the laws law of the nation of the decedent. (n)
in force in the country where they
were performed, and valid there as Rule 132 Sec. 25
such, shall also be valid in this country,
What attestation of copy must state. Whenever a copy
except bigamous, polygamous, or
of a document or record is attested for the purpose of
incestuous marriages as determined by
evidence, the attestation must state, in substance, that the
Philippine law. (19a)
copy is a correct copy of the original, or a specific part
thereof, as the case may be. The attestation must be under
Art. 124. If the marriage is between a citizen of
the Philippines and a foreigner, the official seal of the attesting officer, if there be any, or if
whether celebrated in the Philippines he be the clerk of a court having a seal, under the seal of
or abroad, the following rules shall such court. (26a)
prevail:
Rule 130
1. If the husband is a citizen of the
Philippines while the wife is a Section 45. Commercial lists and the like. Evidence of
foreigner, the provisions of this statements of matters of interest to persons engaged in an
Code shall govern their relations; occupation contained in a list, register, periodical, or other
2. If the husband is a foreigner and published compilation is admissible as tending to prove the
the wife is a citizen of the truth of any relevant matter so stated if that compilation is
Philippines, the laws of the
published for use by persons engaged in that occupation
husband's country shall be
followed, without prejudice to the and is generally used and relied upon by them therein. (39)
provisions of this Code with
regard to immovable property. Section 46. Learned treatises. A published treatise,
(1325a) periodical or pamphlet on a subject of history, law, science,
or art is admissible as tending to prove the truth of a
matter stated therein if the court takes judicial notice, or a

Private International Law Page 9


witness expert in the subject testifies, that the writer of shall enjoy full civil and political rights and be subject to all
the statement in the treatise, periodical or pamphlet is attendant liabilities and responsibilities under existing laws
recognized in his profession or calling as expert in the of the Philippines and the following conditions:
subject. (40a)
1. Those intending to exercise their right of suffrage
How to acquire nationality must meet the requirements under Section 1, Article
1. by birth V of the Constitution, Republic Act No. 9189,
2. by repatriation otherwise known as "The Overseas Absentee Voting
3. by naturalization
Act of 2003" and other existing laws;
4. by subrogation and cession
2. Those seeking elective public office in the Philippines
how to lose a nationality shall meet the qualifications for holding such public
1. release office as required by the Constitution and existing
2. deprivation laws and, at the time of the filing of the certificate of
3. expiration candidacy, make a personal and sworn renunciation
4. renunciation of any and all foreign citizenship before any public
officer authorized to administer an oath;
domicile 3. Those appointed to any public office shall subscribe
1. intent to stay
and swear to an oath of allegiance to the Republic of
2. physical presence
3. conduct indicative of such intention the Philippines and its duly constituted authorities
prior to their assumption of office: Provided, That
Rep. Act No. 9225 they renounce their oath of allegiance to the country
signed into law by President Gloria M. Arroyo on August where they took that oath;
29, 2003

SECTION 1. Short Title.-This Act shall be known as the


"Citizenship Retention and Reacquisition Act of 2003." 4. Those intending to practice their profession in the
Philippines shall apply with the proper authority for a
SEC. 2. Declaration of Policy.-It is hereby declared the license or permit to engage in such practice; and
policy of the State that all Philippine citizens who become 5. That right to vote or be elected or appointed to any
citizens of another country shall be deemed not to have public office in the Philippines cannot be exercised by,
lost their Philippine citizenship under the conditions of or extended to, those who:
this Act. a. are candidates for or are occupying any public
office in the country of which they are
SEC. 3. Retention of Philippine Citizenship.-Any provision of naturalized citizens; and/or
law to the contrary notwithstanding, natural-born citizens b. are in the active service as commissioned or
of the Philippines who have lost their Philippine noncommissioned officers in the armed forces of
citizenship by reason of their naturalization as citizens of the country which they are naturalized citizens.
a foreign country are hereby deemed to have reacquired
Philippine citizenship upon taking the following oath of SEC. 6. Separability Clause. - If any section or provision of
allegiance to the Republic: this Act is held unconstitutional or invalid, any other
section or provision not affected thereby shall remain valid
"I ___________________________, solemnly swear (or and effective.
affirm) that I will support and defend the Constitution of
the Republic of the Philippines and obey the laws and legal SEC. 7. Repealing Clause. - All laws, decrees, orders, rules
orders promulgated by the duly constituted authorities of and regulations inconsistent with the provisions of this Act
the Philippines; and I hereby declare that I recognize and are hereby repealed or modified accordingly.
accept the supreme authority of the Philippines and will
maintain true faith and allegiance thereto; and that I SEC. 8. Effectivity Clause. - This Act shall take effect after
impose this obligation upon myself voluntarily without fifteen (15) days following its publication in the Official
mental reservation or purpose of evasion." Gazette or two (2) newspapers of general circulation.

Natural-born citizens of the Philippines who, after the ARTICLE IV


effectivity of this Act, become citizens of a foreign
CITIZENSHIP
country shall retain their Philippine citizenship upon
taking the aforesaid oath. Section 1. The following are citizens of the
Philippines:
SEC. 4. Derivative Citizenship. - The unmarried child,
whether legitimate, illegitimate or adopted, below 1. Those who are citizens of the
eighteen (18) years of age, of those who reacquire Philippines at the time of the
Philippine citizenship upon effectivity of this Act shall be adoption of this Constitution;
deemed citizens of the Philippines.
2. Those whose fathers or
SEC. 5. Civil and Political Rights and Liabilities. - Those who mothers are citizens of the
retain or reacquire Philippine citizenship under this Act Philippines;

Private International Law Page 10


3. Those born before January Art. 1039. Capacity to succeed is governed by the
17, 1973, of Filipino mothers, law of the nation of the decedent. (n)
who elect Philippine
citizenship upon reaching the Art. 50. For the exercise of civil rights and the
age of majority; and fulfillment of civil obligations, the
domicile of natural persons is the
4. Those who are naturalized in place of their habitual residence. (40a)
accordance with law.
Art. 51. When the law creating or recognizing
Section 2. Natural-born citizens are those who them, or any other provision does not
are citizens of the Philippines from fix the domicile of juridical persons, the
birth without having to perform any same shall be understood to be the
act to acquire or perfect their place where their legal representation
Philippine citizenship. Those who elect is established or where they exercise
Philippine citizenship in accordance their principal functions. (41a)
with paragraph (3), Section 1 hereof
shall be deemed natural-born citizens. Art. 99. No person shall be entitled to a legal
separation who has not resided in the
Section 3. Philippine citizenship may be lost or Philippines for one year prior to the
reacquired in the manner provided by law. filing of the petition, unless the cause
for the legal separation has taken place
Section 4. Citizens of the Philippines who marry within the territory of this Republic.
aliens shall retain their citizenship, (Sec. 2a, Act No. 2710)
unless by their act or omission, they
are deemed, under the law, to have Art. 1251. Payment shall be made in the place
renounced it. designated in the obligation.

Section 5. Dual allegiance of citizens is inimical to There being no express stipulation and
the national interest and shall be dealt if the undertaking is to deliver a
with by law. determinate thing, the payment shall
be made wherever the thing might be
NCC Art. 15. Laws relating to family rights and at the moment the obligation was
duties, or to the status, condition and constituted.
legal capacity of persons are binding
upon citizens of the Philippines, even In any other case the place of payment
though living abroad. (9a) shall be the domicile of the debtor.

Art. 16. Real property as well as personal If the debtor changes his domicile in
property is subject to the law of the bad faith or after he has incurred in
country where it is stipulated. delay, the additional expenses shall be
borne by him.
However, intestate and testamentary
successions, both with respect to the These provisions are without prejudice
order of succession and to the amount to venue under the Rules of Court.
of successional rights and to the (1171a)
intrinsic validity of testamentary
provisions, shall be regulated by the Art. 58. Save marriages of an exceptional
national law of the person whose character authorized in Chapter 2 of
succession is under consideration, this Title, but not those under Article
whatever may be the nature of the 75, no marriage shall be solemnized
property and regardless of the country without a license first being issued by
wherein said property may be found. the local civil registrar of the
(10a) municipality where either contracting
party habitually resides. (7a)
Art. 66. When either or both of the contracting
parties are citizens or subjects of a Art. 829. A revocation done outside the
foreign country, it shall be necessary, Philippines, by a person who does not
before a marriage license can be have his domicile in this country, is
obtained, to provide themselves with a valid when it is done according to the
certificate of legal capacity to contract law of the place where the will was
marriage, to be issued by their made, or according to the law of the
respective diplomatic or consular place in which the testator had his
officials. (13a) domicile at the time; and if the
revocation takes place in this country,

Private International Law Page 11


when it is in accordance with the of confronting the issue of whether or not there is dual
provisions of this Code. (n)
allegiance to the concerned foreign country. What
Cases: happens to the other citizenship was not made a concern
of Rep. Act No. 9225.
AASJS vs. Datumanong
For its part, the OSG counters that pursuant to Section 5,
G.R. No. 160869, May 11, 2007
Article IV of the 1987 Constitution, dual allegiance shall be
Sec. 3, RA 9225 stayed clear of the problem of dual dealt with by law. Thus, until a law on dual allegiance is
allegiance and shifted the burden of confronting the issue enacted by Congress, the Supreme Court is without any
of whether or not there is dual allegiance to the concerned
jurisdiction to entertain issues regarding dual allegiance.
foreign country.

FACTS: Moreover, Section 5, Article IV of the Constitution is a


declaration of a policy and it is not a self-executing
Petitioner filed the instant petition against respondent, provision. The legislature still has to enact the law on
then Secretary of Justice Simeon Datumanong, the official dual allegiance. In Sections 2 and 3 of Rep. Act No. 9225,
tasked to implement laws governing citizenship in order to the framers were not concerned with dual citizenship per
prevent the Justice Secretary from implementing R. A. se, but with the status of naturalized citizens who
9225. Petitioner argues that RA 9225 is unconstitutional as maintain their allegiance to their countries of origin even
it violates Sec. 5, Article VI of the Constitution which states after their naturalization. Congress was given a mandate
that dual allegiance of citizens is inimical to national to draft a law that would set specific parameters of what
interest and shall be dealt with by law. The Office of the really constitutes dual allegiance. Until this is done, it
Solicitor General (OSG) claims that Section 2 merely would be premature for the judicial department, including
declares as a state policy that "Philippine citizens who this Court, to rule on issues pertaining to dual allegiance.
become citizens of another country shall be deemed not to
ELLIS V. REPUBLIC
have lost their Philippine citizenship." The OSG further 7 SCRA 962
claims that the oath in Section 3 does not allow dual
allegiance since the oath taken by the former Filipino Petitioner Marvin G. Ellis, a native of San Fransisco,
California and is married to Gloria G. Ellis in Banger, Maine,
citizen is an effective renunciation and repudiation of his
United States. Both are citizens of the United States. Baby
foreign citizenship. The fact that the applicant taking the
Rose was born on September 26, 1959. Four or five days
oath recognizes and accepts the supreme authority of the later, the mother of Rose left her with the Heart of Mary
Philippines is an unmistakable and categorical affirmation Villa an institution for unwed mothers and their babies
of his undivided loyalty to the Republic. stating that she (the mother) could not take of Rose
without bringing disgrace upon her (the mother's family.).
Issue:
Being without issue, on November 22, 1959, Mr. and Mrs.
Whether R.A. 9225 is unconstitutional and whether the Ellis filed a petition with the Court of First Instance of
court Pampanga for the adoption of the aforementioned baby.
At the time of the hearing of the petition on January 14,
Held:
1960, petitioner Marvin G. Ellis and his wife had been in
the Philippines for three (3) years, he being assigned
R.A. 9225 is constitutional and that the Court has no thereto as staff sergeant in the United States Air Force
jurisdiction yet to pass upon the issue of dual allegiance. Base, in Angeles, Pampanga where both lived at that time.
The court held that that the intent of the legislature in They had been in the Philippines before, or, to exact, in
drafting Rep. Act No. 9225 is to do away with the 1953.
provision in Commonwealth Act No. 635 which takes
Issue:
away Philippine citizenship from natural-born Filipinos
who become naturalized citizens of other countries. What Whether or not being permanent residents in the
Rep. Act No. 9225 does is allow dual citizenship to natural- Philippines, petitioners are qualified to adopt Baby Rose.
born Filipino citizens who have lost Philippine citizenship
Held:
by reason of their naturalization as citizens of a foreign
country. On its face, it does not recognize dual allegiance. Inasmuch as petitioners herein are not domiciled in the
By swearing to the supreme authority of the Republic, Philippines and, hence, non-resident aliens the Court
the person implicitly renounces his foreign citizenship. cannot assume and exercise jurisdiction over the status,
under either the nationality theory or the domiciliary
Plainly, from Section 3, Rep. Act No. 9225 stayed clear out
theory.
of the problem of dual allegiance and shifted the burden

Private International Law Page 12


Article 335 of the Civil Code of the Philippines, provides As such, the petitioners Miguela Tan Suat and Chan Po Lan
that: were declared a Filipino citizens by marriage and the
"The following cannot adopt: Commissioner of Immigration is hereby ordered to cancel
xxx xxx xxx
the necessary alien certificate of registration and
(4) Non-resident aliens;".
xxx xxx xxx immigrant certificate of residence of the petitioner and to
This legal provisions is too clear to require interpretation. issue the corresponding identification card.

Since adoption is a proceedings in rem, no court may The Solicitor General filed the instant petitions instead,
entertain unless it has jurisdiction, not only over the including the Commissioner of Immigration as co-
subject matter of the case and over the parties, but also petitioner in view of the fact that the dispositive parts of
over the res, which is the personal status of Baby Rose as the decisions of the lower court are addressed to him for
well as that of petitioners herein. compliance.

Art. 15 adheres to the theory that jurisdiction over the Issue:


status of a natural person is determined by the latters'
Whether or not a person claiming to be a citizen may get a
nationality. Pursuant to this theory, the Court has
judicial declaration of citizenship.
jurisdiction over the status of Baby Rose, she being a
citizen of the Philippines, but not over the status of the
Held:
petitioners, who are foreigners.
Under Philippine laws, there can be no action or
Under the Philippines political law, which is patterned
proceeding for the judicial declaration of the citizenship
after the Anglo-American legal system, the Court have,
of an individual. Courts of justice exist for the settlement
likewise, adopted the latter's view to the effect that
of justiciable controversies, which imply a given right,
personal status, in general, is determined by and/or
legally demandable and enforceable, an act or omission
subject to the jurisdiction of the domiciliary law
violative of said right, and a remedy, granted or sanctioned
(Restatement of the Law of Conflict of Laws, p. 86; The
by law, for said breach of right.
Conflict of Laws by Beale, Vol. I, p. 305, Vol. II, pp. 713-
714). As an incident only of the adjudication of the right of the
parties to a controversy, the court may pass upon, and
This, perhaps, is the reason why our Civil Code does not
make a pronouncement relative to, their status.
permit adoption by non-resident aliens, and we have
Otherwise, such a pronouncement is beyond judicial
consistently refused to recognize the validity of foreign
power. Thus, for instance, no action or proceeding may be
decrees of divorce regardless of the grounds upon
instituted for a declaration to the effect that plaintiff or
which the same are based involving citizens of the
petitioner is married, or single, or a legitimate child,
Philippines who are not bona fide residents of the forum,
although a finding thereon may be made as a necessary
even when our laws authorized absolute divorce in the
premise to justify a given relief available only to one
Philippines (Ramirez v. Gmur, 42 Phil. 855; Gonayeb v.
enjoying said status. At times, the law permits the
Hashim, 30 Phil. 22; Cousine Hix v. Fleumer, 55 Phil. 851;
acquisition of a given status, such as naturalization by
Barretto Gonzales v. Gonzales, 58 Phil. 67; Recto v. Harden,
judicial decree. But there is no similar legislation
L-6897, Nov. 29, 1955)".
authorizing the institution of a judicial proceeding to
declare that a given person is a Filipino Citizen. (Tan v.
Republic v. Maddela
Republic, L-14159, April 18, 1960).
27 SCRA 702
Burca v. Republic
This is a petition to have the petitioners Miguela Tan Suat,
51 SCRA 248
and Chan Po Lan, all Chinese Nationals, to be declared a
Filipino citizens.
1st case: Burca not granted citizenship because such power
is granted under the executive branch.
That sometime in the year 1937 Miguela was legally
married to Sy Ing Seng, a Filipino citizen; likewise, in the
Moya Lim Yao: alien wife deemed ipso facto Filipino citizen
year 1961, Chan Po Lan was legally married to Cu Bon Piao,
as long as no disqualifications, no need to prove the
a Filipino citizen; and that the petitioners have all the
qualifications;
qualifications and none of the disqualifications to become
Filipino citizens. Burca motion for reconsideration:

The court inquired from Fiscal Veluz, who represents the Petitioner seeks reconsideration of the decision in this case
Solicitor General, if he has any opposition to the petition to which reversed that of the Court of First Instance of Leyte
which the Fiscal answered that he has no opposition. The declaring her a citizen of the Philippines, the said court
Court had it announced to the public if there is any have found her to be married to a Filipino citizen and to
opposition to the petitions of both to be declared a Filipino possess all the qualifications and none of the
citizen and nobody in the crowded courtroom registered disqualifications to become Filipino citizen enumerated in
his opposition. the Naturalization Law.

Private International Law Page 13


Issue: 41 SCRA 292

Whether or not a court may grant a judicial declaration of Facts:


citizenship.
On 8 February 1961, Lau Yuen Yeung applied for a
Held: passport visa to enter the Philippines as a non-immigrant.
In the interrogation made in connection with her
The Court cannot grant petitioner-appellee's prayer for the application for a temporary visitor's visa to enter the
affirmance of the trial court's judgment declaring her a Philippines, she stated that she was a Chinese residing at
Filipino citizen. It must be noted that the sole and only Kowloon, Hongkong, and that she desired to take a
purpose of the petition is to have petitioner declared a pleasure trip to the Philippines to visit her great-grand-
Filipino citizen. Under Philippine laws there can be no uncle Lau Ching Ping for a period of one month. She was
judicial action or proceeding for the declaration of the permitted to come into the Philippines on 13 March 1961,
citizenship of an individual. It is as an incident only of the and was permitted to stay for a period of one month
adjudication of the rights of the parties to a controversy, which would expire on 13 April 1961. On the date of her
that the courts may pass upon, and make a arrival, Asher Y, Cheng filed a bond in the amount of
pronouncement relative to, their status. P1,000.00 to undertake, among others, that said Lau Yuen
Yeung would actually depart from the Philippines on or
In Moy Ya Lim Yao, the Court emphasized the
before the expiration of her authorized period of stay in
administrative procedure that needs to be followed in the
this country or within the period as in his discretion the
Bureau Immigration regarding the steps to be taken by an
Commissioner of Immigration or his authorized
alien woman married to a Filipino for the cancellation of
representative might properly allow. After repeated
her alien certificate of registration, and thus secure
extensions, Lau Yuen Yeung was allowed to stay in the
recognition of her status Filipino citizen. Such a procedure
Philippines up to 13 February 1962. On 25 January 1962,
could be availed of Petitioner. Judicial recourse would be
she contracted marriage with Moy Ya Lim Yao alias
avoidable to Petitioner in case of an adverse action by the
Edilberto Aguinaldo Lim an alleged Filipino citizen.
Immigration Commissioner.
Because of the contemplated action of the Commissioner
of Immigration to confiscate her bond and order her arrest
However, if the decision of an administrative agency on
and immediate deportation, after the expiration of her
the question of citizenship, is affirmed by this Court on
authorized stay, she brought an action for injunction with
the ground that the same is supported by substantial
preliminary injunction. At the hearing which took place
evidence on the whole record, there appears to be no
one and a half years after her arrival, it was admitted that
valid reason why such finding should have no conclusive
Lau Yuen Yeung could not write either English or Tagalog.
effect in other cases, where the same issue is involved.
Except for a few words, she could not speak either English
The same observation holds true with respect to a decision
or Tagalog. She could not name any Filipino neighbor, with
of a court on the matter of citizenship as a material matter
a Filipino name except one, Rosa. She did not know the
in issue in the case before it, which is affirmed by this
names of her brothers-in-law, or sisters-in-law. The Court
Court. For the "effective operation of courts in the social
of First Instance of Manila (Civil Case 49705) denied the
and economic scheme requires that their decision have the
prayer for preliminary injunction. Moya Lim Yao and Lau
respect of and be observed by the parties, the general
Yuen Yeung appealed.
public and the courts themselves. According insufficient
weight to prior decisions encourages disrespect and
Issue:
disregard of courts and their decisions and invites
litigation" (Clear, Res Judicata Reexamined, 57 Yale Law Whether Lau Yuen Yeung ipso facto became a Filipino
Journal, 345). citizen upon her marriage to a Filipino citizen.

Wherefore, the Court declared it to be a sound rule, that Held:


where citizenship of a party in a case is definitely
resolved by a court or by an administrative agency, as a Lau Yuen Yeung, was declared to have become a Filipino
material issue in controversy, after a full-blown hearing, citizen from and by virtue of her marriage to Moy Ya Lim
with the act participation of the Solicitor General or his Yao al as Edilberto Aguinaldo Lim, a Filipino citizen of 25
authority representative, and this finding on the January 1962.
Citizenship of the party is affirmed by this Court, the
decision on the matter shows constitute conclusive proof Under Section 15 of Commonwealth Act 473, an alien
of such person's citizenship, in another case or woman marrying a Filipino, native born or naturalized,
proceeding. But it is made clear that in instance will a becomes ipso facto a Filipina provided she is not
decision on the question of citizenship in such cases be disqualified to be a citizen of the Philippines under
considered conclusive or binding in any other case Section 4 of the same law. Likewise, an alien woman
proceeding, unless obtained in accordance with the married to an alien who is subsequently naturalized here
procedure herein stated. follows the Philippine citizenship of her husband the
moment he takes his oath as Filipino citizen, provided
Moy Ya Lim Yao v. Commissioner that she does not suffer from any of the disqualifications
under said Section 4. Whether the alien woman requires

Private International Law Page 14


to undergo the naturalization proceedings, Section 15 is a alliance with the said foreign country;
parallel provision to Section 16. Thus, if the widow of an or
applicant for naturalization as Filipino, who dies during the b. The said foreign country maintains
proceedings, is not required to go through a naturalization armed forces on Philippine territory
proceedings, in order to be considered as a Filipino citizen with the consent of the Republic of
hereof, it should follow that the wife of a living Filipino the Philippines: Provided, That the
cannot be denied the same privilege. This is plain common Filipino citizen concerned, at the time
sense and there is absolutely no evidence that the of rendering said service, or
Legislature intended to treat them differently. As the laws acceptance of said commission, and
of our country, both substantive and procedural, stand taking the oath of allegiance incident
today, there is no such procedure (a substitute for thereto, states that he does so only in
naturalization proceeding to enable the alien wife of a connection with his service to said
Philippine citizen to have the matter of her own foreign country: And provided, finally,
citizenship settled and established so that she may not That any Filipino citizen who is
have to be called upon to prove it everytime she has to rendering service to, or is
perform an act or enter into a transaction or business or commissioned in, the armed forces of a
exercise a right reserved only to Filipinos), but such is no foreign country under any of the
proof that the citizenship is not vested as of the date of circumstances mentioned in paragraph
marriage or the husband's acquisition of citizenship, as the (a) or (b), shall not be permitted to
case may be, for the truth is that the situation obtains even participate nor vote in any election of
as to native-born Filipinos. Everytime the citizenship of a the Republic of the Philippines during
person is material or indispensible in a judicial or the period of his service to, or
administrative case, Whatever the corresponding court or commission in, the armed forces of
administrative authority decides therein as to such said foreign country. Upon his
citizenship is generally not considered as res adjudicata, discharge from the service of the said
hence it has to be threshed out again and again as the foreign country, he shall be
occasion may demand. automatically entitled to the full
enjoyment of his civil and political
Nota bene: rights as a Filipino citizen;

There are two laws, which govern the Loss of Philippine 5. By cancellation of the of the certificates of
citizenship. These are Commonwealth Act No. 63 and naturalization;
Commonwealth Act No. 473. The former applies to both 6. By having been declared by competent authority, a
natural-born and naturalized citizens and the latter applies deserter of the Philippine armed forces in time of
only to naturalized citizens. war, unless subsequently, a plenary pardon or
amnesty has been granted; and
As stated in Commonwealth Act. No. 63, 7. In the case of a woman, upon her marriage to a
foreigner if, by virtue of the laws in force in her
A Filipino citizen may lose his citizenship in any of the husband's country, she acquires his nationality.
following ways and/or events:
Commonwealth Act No. 473, Section 18 of said law
1. By naturalization in a foreign country; provides that:
2. By express renunciation of citizenship;
3. By subscribing to an oath of allegiance to support A naturalization certificate may be cancelled by a
the constitution or laws of a foreign country upon competent judge on any of the following grounds:
attaining twenty-one years of age or more: Provided,
however, That a Filipino may not divest himself of 1. If it is shown that said naturalization certificate
Philippine citizenship in any manner while the was obtained fraudulently or illegally;
Republic of the Philippines is at war with any 2. If the person naturalized shall, within five years
country; next following the issuance of said
4. By rendering services to, or accepting commission naturalization certificated, return to his native
in, the armed forces of a foreign country: Provided, country or to some foreign country and
That the rendering of service to, or the acceptance of establish his permanent residence there:
such commission in, the armed forces of a foreign Provided, That the fact of the person naturalized
country, and the taking of an oath of allegiance remaining for more than one year in his native
incident thereto, with the consent of the Republic of country or the country of his former nationality,
the Philippines, shall not divest a Filipino of his or two years in any other foreign country, shall
Philippine citizenship if either of the following be considered prima facie evidence of his
circumstances is present: intention of taking up his permanent residence in
the same;
a. The Republic of the Philippines has a 3. If the petition was made on an invalid
defensive and/or offensive pact of declaration of intention

Private International Law Page 15


4. If it is shown that the minor children of the Held:
person naturalized failed to graduate from a
public high school recognized by the Office of First issue:
Private Education of the Philippines, where
The order of February 9, 1966 (oath-taking) had not
Philippine history, government and civics are
and up to the present has not become final and
taught as part of the school curriculum, through
executory in view of the appeal duly taken by the
the fault of their parents either by neglecting to
Government.
support them or by transferring them to another
school or schools. A certified copy of the decree
2nd Issue:
of naturalization certificate shall be forwarded by
the Clerk of Court of the Department of Interior It is argued that the permission is not required by our
and the Bureau of Justice. laws and that the naturalization of an alien, as a citizen of
5. If it is shown that the naturalized citizen has the Philippines, is governed exclusively by such laws and
allowed himself to be used as a dummy in cannot be controlled by any foreign law. However, the
violation of the constitutional provisions question of how a Chinese citizen may strip himself of
requiring Philippine citizenship as a requisite for that status is necessarily governed pursuant to Articles
the exercise, use or enjoyment of a right, 15 and 16 of our Civil Code by the laws of China, not by
franchise or privilege. those of the Philippines.

The main reason why a decision in a naturalization As a consequence, a Chinese national cannot be
proceeding is not res judicata is because such is not a naturalized as a citizen of the Philippines, unless he has
judicial adversarial proceeding. Similarly, estoppel or complied with the laws of Nationalist China requiring
laches cannot apply to the government in action for the previous permission of its Minister of the Interior for the
cancellation of a certificate of naturalization, since it is a renunciation of nationality.
known principle that the government is never estopped by
the mistakes on the part of its agents. Section 12 of Commonwealth Act No. 473
provides, however, that before the naturalization
However, according to the Constitution, marriage to an certificate is issued, the petitioner shall
alien would not automatically divest a person of his "solemnly swear," inter alia, that he renounces
citizenship, unless he or she performs certain acts or "absolutely and forever all allegiance and fidelity
omission which would result to the loss of his or her to any foreign prince, potentate" and particularly
citizenship. This provision, however, is not retroactive; to the state "of which" he is "a subject or
thus, does not repatriate those who lost their Philippine citizen." The obvious purpose of this
citizenship by marriage under the 1935 and 1973 requirement is to divest him of his former
Constitutions. nationality, before acquiring Philippine
citizenship, because, otherwise, he would have
Oh Hek How v. Republic
two nationalities and owe allegiance to two (2)
29 SCRA 94 distinct sovereignties, which our laws do not
permit, except that, pursuant to Republic Act No.
Petitioner Oh Hek How having been granted naturalization 2639, "the acquisition of citizenship by a natural-
through his petition filed a motion alleging that he had born Filipino citizen from one of the Iberian and
complied with the requirements of Republic Act No. 530 any friendly democratic Ibero-American
and praying that he be allowed to take his oath of countries shall not produce loss or forfeiture of
allegiance as such citizen and issued the corresponding his Philippine citizenship, if the law of that
certificate of naturalization. country grants the same privilege to its citizens
and such had been agreed upon by treaty
The Court of First Instance of Zamboanga del Norte issued between the Philippines and the foreign country
forthwith an order authorizing the taking of said oath. On from which citizenship is acquired."
that same date, petitioner took it and the certificate of
naturalization was issued to him. Tecson v. Comelec

The Government seasonably gave notice of its intention to 424 SCRA 277
appeal from said order of February 9, 1966 and filed its
record on appeal among the grounds that the oath was Facts:
taken prior to judgment having been final and executor.
On 31 December 2003, Ronald Allan Kelly Poe, also
Issue: known as Fernando Poe, Jr. (FPJ), filed his certificate of
candidacy for the position of President of the Republic of
Is the oath valid? the Philippines under the Koalisyon ng Nagkakaisang
Pilipino (KNP) Party, in the 2004 national elections. In his
Whether or not a permission to renounce citizenship is certificate of candidacy, FPJ, representing himself to be a
necessary from the Minister of the Interior of Nationalist natural-born citizen of the Philippines, stated his name to
China. be "Fernando Jr.," or "Ronald Allan" Poe, his date of birth

Private International Law Page 16


to be 20 August 1939 and his place of birth to be Manila. and prohibition before the RTC of Manila. A motion to
Victorino X. Fornier, initiated, on 9 January 2004, a petition dismiss was filed but denied.
before the Commission on Elections (COMELEC) to
Petitioners, claim that respondent is an alien. In support of
disqualify FPJ and to deny due course or to cancel his
their position, petitioners point out that Santiago
certificate of candidacy upon the thesis that FPJ made a Gatchalian's marriage with Chu Gim Tee in China as well as
material misrepresentation in his certificate of candidacy the marriage of Francisco (father of William) Gatchalian to
by claiming to be a natural-born Filipino citizen when in Ong Chiu Kiok, likewise in China, were not supported by
truth, according to Fornier, his parents were foreigners; any evidence other than their own self-serving testimony
his mother, Bessie Kelley Poe, was an American, and his nor was there any showing what the laws of China were. It
father, Allan Poe, was a Spanish national, being the son of is the postulate advanced by petitioners that for the said
marriages to be valid in this country, it should have been
Lorenzo Pou, a Spanish subject. And even if Allan F. Poe
shown that they were valid by the laws of China wherein
was a Filipino citizen, he could not have transmitted his the same were contracted. There being none, petitioners
Filipino citizenship to FPJ, the latter being an illegitimate conclude that the aforesaid marriages cannot be
child of an alien mother. Fornier based the allegation of considered valid. Hence, Santiago's children, including
the illegitimate birth of FPJ on two assertions: (1) Allan F. Francisco, followed the citizenship of their mother, having
Poe contracted a prior marriage to a certain Paulita Gomez been born outside of a valid marriage. Similarly, the
before his marriage to Bessie Kelley and, (2) even if no validity of the Francisco's marriage not having been
demonstrated, William and Johnson followed the
such prior marriage had existed, Allan F. Poe, married
citizenship of their mother, a Chinese national.
Bessie Kelly only a year after the birth of FPJ.
Issue:
Issue:
Whether or not William Gatchalian is to be declared as a
Whether FPJ was a natural born citizen, so as to be allowed Filipino citizen
to run for the offcie of the President of the Philippines.
Held:
Held:
In Miciano vs. Brimo (50 Phil. 867 [1924]; Lim and Lim vs.
Any conclusion on the Filipino citizenship of Lorenzo Pou Collector of Customs, 36 Phil. 472; Yam Ka Lim vs. Collector
could only be drawn from the presumption that having of Customs, 30 Phil. 46 [1915]), this Court held that in the
died in 1954 at 84 years old, Lorenzo would have been absence of evidence to the contrary, foreign laws on a
particular subject are presumed to be the same as those
born sometime in the year 1870, when the Philippines
of the Philippines. In the case at bar, there being no proof
was under Spanish rule, and that San Carlos, Pangasinan, of Chinese law relating to marriage, there arises the
his place of residence upon his death in 1954, in the presumption that it is the same as that of Philippine law.
absence of any other evidence, could have well been his
place of residence before death, such that Lorenzo Pou The lack of proof of Chinese law on the matter cannot be
would have benefited from the "en masse Filipinization" blamed on Santiago Gatchalian much more on respondent
William Gatchalian who was then a twelve-year old minor.
that the Philippine Bill had effected in 1902. That
The fact is, as records indicate, Santiago was not pressed
citizenship (of Lorenzo Pou), if acquired, would thereby
by the Citizenship Investigation Board to prove the laws
extend to his son, Allan F. Poe, father of respondent FPJ. of China relating to marriage, having been content with
The 1935 Constitution, during which regime respondent the testimony of Santiago that the Marriage Certificate
FPJ has seen first light, confers citizenship to all persons was lost or destroyed during the Japanese occupation of
whose fathers are Filipino citizens regardless of whether China.
such children are legitimate or illegitimate.
The testimonies of Santiago Gatchalian and Francisco
Gatchalian before the Philippine consular and immigration
Board of Commissioners v. Dela Rosa authorities regarding their marriages, birth and
Facts: relationship to each other are not self-serving but are
admissible in evidence as statements or declarations
On July 12, 1960, Santiago Gatchalian, grandfather of regarding family reputation or tradition in matters of
William Gatchalian, was recognized by the Bureau of pedigree (Sec. 34, Rule 130).
Immigration as a native born Filipino citizen following the
citizenship of natural mother Mariana Gatchalian. On June Philippine law, following the lex loci celebrationis,
27, 1961, Willian, then twelve years old, arrives in Manila adheres to the rule that a marriage formally valid where
from Hongkong together with a daughter and a son of celebrated is valid everywhere. Referring to marriages
Santiago. They had with them certificate of registration contracted abroad, Art. 71 of the Civil Code (now Art. 26 of
and identity issued by the Philippine consulate in the Family Code) provides that "all marriages performed
Hongkong based on a cablegram bearing the signature of outside of the Philippines in accordance with the laws in
the secretary of foreign affairs, Felixberto Serrano, and force in the country where they were performed, and valid
sought admission as Filipino citizens. there as such, shall also be valid in this country . . ."

On August 15, 1990, the Commission on Immigration and And any doubt as to the validity of the matrimonial unity
Deportatiion ordered the arrest of William and was and the extent as to how far the validity of such marriage
released upon posting P 200,000 cash bond. Thus on the may be extended to the consequences of the coverture is
29th of the same month, he filed a petition for certiorari answered by Art. 220 of the Civil Code in this manner: "In
case of doubt, all presumptions favor the solidarity of the
family. Thus, every intendment of law or facts leans

Private International Law Page 17


toward the validity of marriage, the indissolubility of the res judicata and dismissing the quo warranto
marriage bonds, the legitimacy of children, the proceedings instituted by the said Gregorio
community of property during marriage, the authority of Nuval against Norbeto Guray and others, with
parents over their children, and the validity of defense for
costs against the petitioner.
any member of the family in case of unlawful
Gregorio Nuval filed, in his dual capacity as a
aggression." (Emphasis supplied). Bearing in mind the
"processual presumption" enunciated in Miciano and voter duly qualified and registered in the election
other cases, he who asserts that the marriage is not valid list of the municipality of Luna and as a duly
under our law bears the burden of proof to present the registered candidate for the office of municipal
foreign law. president of said municipality, a petition against
Norberto Guray asking for the exclusion of his
Having declared the assailed marriages as valid, name from the election list of said municipality,
respondent William Gatchalian follows the citizenship of
not being a qualified voter of said municipality
his father Francisco, a Filipino, as a legitimate child of the
latter. Francisco, in turn is likewise a Filipino being the and he had not resided therein for six months as
legitimate child of Santiago Gatchalian who (the latter) is required by section 431 of the said
admittedly a Filipino citizen whose Philippine citizenship Administrative Code.
was recognized by the Bureau of Immigration in an order Norbeto Guray was elected to the office of
dated July 12, 1960. municipal president of Luna by a plurality of
votes, Gregorio Nuval obtaining second place.
Finally, respondent William Gatchalian belongs to the class Gregorio Nuval filed the present action of quo
of Filipino citizens who became as such at the time of the
warranto asking that Norberto Guray be
adoption of the Constitution. . . .
declared ineligible had a legal residence of one
Nota bene: for Gatchalian year previous to the election as required by
section 2174 of the said Administrative Code in
In Moy Ya Lim vs. Commissioner of Immigration (41 SCRA order to be eligible to an elective municipal
292 [1971]) and in Lee vs. Commissioner of Immigration office.
(supra), this Court declared that: Norberto Guray had resided in the municipality
of Luna, his birthplace, where he had married
Everytime the citizenship of a person is material or
and had held the office of municipal treasurer.
indispensable in a judicial or administrative case, whatever
the corresponding court or administrative authority On that date he was appointed municipal
decides therein as to such citizenship is generally not treasurer of Balaoan, Province of La Union. The
considered as res adjudicata, hence it has to be threshed rules of the provincial treasurer of La Union, to
out again and again as the occasion may demand. which Norberto Guray was subject as such
municipal treasurer, require that municipality
An exception to the above rule was laid by this Court in
treasurers live continuously in the municipality
Burca vs. Republic (51 SCRA 248 [1973]), viz:
where they perform they official duties, in order
We declare it to be a sound rule that where the to be able to give an account of their acts as such
citizenship of a party in a case is definitely resolved by a treasurers at any time. In order to qualify and be
court or by an administrative agency, as a material issue in a position to vote as an elector in Balaoan in
in the controversy, after a full-blown hearing with the the general election of 1925, Norberto Guray
active participation of the Solicitor General or his asked for the cancellation of his name in the
authorized representative, and this finding or the
election lists of Luna, where he had voted in the
citizenship of the party is affirmed by this Court, the
decision on the matter shall constitute conclusive proof of general elections of 1922, alleging as a ground
such party's citizenship in any other case or proceeding. therefore the following: "On the ground of
But it is made clear that in no instance will a decision on transfer of any residence which took place on
the question of citizenship in such cases be considered the 28th day of June, 1922. My correct and new
conclusive or binding in any other case or proceeding, address is Poblacion, Balaoan, La Union;" and in
unless obtained in accordance with the procedure herein order to be registered in the subscribed affidavit
stated.
Exhibit F-1 before the board of election
Thus, in order that the doctrine of res judicata may be inspectors of precinct No. 1 of Balaoan, by virtue
applied in cases of citizenship, the following must be of which he was registered as an elector of the
present: 1) a person's citizenship must be raised as a said precinct, having made use of the right of
material issue in a controversy where said person is a suffrage in said municipality in the general
party; 2) the Solicitor General or his authorized elections of 1925. In his cedula certificates issued
representative took active part in the resolution thereof,
by himself as municipal treasurer of Balaoan
and 3) the finding or citizenship is affirmed by this Court.
from the year 1923 to 1928, included, he made it
NUVAL VS. GURRAY appear that his residence was the residential
district of Balaoan. In the year 1926, his wife and
Facts: children who, up to that time, had lived in the
municipality of Balaoan, went back to live in the
This appeal was taken by the petitioner Gregorio town of Luna in the house of his wife's parents,
Nuval from the judgment of the Court of First due to the high cost of living in that municipality.
Instance of La Union, upholding the defense of Norberto Guray used to go home to Luna in the

Private International Law Page 18


afternoons after office hours, and there he That Arthur Graydon Moody executed in the
passed the nights with his family. His children Philippine Islands a will, by virtue of which will,
studied in the public school of Luna. In January, he bequeathed all his property to his only sister,
Ida M. Palmer, who then was and still is a citizen
1927, he commenced the construction of a
and resident of the State of New York, United
house of strong materials in Luna, which has
States of America.
not yet been completed, and neither be nor his
family has lived in it. On February 1, 1928,
That on February 24,1931, a petition for
Norberto Guray applied for and obtained appointment of special administrator of the
vacation leave to be spent in Luna, and on the estate of the deceased Arthur Graydon Moody
16th of the same month he filed his resignation was filed by W. Maxwell Thebaut with the Court
by telegraph, which was accepted on the same of First Instance of Manila.
day, also by telegraph. Nothwithstanding that he
was already provided with a cedula by himself as That subsequently or on April 10, 1931, a
municipal treasurer of Balaoan on January 31, petition to the will of the deceased Arthur
Graydon Moody, and the same was, after
1928, declaring him resident of said town, he
hearing, duly probated by the court in a decree
obtained another cedula from the municipality dated May 5, 1931.
of Luna on February 20, 1928, which was dated
January 15, 1928, in which it is presented that he That on July 14, 1931, Ida M. Palmer was
resided in the barrio of Victoria, municipality of declared to be the sole and only heiress of the
Luna, Province of La Union. On February 23, deceased Arthur Graydon Moody
1928, Norberto Guray applied for and obtained
the cancellation of his name in the election list of That the property left by the late Arthur
the municipality of Balaoan, and on April 14, Graydon Moody consisted principally of bonds
1928, he applied for registration as a voter in and shares of stock of corporations organized
Luna, alleging that he had been residing in said under the laws of the Philippine Islands, bank
deposits and other personal properties.
municipality for thirty years. For this purpose he
made of the cedula certificate antedated.
That on July 22, 1931, the Bureau of Internal
Revenue prepared for the estate of the late
Arthur Graydon Moody an inheritance tax
return.
Issue:

Whether or not Norberto Guray had the legal residence of That on September 9, 1931, an income tax return
one year immediately prior to the general elections of June for the fractional period from January 1, 1931 to
June 30, 1931, was also prepared by the Bureau
5, 1928, in order to be eligible to the office of municipal
of Internal Revenue for the estate of the said
president of Luna, Province of La Union? deceased Arthur Graydon Moody.

Ruling: That on December 3, 1931, the committee on


claims and appraisals filed with the court its
It is an established rule that "where a voter abandons his report.
residence in a state and acquires one in another state, he
cannot again vote in the state of his former residence That on November 4, 1931, and in answer to the
until he has qualified by a new period of residence" (20 letter mentioned in the preceding paragraph, the
Corpus Juris, p. 71, par. 28). "The term 'residence' as so Bureau of Internal Revenue addressed to the
used is synonymous with 'domicile,' which imports not attorney for Ida M. Palmer another letter, copy of
only intention to reside in a fixed place, but also personal which marked Exhibit NN is hereto attached and
presence in that place, coupled with conduct indicative of made a part hereof.
such intention." (People vs. Bender, 144 N. Y. S., 145.)
That the estate of the late Arthur Graydon
Since Norberto Guray abandoned his first residence in the Moody paid under protest the sum of P50,000
municipality of Luna and acquired another in Balaoan, in on July 22, 1931, and the other sum of
order to vote and be a candidate in the municipality of P40,019.75 on January 19, 1932, making
Luna, he needed to reacquire residence in the latter assessment for inheritance tax and the sum of
municipality for the length of time prescribed by the law, P13,001.41 covers the assessment for income tax
and for such purpose, he needed not only the intention to against said estate.
do so, but his personal presence in said municipality.
That on January 21, 1932, the Collector of
VELILLA VS. POSADA Internal Revenue overruled the protest made by
Ida M. Palmer through her attorney.
Facts:
The parties reserve their right to introduce
That Arthur Graydon Moody died in Calcutta, additional evidence at the hearing of the present
India, on February 18, 1931. case.

Private International Law Page 19


Manila, August 15, 1933. Administration of the United States of America. He
has no record of conviction and it is his intention to
In addition to the foregoing agreed statement of renounce his allegiance to the U.S.A. After hearing,
facts, both parties introduced oral and the court a quo rendered decision denying the
documentary evidence from which it appears petition on the ground that petitioner did not have
that Arthur G. Moody, an American citizen, came
the residence required by law six months before he
to the Philippine Islands in 1902 or 1903 and
filed his petition for reacquisition of Philippine
engaged actively in business in these Islands up
to the time of his death in Calcutta, India, on citizenship.
February 18, 1931. He had no business
elsewhere and at the time of his death left an Ruling:
estate consisting principally of bonds and shares
of stock of corporations organized under the The court a quo, in denying the petition, made the
laws of the Philippine Islands, bank deposits and following comment: "One of the qualifications for
other intangibles and personal property valued
reacquiring Philippine citizenship is that the
by the commissioners of appraisal and claims at
applicant 'shall have resided in the Philippines at
P609,767.58 and by the Collector of Internal
Revenue for the purposes of inheritance tax at least six months before he applies for naturalization'
P653,657.47. All of said property at the time of [Section 3(1), Commonwealth Act No. 63]. This
his death was located and had its situs within 'residence' requirement in cases of naturalization, has
the Philippine Islands. So far as this record already been interpreted to mean the actual or
shows, he left no property of any kind located constructive permanent home otherwise known as
anywhere else. In his will,he made a statement
legal residence or domicile (Wilfredo Uytengsu vs.
that: Arthur G. Moody, a citizen of the United
States of America, residing in the Philippine Republic of the Philippines, 95 Phil. 890). A place in a
Islands, hereby publish and declare the following country or state where he lives and stays
as my last Will and Testament . . .. permanently, and to which he intends to return after
a temporary absence, no matter how long, is his
Ruling: domicile. In other words domicile is characterized by
animus manendi. So an alien who has been admitted
To effect the abandonment of one's domicile, there must into this country as a temporary visitor, either for
be a deliberate and provable choice of a new domicile, business or pleasure, or for reasons of health, though
coupled with actual residence in the place chosen, with a actually present in this country cannot be said to have
declared or provable intent that it should be one's fixed
established his domicile here because the period of
and permanent place of abode, one's home. There is a
his stay is only temporary in nature and must leave
complete dearth of evidence in the record that Moody
ever established a new domicile in a foreign country. when the purpose of his coming is accomplished. In
the present case, petitioner, who is presently a
Finding no merit in any of the assignments of error of the citizen of the United States of America, was admitted
appellant, the court affirm the judgment of the trial court, into this country as a temporary visitor, a status he
first, because the property in the estate of Arthur G. has maintained at the time of the filing of the
Moody at the time of his death was located and had its present petition for reacquisition of Philippine
situs within the Philippine Islands and, second, because his citizenship and which continues up to the present.
legal domicile up to the time of his death was within the
Such being the case, he has not complied with the
Philippine Islands.
specific requirement of law regarding six months
residence before filing his present petition."

CAASI VS. COURT OF APPEALS


UJANO VS. REPUBLIC
Facts:
Facts:
These two cases were consolidated because they
Petitioner seeks to reacquire his Philippine have the same objective; the disqualification
citizenship in a petition filed before the Court of First under Section 68 of the Omnibus Election Code
Instance of Ilocos Sur. Petitioner was born 66 years of the private respondent, Merito Miguel for the
ago of Filipino parents in Magsingal Ilocos Sur. He is position of municipal mayor of Bolinao,
married to Maxima O. Ujano with whom he has one Pangasinan, to which he was elected in the local
son, Prospero, who is now of legal age. He left the elections of January 18, 1988, on the ground that
Philippines for the United States of America in 1927 he is a green card holder, hence, a permanent
where after a residence of more than 20 years he resident of the United States of America, not of
acquired American citizenship by naturalization. He Bolinao.
In his answer to both petitions, Miguel admitted
returned to the Philippines on November 10, 1960 to
that he holds a green card issued to him by the
which he was admitted merely for a temporary stay.
US Immigration Service, but he denied that he is
He owns an agricultural land and a residential house
a permanent resident of the United States. He
situated in Magsingal, Ilocos Sur. He receives a
allegedly obtained the green card for
monthly pension from the Social Security
convenience in order that he may freely enter

Private International Law Page 20


the United States for his periodic medical The mischief which this provision reproduced verbatim
examination and to visit his children there. He from the 1973 Constitution seeks to prevent is the
alleged that he is a permanent resident of possibility of a "stranger or newcomer unacquainted with
Bolinao, Pangasinan, that he voted in all previous the conditions and needs of a community and not
elections, including the plebiscite on February identified with the latter, from an elective office to serve
2,1987 for the ratification of the 1987 that community."
Constitution, and the congressional elections on
May 18,1987. Petitioner Imelda Romualdez-Marcos filed her Certificate
of Candidacy for the position of Representative of the First
After hearing the consolidated petitions before District of Leyte with the Provincial Election Supervisor on
it, the COMELEC dismissed the petitions on the March 8, 1995.
ground that: The possession of a green card by
the respondent (Miguel) does not sufficiently Private respondent Cirilo Roy Montejo, the incumbent
establish that he has abandoned his residence in Representative of the First District of Leyte and a candidate
the Philippines. On the contrary, inspite (sic) of
for the same position, filed a "Petition for Cancellation and
his green card, Respondent has sufficiently
indicated his intention to continuously reside in Disqualification" with the Commission on Elections alleging
Bolinao as shown by his having voted in that petitioner did not meet the constitutional
successive elections in said municipality. As the requirement for residency. In his petition, private
respondent meets the basic requirements of respondent contended that Mrs. Marcos lacked the
citizenship and residence for candidates to Constitution's one year residency requirement for
elective local officials (sic) as provided for in candidates for the House of Representatives.
Section 42 of the Local Government Code, there
is no legal obstacle to his candidacy for mayor of
Held:
Bolinao, Pangasinan. (p. 12, Rollo, G.R. No.
84508).
So settled is the concept (of domicile) in our election law
that in these and other election law cases, this Court has
stated that the mere absence of an individual from his
permanent residence without the intention to abandon it
Issues: (1) whether or not a green card is proof that the
does not result in a loss or change of domicile.
holder is a permanent resident of the United States, and
It stands to reason therefore, that petitioner merely
(2) whether respondent Miguel had waived his status as a
committed an honest mistake in jotting the word "seven"
permanent resident of or immigrant to the U.S.A. prior to
the local elections on January 18, 1988. in the space provided for the residency qualification
requirement. It would be plainly ridiculous for a candidate
Ruling: Despite his vigorous disclaimer, Miguel's to deliberately and knowingly make a statement in a
immigration to the United States in 1984 constituted an certificate of candidacy which would lead to his or her
abandonment of his domicile and residence in the disqualification.
Philippines. For he did not go to the United States merely
to visit his children or his doctor there; he entered the Residence in the civil law is a material fact, referring to the
limited States with the intention to have there physical presence of a person in a place. A person can have
permanently as evidenced by his application for an two or more residences, such as a country residence and a
immigrant's (not a visitor's or tourist's) visa. Based on that
city residence. Residence is acquired by living in place; on
application of his, he was issued by the U.S. Government
the requisite green card or authority to reside there the other hand, domicile can exist without actually living in
permanently. the place. The important thing for domicile is that, once
residence has been established in one place, there be an
To be "qualified to run for elective office" in the intention to stay there permanently, even if residence is
Philippines, the law requires that the candidate who is a also established in some other place.
green card holder must have "waived his status as a
permanent resident or immigrant of a foreign country." For political purposes the concepts of residence and
Therefore, his act of filing a certificate of candidacy for domicile are dictated by the peculiar criteria of political
elective office in the Philippines, did not of itself constitute laws. As these concepts have evolved in our election law,
a waiver of his status as a permanent resident or
what has clearly and unequivocally emerged is the fact
immigrant of the United States. The waiver of his green
card should be manifested by some act or acts that residence for election purposes is used
independent of and done prior to filing his candidacy for synonymously with domicile.
elective office in this country. Without such prior waiver,
he was "disqualified to run for any elective office" (Sec. 68, In Nuval vs. Guray, the Court held that "the term
Omnibus Election Code). residence. . . is synonymous with domicile which imports
not only intention to reside in a fixed place, but also
MARCOS vs COMELEC personal presence in that place, coupled with conduct
indicative of such intention." Larena vs. Teves reiterated
(residence and domicile, for purposes of election laws are the same doctrine in a case involving the qualifications of
synonymous) the respondent therein to the post of Municipal President
of Dumaguete, Negros Oriental. Faypon vs. Quirino, held

Private International Law Page 21


that the absence from residence to pursue studies or The law specifically enumerates the legal grounds that
practice a profession or registration as a voter other than must be proved to exist by indubitable evidence to annul a
in the place where one is elected does not constitute loss marriage. In the case at bar, the annulment of the
of residence. So settled is the concept (of domicile) in our marriage in question was decreed upon the sole testimony
election law that in these and other election law cases, this of the husband who was expected to give testimony
Court has stated that the mere absence of an individual tending or aiming at securing the annulment of his
from his permanent residence without the intention to marriage he sought and seeks. Whether the wife is really
abandon it does not result in a loss or change of domicile. impotent cannot be deemed to have been satisfactorily
established because from the commencement of the
In Co vs. Electoral Tribunal of the House of proceedings until the entry of the decree she had
Representatives, this Court concluded that the framers of abstained from taking part therein.
the 1987 Constitution obviously adhered to the definition
given to the term residence in election law, regarding it as Although her refusal to be examined or failure to appear in
having the same meaning as domicile. court show indifference on her part, yet from such attitude
the presumption arising out of the suppression of evidence
A Person cannot have 2 domiciles. As long as the Domicile
could not arise or be inferred because women of this
was not lost, it continues to be the same until replaced by
country are by nature coy, bashful and shy and would not
a new one. Marcos did not overtedly abandon her
submit to a physical examination unless compelled to by
domicile since even if living in Malakanyang, she
competent authority.
constantly goes home to her domicile.

A physical examination in this case is not self-incriminating.


Prior to this: She is not charged with any offense . She is not being
compelled to be a witness against herself.
The civil code provides that the wife follows the domicile
of her husband.
Impotency being an abnormal condition should not be
presumed. The presumption is in favor of potency. The
JIMENEZ V. REPUBLIC
lone testimony of the husband that his wife is physically
109 PHIL 273
incapable of sexual intercourse is insufficient to tear
FACTS: asunder the ties that have bound them together as
husband and wife.

1. Plaintiff Joel Jimenez filed a complaint praying of a Recto v. Harden


decree annulling his marriage with Remedios 100 Phil 427
Canizares.
2. He claimed that the orifice of her genitals was too Facts:
small to allow the penetration of a male organ or
penis for copulation. Recto and Harden entered into a contract for professional
3. He also claimed that the condition of her genitals services wherein the latter engaged the services of the
existed at the time of marriage and continues to exist. former as her counsel against her husband for a claim in
4. The wife was summoned and served with a copy of their conjugal property. Mr. Harden previously filed for
the complaint but she did not file an answer. divorce against the Mrs. The Court awarded Mrs. Harden
5. The court entered an order requiring defendant to an amount totaling to almost 4 million pesos plus litis
submit to a physical examination by a competent lady expensae.
physician to determine her physical capacity for
copulation. Subsequently however, Mrs. Harden ordered her counsel
6. Defendant did not submit herself to the to vacate all orders and judgments rendered therein, and
examination and the court entered a decree annulling abandon and nullify all her claims to the conjugal
the marriage. partnership existing between her and Mr. Harden. Later,
7. The City Attorney filed a Motion for she entered into an amicable settlement with Mr. Harden
Reconsideration, among the grounds that the agreeing to a share of a lesser amount.
defendants impotency has not been satisfactorily
established as required by law; that she had not been Appellee counsel for Mrs. Harden alleged that the
physically examined because she refused to be purpose of the said instruments, executed by Mr. and Mrs.
examined. Harden, was to defeat the claim of the former for
attorneys fees, for which reason, he prayed that the court
ISSUE: grant him the necessary fees.

Whether or not the marriage may be annulled on the Appellants assail the contract for professional services as
strength only of the lone testimony of the husband who void, mainly, upon the grounds that:
claimed and testified that his wife is impotent.
1. that Mrs. Harden cannot bind the conjugal
partnership without her husbands consent; c
HELD:
2. that Article 1491 of the Civil Code of the
Philippines in effect prohibits contingent fees; c
3. that the contract in question has for its purpose
to secure a decree of divorce, allegedly in

Private International Law Page 22


violation of Articles 1305, 1352 and 1409 of the
Civil Code of the Philippines; BARNUEVO V. FUSTER
4. that the terms of said contract are harsh, 29 PHIL 606
inequitable and oppressive.
FACTS:
Held:
Gabriel and Constanza were married in Spain. Thereafter,
The first objection has no foundation in fact, for the Gabriel went to the Philippines, settled, and acquired real
contract in dispute does not seek to bind the conjugal and personal properties. Constanza later followed. A few
partnership. By virtue of said contract, Mrs. Harden merely years however, both parties executed a contract for their
bound herself or assumed the personal obligation to separation wherein the wife returned to Spain and has
pay, by way of contingent fees, 20% of her share in said agreed to be supported by the husband to be paid in
partnership. The contract neither gives, nor purports to Madrid, Spain. Eventually, the wife instituted a petition for
give, to the Appellee any right whatsoever, personal or divorce here in the Philippines against the husband. The
real, in and to her aforesaid share. The amount thereof is husband opposed the petition on the grounds that:
simply a basis for the computation of said fees.
a. neither the trial court nor any other court in the
For the same reason, the second objection is, likewise, Philippine Islands has jurisdiction over the
untenable. Moreover, it has already been held that subject matter of the complaint, because, as to
contingent fees are not prohibited in the Philippines and the allowance for support, since neither the
are impliedly sanctioned by our Cannons (No. 13) of plaintiff nor the defendant are residents of
Professional Ethics. (see, also, Ulanday vs. Manila Railroad Manila, or of any other place in the Philippine
Co., 45 Phil., 540, 554.) Such is, likewise, the rule in the Islands, the agreement upon the subject was
United States (Legal Ethics by Henry S. Drinker, p. 176). neither celebrated, nor was it to be fulfilled, in
the Philippine Islands;
b. and as to the divorce, because the action
in the United States, the great weight of authority therefore ought to be tried by the ecclesiastical
recognizes the validity of contracts for contingent fees, courts.
provided such contracts are not in contravention of public
policy, and it is only when the attorney has taken an In deciding the case, the Court of First Instance of the city
unfair or unreasonable advantage of his client that such a of Manila held itself to have jurisdiction, decreed the
claim is condemned. (See 5 Am. Jur. 359 et seq; suspension of life in common between the plaintiff and
Ballentine, Law Dictionary, 2nd ed., p. 276.) defendant;

Issue:
The third objection is not borne out, either by the
language of the contract between them, or by the intent of Do Philippine Courts have jurisdiction over the petition for
the parties thereto. Its purpose was not to secure a divorce?
divorce, or to facilitate or promote the procurement of a What law should apply in construing the term pesetas?
divorce. It merely sought to protect the interest of Mrs.
Harden in the conjugal partnership, during the pendency Held:
of a divorce suit she intended to file in the United States.
What is more, inasmuch as Mr. and Mrs. Harden are The lower court did not commit this error attributed to
admittedly citizens of the United States, their status and him. The defendant had not proved that he had
the dissolution thereof are governed pursuant to elsewhere a legal domicile other than that which he
Article 9 of the Civil Code of Spain (which was in force in manifestly had in the Philippines during the seventeen
the Philippines at the time of the execution of the years preceding the date of the complaint. On the
contract in question) and Article 15 of the Civil Code of contrary, it plainly appears, without proof to the contrary,
the Philippines by the laws of the United States, which that during this not inconsiderable period, extending from
sanction divorce. In short, the contract of services, the year 1892 until a month prior to the arrival of his wife
between Mrs. Harden and herein Appellee, is not contrary in the Philippines in March, 1909, he had constantly
to law, morals, good customs, public order or public policy. resided in the said Islands, had kept open house, and had
It is a basic principle that status, once established by the acquired in the city of Manila quite a little real property
personal law of the party, is given universal recognition. which is now the object of the division of the conjugal
Therefore, aliens can sue and be sued in our courts society.
subject to Philippine procedural law even on matters
relating to their status and capacity. However, the law to It has been established that defendant is domiciled in the
be applied by Philippine courts in determining their Philippines.
capacity and status is their personal law. Defendant, although a Spanish subject, was a resident of
these Islands. Article 26 of the Civil Code that he cites itself
The last objection is based upon principles of equity, but, provides that "Spaniards who change their domicile to a
pursuant thereto, one who seeks equity must come with foreign country, where they may be considered as natives
clean hands (Bastida, et al., vs. Dy Buncio & Co., 93 Phil., without other conditions than that of residents therein,
195; 30 C.J. S. 475), and Appellants have not done so, for shall be required, in order to preserve the Spanish
the circumstances surrounding the case show, to our nationality, to state that such is their wish before the
satisfaction, that their aforementioned agreements, Spanish diplomatic or consular agent, who must record
ostensibly for the settlement of the differences between them in the registry of Spanish residents, as well as their
husband and wife, were made for the purpose of spouses, should they be married, and any children they
circumventing or defeating the rights of herein Appellee, may have." From this provision, which is the exclusive and
under his above-quoted contract of services with Mrs. irrefutable law governing the defendant, we are to
Harden.

Private International Law Page 23


conclude that the domicile of the defendant and the subjects, they were residents of this city and had their
plaintiff is fully proven, irrespective of the Treaty of Paris. domicile herein.

Section 377 of the Code of Civil Procedure leaves to the The Courts of First Instance of the Philippine Islands have
election of the plaintiff the bringing of a personal action the power and jurisdiction to try actions for divorce. That
like the one at bar either in the place where the of the city of Manila did not lack jurisdiction by reason of
defendant may reside or be found, or in that where the the subject matter of the litigation.
plaintiff resides.
With respect to their property regime, the Foral Law
As held by the Husband:
presented by the husband in an affidavit, with which
conjugal partnership is known to be inexistent, cannot
That by the express provision of article 80 of the Civil Code
apply since this affidavit was never presented in proof,
of Spain, "jurisdiction in actions for divorce and
was never received by the trial judge, and cannot
nullification of canonical marriages lies with ecclesiastical
seriously be considered as an effort to establish the law
courts," while that of civil tribunals is limited to civil
of a foreign jurisdiction. Sections 300, 301 and 302 of the
marriages; that this being so, the action for divorce
Code of Civil Procedure, now in force in these islands,
brought by the plaintiff in the cause does not fall within
indicate the method by which the law of a foreign country
the jurisdiction of the civil courts, according to his own
may be proved. The Court maintains that the affidavit of a
law of persons, because these courts ought to apply the
person not versed in the law, which was never submitted
Spanish law in accordance with the said article 9 of the
as proof, never received by the trial court, and which has
Civil Cod of Spain, and this Spanish law grants the
never been subjected to any cross-examination, is not a
jurisdiction over the present cause to the ecclesiastical
means of proving a foreign law on which the defendant
courts, in the place of which no tribunal of these Islands
relies. Thus, since no proof has been submitted to this
con subrogate itself.
effect, all the property of the marriage, says article 1407 of
the Civil Code, shall be considered as conjugal property
However, husband was unable to prove by any law or legal
until it is proven that it belongs exclusively to the husband
doctrine whatever that the personal statute of a foreigner
or to the wife.
carries with it, to whether he transfers his domicile, the
authority established by the law of his nation to decree his
ON PLAINTIFFS APPEAL
divorce, which was what he had to demonstrate.
The court did not commit it in applying the rule contained
As the Court upheld:
in article 1287 of the Civil Code. "The usages or customs of
the country shall be taken into consideration in
"The jurisdiction of courts and other questions relating to
interpreting ambiguity in contracts. . . ." If in the contract
procedure are considered to be of a public nature and
the word " pesetas," not being specific, was ambiguous,
consequently are generally submitted to the territorial
then it was in harmony with this precept to interpret it as
principle. . . . All persons that have to demand justice in a
being the peseta then in use or current when and where
case in which foreigners intervene, since they can gain
the agreement was made, Mexican being then the usual
nothing by a simple declaration, should endeavor to apply
and current money in the Philippines.
to the tribunales of the state which have coercive means
(property situated in the territory) to enforce any decision
QUITA V. PADLAN
they may render. Otherwise, one would expose himself in
GR NO. 124371, DECEMBER 22, 1998
the suit to making useless expenditures which, although he
won his case, would not contribute to secure his rights
because of the court's lack of means to enforce them."
(Torres Campos, "Elementos de Derecho International FE D. QUITA and Arturo T. Padlan, both Filipinos, were
Privado," p. 108.) married in the Philippines on 18 May 1941. They were not
however blessed with children. Somewhere along the way
The provisions of article 80 of the Civil Law of Spain is their relationship soured. Eventually Fe sued Arturo for
only binding within the dominions of Spain. It does not divorce in San Francisco, California, U.S.A. and obtained a
accompany the persons of the Spanish subject wherever final judgment of divorce. She married thrice thereafter.
he may go. He could not successfully invoke it if he resided
in Japan, in China, in Hongkong or in any other territory On 1972 Arturo died. He left no will. Respondent Blandina
not subject to the dominion of Spain. Foreign Catholics Padlan claiming to be the surviving spouse of Arturo
domiciled in Spain, subject to the ecclesiastical courts in Padlan, and Claro, Alexis, Ricardo, Emmanuel, Zenaida and
actions for divorce according to the said article 80 of the Yolanda, all surnamed Padlan, named in the children of
Civil Code, could not allege lack of jurisdiction by Arturo Padlan opposed the petition. Ruperto T. Padlan,
invoking, as the law of their personal statute, a law of claiming to be the sole surviving brother of the deceased
their nation which gives jurisdiction in such a case to Arturo, also intervened.
territorial courts, or to a certain court within or without
the territory of their nation. The court held that no dispute exists as to the right of the
six (6) Padlan children to inherit from the decedent
because there are proofs that they have been duly
(Benedicto vs. De la Rama, 3 Phil. Rep., 34, and Ibaez vs. acknowledged by him and petitioner herself even
Ortiz, 5 Phil. Rep., 325). recognizes them as heirs of Arturo Padlan;

In the present action for divorce the Court of First Instance Issue:
of the city of Manila did not lack jurisdiction over the
persons of the litigants, for, although Spanish Catholic Whether or not petitioner could inherit as the surviving
spouse of Arturo?

Private International Law Page 24


Held: Art. 16. Real property as well as personal property is
subject to the law of the country where it is situated.
Case remanded subject to determination of petitioners
citizenship. However, Private respondent's claim to First, there is no such thing as one American law. The
heirship was already resolved by the trial court. She and "national law" indicated in Article 16 of the Civil Code
Arturo were married on 22 April 1947 while the prior cannot possibly apply to general American law. There is no
marriage of petitioner and Arturo was subsisting thereby such law governing the validity of testamentary provisions
resulting in a bigamous marriage considered void from in the United States. Each State of the union has its own
the beginning under Arts. 80 and 83 of the Civil Code. law applicable to its citizens and in force only within the
Consequently, she is not a surviving spouse that can inherit State. It can therefore refer to no other than the law of
from him as this status presupposes a legitimate the State of which the decedent was a resident. Second,
relationship. there is no showing that the application of the renvoi
doctrine is called for or required by New York State law.
Case remanded to the court a quo for further proceedings
since the trial court was not able to completely ascertain However, intestate and testamentary succession, both
petitioners citizenship. The trial court must have with respect to the order of succession and to the amount
overlooked the materiality of this aspect. Once proved of successional rights and to the intrinsic validity of
that she was no longer a Filipino citizen at the time of testamentary provisions, shall be regulated by the
their divorce, Van Dorn would become applicable and national law of the person whose succession is under
petitioner could very well lose her right to inherit from consideration, whatever may be the nature of the
Arturo. property and regardless of the country wherein said
property may be found. (emphasis ours)
Nota bene:

Van dorn: ang bana ga laot sa wife since under Philippine Likewise, Lorenzo Llorente was already an American
law, theyre still married. citizen when he divorced Paula. Such was also the
situation when he married Alicia and executed his will. As
stated in Article 15 of the civil code, aliens may obtain
LLORENTE V. COURT OF APPEALS
divorces abroad, provided that they are valid in their
GR No. 124371, November 23, 2000
National Law. Thus the divorce obtained by Llorente is
valid because the law that governs him is not Philippine
FACTS: Law but his National Law since the divorce was contracted
after he became an American citizen. Furthermore, his
National Law allowed divorce.
Lorenzo Llorente and petitioner Paula Llorente were
married in 1937 in the Philippines. Lorenzo was an
enlisted serviceman of the US Navy. Soon after, he left for The case was remanded to the court of origin for
the US where through naturalization, he became a US determination of the intrinsic validity of Lorenzo Llorentes
Citizen. Upon his visit to his wife, he discovered that she will and determination of the parties successional rights
was living with his brother and a child was born. The child allowing proof of foreign law.
was registered as illegitimate but the name of the father
was left blank. Llorente filed a divorce in California in Nota bene:
which Paula was represented by counsel, John Riley, and
actively participated in the proceedings, which later on The following are the requisites of res judicata:
became final. He married Alicia and they lived together for 1. the former judgment must be final;
25 years bringing 3 children. He made his last will and 2. the court that rendered it had jurisdiction over
testament stating that all his properties will be given to his the subject matter and the parties;
second marriage. He filed a petition of probate that made 3. it is a judgment on the merits; and
or appointed Alicia his special administrator of his estate. 4. there is between the first and the second
Before the proceeding could be terminated, Lorenzo died. actions an identity of parties, subject matter
Paula filed a letter of administration over Llorentes estate. and cause of action.
The trial granted the letter and denied the motion for
reconsideration. An appeal was made to the Court of It has been held that in order that a judgment in one
Appeals, which affirmed and modified the judgment of the action can be conclusive as to a particular matter in
Trial Court that she be declared co-owner of whatever another action between the same parties or their privies,
properties, she and the deceased, may have acquired in it is essential that:
their 25 years of cohabitation.
1. the issue be identical;
ISSUE: 2. If a particular point or question is in issue in the
second action,
3. and the judgment will depend on the
Whether or not national law shall apply? determination of that particular point or
question,
4. a former judgment between the same parties or
RULING: their privies will be final and conclusive in the
second if that same point or question was in
Art. 15. Laws relating to family rights and duties, or to the issue and adjudicated in the first suit (Nabus v.
status, condition and legal capacity of persons are binding Court of Appeals, 193 SCRA 732 [1991]).
upon citizens of the Philippines, even though living 5. Identity of cause of action is not required but
abroad. merely identity of issue.

Private International Law Page 25


A 16-year old Cambodian inherited real properties from
Art. 15. Laws relating to family rights and the Philippines; he died and left a will which disposes of his
duties, or to the status, condition and properties in the Philippines; supposing, Cambodian law
legal capacity of persons are binding provides that age of majority is 16, is the will now if
upon citizens of the Philippines, even probated in the Philippines valid?
though living abroad.
(FRDSCL) If an 18-year old foreigner, where in his country, the age of
majority is 21 goes to the Philippines and gets married, is
Personal status legal position of an individual in a such marriage valid?
society
Capacity power to acquire and Family rights, duties, status, condition and legal capacity
exercise rights of persons accompany a person even when he moves to a
Incidental to personal status foreign country;

Foreign laws regulating the persons status and capacity


are to be disregarded where they are political or penal in MARRIAGE
character;
Art. II Section 12, 1987 Constitution
Legislative jurisdiction authority of the state of his
nationality or domicile or where he may be physically The State recognizes the sanctity of family life and shall
present to promulgate laws affecting his status; protect and strengthen the family as a basic autonomous
Judicial jurisdiction the authority of the court to hear and
social institution. It shall equally protect the life of the
determine the cause of action
mother and the life of the unborn from conception. The
Beginning of personality conception provided natural and primary right and duty of parents in the
subsequent birth rearing of the youth for civic efficiency and the
End death development of moral character shall receive the support
of the Government.
Other questions of status
Art. XV, Sec. 2, 1987 Constitution
1. Absence a special legal status, if one is not in
his domicile and his whereabouts is unknown; Marriage, as an inviolable social institution, is the
When can be declared: under civil code
foundation of the family and shall be protected by the
a. Lapse of 2 years without news or since the
State.
receipt of last news
b. Lapse of 5 years if absentee has left
someone in charge of the administration of NCC Art. 15. Laws relating to family rights and
his properties duties, or to the status, condition and
legal capacity of persons are binding
Who may declare: upon citizens of the Philippines, even
a. Spouse though living abroad. (9a)
b. Heirs
c. Relatives who will benefit in intestacy Art. 17. The forms and solemnities of contracts,
d. Those who have rights over the properties
wills, and other public instruments
2. Conflicts problems respecting an individuals shall be governed by the laws of the
name and extent of protection against abuse of country in which they are executed.
his name
When the acts referred to are executed
Right to use a title of nobility is determined in before the diplomatic or consular
accordance with his national law officials of the Republic of the
Philippines in a foreign country, the
3. Minority solemnities established by Philippine
laws shall be observed in their
CAPACITY execution.

1. Juridical capacity the fitness to be the subject


Prohibitive laws concerning persons,
of legal relations; considered inherent in every
their acts or property, and those which
natural person and is lost only through death;
have, for their object, public order,
2. Capacity to act the power to do acts with legal
public policy and good customs shall
effects; it is acquired and may be lost;
not be rendered ineffective by laws or
judgments promulgated, or by
Nota bene:
determinations or conventions agreed
upon in a foreign country. (11a)
Under Philippine law, the capacity to dispose of real
property is governed by the lex situs while the capacity to
succeed is governed by the national law of the deceased. Art. 66. When either or both of the contracting
parties are citizens or subjects of a
Question: foreign country, it shall be necessary,
before a marriage license can be
obtained, to provide themselves with a

Private International Law Page 26


certificate of legal capacity to contract or in open court, in the church, chapel
marriage, to be issued by their or temple, or in the office the consul-
respective diplomatic or consular general, consul or vice-consul, as the
officials. (13a) case may be, and not elsewhere,
except in cases of marriages contracted
Art. 71. All marriages performed outside the on the point of death or in remote
Philippines in accordance with the laws places in accordance with Article 29 of
in force in the country where they this Code, or where both of the parties
were performed, and valid there as request the solemnizing officer in
such, shall also be valid in this country, writing in which case the marriage may
except bigamous, polygamous, or be solemnized at a house or place
incestuous marriages as determined by designated by them in a sworn
Philippine law. (19a) statement to that effect. (57a)

FC Art. 10. Marriages between Filipino citizens


Art. 75. Marriages between Filipino citizens
abroad may be solemnized by a consul-
abroad may be solemnized by consuls
general, consul or vice-consul of the
and vice-consuls of the Republic of the
Republic of the Philippines. The
Philippines. The duties of the local civil
issuance of the marriage license and
registrar and of a judge or justice of the
the duties of the local civil registrar and
peace or mayor with regard to the
of the solemnizing officer with regard
celebration of marriage shall be
to the celebration of marriage shall be
performed by such consuls and vice-
performed by said consular official.
consuls. (n)
(75a)

FC Article 1. Marriage is a special contract of


permanent union between a man and
Art. 26. All marriages solemnized outside the
a woman entered into in accordance
Philippines, in accordance with the
with law for the establishment of
laws in force in the country where they
conjugal and family life. It is the
were solemnized, and valid there as
foundation of the family and an
such, shall also be valid in this country,
inviolable social institution whose
except those prohibited under Articles
nature, consequences, and incidents
35 (1), (4), (5) and (6), 3637 and 38.
are governed by law and not subject to
(17a)
stipulation, except that marriage
settlements may fix the property
relations during the marriage within Where a marriage between a Filipino
the limits provided by this Code. (52a) citizen and a foreigner is validly
celebrated and a divorce is thereafter
validly obtained abroad by the alien
Art. 7. Marriage may be solemnized by:
spouse capacitating him or her to
remarry, the Filipino spouse shall have
1. Any incumbent member of the capacity to remarry under Philippine
judiciary within the court's law. (As amended by Executive Order
jurisdiction; 227)
2. Any priest, rabbi, imam, or
minister of any church or religious
FC Art. 35. The following marriages shall be void
sect duly authorized by his church
from the beginning:
or religious sect and registered
1. Those contracted by any party
with the civil registrar general,
below eighteen years of age even
acting within the limits of the
with the consent of parents or
written authority granted by his
guardians;
church or religious sect and
2. Those solemnized by any person
provided that at least one of the
not legally authorized to perform
contracting parties belongs to the
marriages unless such marriages
solemnizing officer's church or
were contracted with either or
religious sect;
both parties believing in good
3. Any ship captain or airplane chief
faith that the solemnizing officer
only in the case mentioned in
had the legal authority to do so;
Article 31;
3. Those solemnized without license,
4. Any military commander of a unit
except those covered the
to which a chaplain is assigned, in
preceding Chapter;
the absence of the latter, during a
4. Those bigamous or polygamous
military operation, likewise only in
marriages not failing under Article
the cases mentioned in Article 32;
41;
5. Any consul-general, consul or
5. Those contracted through mistake
vice-consul in the case provided in
of one contracting party as to the
Article 10. (56a)
identity of the other; and
6. Those subsequent marriages that
Article. 8. The marriage shall be solemnized are void under Article 53.
publicly in the chambers of the judge

Private International Law Page 27


Art. 36. A marriage contracted by any party not in accordance with
who, at the time of the celebration, Muslim law or this
was psychologically incapacitated to Code, the Civil Code of
comply with the essential marital the Philippines shall
obligations of marriage, shall likewise apply. .chan robles
be void even if such incapacity virtual law library
becomes manifest only after its (3) Subject to the
solemnization. (As amended by provisions of the
Executive Order 227) preceding paragraphs,
the essential requisites
Art. 37. Marriages between the following are and legal impediments
incestuous and void from the to marriage, divorce,
beginning, whether relationship paternity and filiation,
between the parties be legitimate or guardianship and
illegitimate: custody of minors,
support and
maintenance, claims for
1. Between ascendants and
customary dower
descendants of any degree; and
(mahr), betrothal,
2. Between brothers and sisters,
breach of contract to
whether of the full or half blood.
marry, solemnization
(81a)
and registration of
marriage and divorce,
Art. 38. The following marriages shall be void rights and obligations
from the beginning for reasons of public policy: between husband and
wife parental authority,
1. Between collateral blood and the properly
relatives whether legitimate relations between
or illegitimate, up to the husband and wife shall
fourth civil degree; be governed by this
2. Between step-parents and Code and other
step-children; applicable Muslim laws.
3. Between parents-in-law and
children-in-law;
Art. 14. Nature. Marriage is not only a civil
4. Between the adopting parent
contract but a social institution. Its
and the adopted child;
nature, consequences and incidents
5. Between the surviving
are governed by this Code and the
spouse of the adopting
Shari'a and not subject to stipulation,
parent and the adopted
except that the marriage settlements
child;
may to a certain extent fix the property
6. Between the surviving
relations of the spouses.
spouse of the adopted child
and the adopter;
7. Between an adopted child Art. 180. Law applicable. The provisions of
and a legitimate child of the the Revised Penal Code relative to the
adopter; crime of bigamy shall not apply to a
8. Between adopted children of person married in accordance with the
the same adopter; and provisions of this Code or, before its
9. Between parties where one, effectivity, under Muslim law.
with the intention to marry
the other, killed that other
person's spouse, or his or her Essential and Formal Requisites
own spouse. (82)
FC Art. 2. No marriage shall be valid, unless these essential
Muslim Code PD 1083, Art. 13. Application. requisites are present:

(1) The provisions of this 1. Legal capacity of the


Title shall apply to contracting parties who must
marriage and divorce be a male and a female; and
wherein both parties 2. Consent freely given in the
are Muslims, or wherein presence of the solemnizing
only the male party is a officer. (53a)
Muslim and the
marriage is solemnized
Art. 3. The formal requisites of marriage are:
in accordance with
Muslim law or this Code
1. Authority of the solemnizing
in any part of the
officer;
Philippines.
2. A valid marriage license
(2) In case of marriage
except in the cases provided
between a Muslim and a
non-Muslim, solemnized

Private International Law Page 28


for in Chapter 2 of this Title; a. Legal capacity of the contracting
and parties;
3. A marriage ceremony which b. Mutual consent of the parties freely
takes place with the given;
appearance of the c. Offer (ijab) and acceptance (qabul)
contracting parties before duly witnessed by at least two
the solemnizing officer and competent persons after the proper
their personal declaration guardian in marriage (wali) has given
that they take each other as his consent; and
husband and wife in the d. Stipulation of customary dower (mahr)
presence of not less than two duly witnessed by two competent
witnesses of legal age. (53a, persons.
55a)

Art. 5. Any male or female of the age of GOITIA VS. CAMPOS RUEDA
eighteen years or upwards not under
any of the impediments mentioned in Facts:
Articles 37 and 38, may contract
marriage. (54a) The parties were legally married in the city of Manila on
January 7, 1915, and immediately thereafter established
Art. 35. The following marriages shall be void their residence at 115 Calle San Marcelino, where they
from the beginning: lived together for about a month, when the plaintiff
returned to the home of her parents.
1. Those contracted by any party
below eighteen years of age even That the defendant, one month after he had contracted
with the consent of parents or marriage with the plaintiff, demanded of her that she
guardians; perform unchaste and lascivious acts on his genital organs;
2. Those solemnized by any person that the plaintiff spurned the obscene demands of the
not legally authorized to perform defendant and refused to perform any act other than legal
marriages unless such marriages and valid cohabitation; that the defendant, since that date
were contracted with either or had continually on other successive dates, made similar
both parties believing in good lewd and indecorous demands on his wife, the plaintiff,
faith that the solemnizing officer who always spurned them, which just refusals of the
had the legal authority to do so; plaintiff exasperated the defendant and induce him to
3. Those solemnized without license, maltreat her by word and deed and inflict injuries upon
except those covered the her lips, her face and different parts of her body; and that,
preceding Chapter; as the plaintiff was unable by any means to induce the
4. Those bigamous or polygamous defendant to desist from his repugnant desires and cease
marriages not failing under Article from maltreating her, she was obliged to leave the conjugal
41; abode and take refuge in the home of her parents.
5. Those contracted through mistake
of one contracting party as to the Issue: whether or not that the wife may claim for support
identity of the other; and against her husband outside of their conjugal abode?
6. Those subsequent marriages that
are void under Article 53. Ruling: marriage partakes of the nature of an ordinary
contract. But it is something more than a mere contract. It
is a new relation, the rights, duties, and obligations of
Art. 4. The absence of any of the essential or
which rest not upon the agreement of the parties but
formal requisites shall render the
upon the general law which defines and prescribes those
marriage void ab initio, except as
rights, duties, and obligations. Marriage is an institution,
stated in Article 35 (2).
in the maintenance of which in its purity the public is
deeply interested. It is a relation for life and the parties
A defect in any of the essential cannot terminate it at any shorter period by virtue of any
requisites shall not affect the validity of contract they may make .The reciprocal rights arising from
the marriage but the party or parties this relation, so long as it continues, are such as the law
responsible for the irregularity shall be determines from time to time, and none other. When the
civilly, criminally and administratively legal existence of the parties is merged into one by
liable. (n) marriage, the new relation is regulated and controlled by
the state or government upon principles of public policy
for the benefit of society as well as the parties. And when
the object of a marriage is defeated by rendering its
Art. 35 (2) supra continuance intolerable to one of the parties and
productive of no possible good to the community, relief in
some way should be obtainable. With these principles to
Muslim Code, Art. 15. Essential requisites.
guide us, we will inquire into the status of the law touching
and governing the question under consideration.
No marriage contract shall be perfected unless the
following essential requisites are compiled with: The mere act of marriage creates an obligation on the
part of the husband to support his wife. This obligation is
founded not so much on the express or implied terms of

Private International Law Page 29


the contract of marriage as on the natural and legal duty latter found that petitioner's claim that she is the lawful
of the husband; an obligation, the enforcement of which is wife of Perfecto Blas was without basis in evidence as it
of such vital concern to the state itself that the laws will was "bereft of substantial proof of husband-wife
not permit him to terminate it by his own wrongful acts in relationship"; that said Board further held that, it
driving his wife to seek protection in the parental home. A appearing that in the entry proceedings of Perfecto Blas
judgment for separate maintenance is not due and payable had on January 23, 1947 he declared that he first visited
either as damages or as a penalty; nor is it a debt in the China in 1935 and married petitioner in 1936, it could not
strict legal sense of the term, but rather a judgment calling possibly sustain her claim that she married Perfecto Blas in
for the performance of a duty made specific by the 1929; that in an affidavit dated August 9, 1962 Perfecto
mandate of the sovereign. This is done from necessity and Blas claimed that he went to China in 1929, 1935 and
with a view to preserve the public peace and the purity of 1941, although in his re-entry declaration he admitted that
the wife; as where the husband makes so base demands he first went to China in 1935, then in 1937, then in 1939,
upon his wife and indulges in the habit of assaulting her. and lastly in 1941; and that Perfecto Blas in the same
The pro tanto separation resulting from a decree for affidavit likewise claimed that he first went to China when
separate support is not an impeachment of that public he was merely four years old so that computed from his
policy by which marriage is regarded as so sacred and date of birth in 1908 it must have been in 1912.
inviolable in its nature; it is merely a stronger policy
overruling a weaker one; and except in so far only as such In view of the discrepancies found in the statements made
separation is tolerated as a means of preserving the public by petitioner and her alleged husband Perfecto Blas in the
peace and morals may be considered, it does not in any several investigations conducted by the immigration
respect whatever impair the marriage contract or for any authorities concerning their alleged marriage before a
purpose place the wife in the situation of a feme sole. village leader in China in 1929, coupled with the fact that
the only basis in support of petitioner's claim that she is
Where the wife, who is forced to leave the conjugal abode the lawful wife of Perfecto Blas is "a mass of oral and
by her husband without fault on her part, may maintain an documentary evidence bereft of substantial proof of
action against the husband for separate maintenance husband-wife relationship," the Board of Commissioners
when she has no other remedy, notwithstanding the motu proprio reviewed the record concerning the
provisions of article 149 of the Civil Code giving the admission of petitioner into the country resulting in its
person who is obliged to furnish support the option to finding that she was improperly admitted.
satisfy it either by paying a fixed pension or by receiving
and maintaining in his own home the one having the right
Issue: whether or not that the marriage is valid?
to the same.

Ruling: there is no documentary evidence to support the


alleged marriage of petitioner to Perfecto Blas but the
WONG WOO YIU VS. VIVO record is punctured with so many inconsistencies which
cannot but lead one to doubt their veracity concerning the
Facts: pretended marriage in China in 1929. This claim cannot
also be entertained under our law on family relations.
The Board of Special Inquiry No. 3 rendered a decision Thus, Article 15 of our new Civil Code provides that laws
finding petitioner to be legally married to Perfecto Blas relating to family rights or to the status of persons are
and admitting her into the country as a non-quota binding upon citizens of the Philippines, even though
immigrant. This decision was affirmed by the Board of living abroad, and it is well-known that in 1929 in order
Commissioners of which petitioner was duly informed in a that a marriage celebrated in the Philippines may be valid
letter sent on the same date by the Secretary of the Board. it must be solemnized either by a judge of any court
However, the same Board of Commissioners, but inferior to the Supreme Court, a justice of the peace, or a
composed entirely of a new set of members, rendered a priest or minister of the gospel of any denomination duly
new decision reversing that of the Board of Special registered in the Philippine Library and Museum (Public
Inquiry No. 3 and ordering petitioner to be excluded from Act 3412, Section 2). Even if we assume, therefore, that
the country. Petitioner filed a motion for new trial the marriage of petitioner to Perfecto Blas before a village
requesting an opportunity to clarify certain points taken in leader is valid in China, the same is not one of those
the decision, but the same was denied for lack of merit. authorized in our country.
Whereupon, petitioner initiated the instant petition for
mandamus with preliminary injunction before the Court of A marriage contracted outside of the Philippines which is
First Instance of Manila which incidentally was considered valid under the law of the country in which it was
by it as a petition for certiorari. celebrated is also valid in the Philippines. But no validity
can be given to this contention because no proof was
Petitioner declared that she came to the Philippines in presented relative to the law of marriage in China. Such
1961 for the first time to join her husband Perfecto Blas being the case, we should apply the general rule that in
to whom she was married in Chingkang, China ;that they the absence of proof of the law of a foreign country it
had several children all of whom are not in the Philippines; should be presumed that it is the same as our own.
that their marriage was celebrated by one Chua Tio, a
village leader; that on June 28, 1961 the Board of Special Since our law only recognizes a marriage celebrated before
Inquiry No. 3 rendered a decision finding, among others, any of the officers mentioned therein, and a village leader
that petitioner is legally married to Perfecto Blas, a Filipino is not one of them, it is clear that petitioner's marriage,
Citizen, and admitted her into the country as a non-quota even if true, cannot be recognized in this jurisdiction.
immigrant; that this decision was affirmed by the Board of
Commissioners of which petitioner was duly notified by ADONG VS. CHAONG SENG GEE
the Secretary of said Board in a letter dated July 12, 1961;
that in a motu proprio decision rendered by the Board of
Commissioners composed of a new set of members the Facts:

Private International Law Page 30


Cheong Boo, a native of China died in Zamboanga, by convincing evidence. A Philippine marriage followed by
Philippine Islands on August 5, 1919 and left property 23 years of uninterrupted marital life, should not be
worth nearly P100,000 which is now being claimed by impugned and discredited, after the death of the husband
two parties - (1) Cheong Seng Gee who alleged that he through an alleged prior Chinese marriage, save upon
was a legitimate child by marriage contracted by Cheong proof so clear, strong and unequivocal as to produce a
Boo with Tan Bit in China in 1985, and (2) Mora Adong moral conviction of the existence of such impediment. A
who alleged that she had been lawfully married to marriage alleged to have been contracted in China and
Cheong Boo in 1896 in Basilan, Philippine Islands and had proven mainly by a so-called matrimonial letter held not to
two daughters with the deceased namely Payang and be valid in the Philippines.
Rosalia. The conflicting claims to Cheong Boos estate were
ventilated in the lower court that ruled that Cheong Seng
Gee failed to sufficiently establish the Chinese marriage
through a mere letter testifying that Cheong Boo and Tan
CHING HUAT VS. CO HEONG
Bit married each other but that because Cheong Seng Gee
had been admitted to the Philippine Islands as the son of
the deceased, he should share in the estate as a natural Facts:
child. With reference to the allegations of Mora Adong and
her daughters, the trial court reached the conclusion that It is alleged in the petition, that the said minor is his
the marriage between Adong and Cheong Boo had been legitimate daughter; that up to June 21, 1946, said minor
adequately proved but that under the laws of the had been living with and had under the custody of
Philippine Islands it could not be held to be a lawful petitioner; that respondent, taking advantage of his
marriage and thus the daughter Payang and Rosalia would confidential and spiritual relation with Maria Ching as her
inherit as natural children. The lower court believes that godfather, persuaded and induced her by means of trick,
Mohammedan marriages are not valid under the promises and cajolery, to leave the parental home and to
Philippine Islands laws this as an Imam as a solemnizing elope with him in the night of June 21, 1946, to Plaridel,
officer and under Quaranic laws. Bulacan, where they were married on the following day
before the Justice of the Peace of said municipality, said
ISSUES: Whether or not the Chinese marriage between Maria Ching being at the time 15 years old; and that ever
Cheong Boo and Tan Dit is valid ? since respondent has had the minor Maria Ching under his
custody in Malolos, Bulacan, and has restrained her at her
liberty.
Whether or not the Mohammedan marriage between
Cheong Boo and Mora Adong is valid?
It is further alleged that respondent had been previously
married in China to Gue Min, said marriage being said to
RULING: The Supreme Court found the (1) Chinese
be subsisting at the time respondent married Maria Ching.
marriage not proved and Chinaman Cheong Seng Gee has
Petitioner further avers that Gue Min has never been
only the rights of a natural child while (2) it found the
declared an absentee nor generally considered as dead
Mohammedan marriage to be proved and to be valid,
and believed to be so by respondent at the time he
thus giving to the widow Mora Adong and the legitimate
married Maria Ching.
children Payang and Rosalia the rights accruing to them
under the law.
Respondent, in his answer, among other things, asserts
that on June 21, 1946, he and Maria Ching alias Avelina
(FOR STATCON) The Supreme Court held that marriage in
Ching were legally married before the Justice of the Peace
this jurisdiction is not only a civil contract but it is a new
of Plaridel, Bulacan, and alleges that the essential
relation, an instruction in the maintenance of which the
requisites for such marriage were complied with.
public is deeply interested. The presumption as to
marriage is that every intendment of the law leans toward
legalizing matrimony. Persons dwelling together in Issue: whether or not petitioner still retains his right to the
apparent matrimony are presumed, in the absence of custody of his minor daughter Maria Ching alias Avelina
counter-presumption or evidence special to the case, to be Ching?
in fact married. The reason is that such is the common
order of society, and if the parties were not what they thus Ruling: the Philippine marriage between said respondent
hold themselves out as being, they would be living in the and Maria Ching before the Justice of the Peace of
constant violation of decency of the law. As to retroactive Plaridel, Bulacan, is undisputed. It is also beyond question
force, marriage laws is in the nature of a curative provision that marriage was contracted by a man much over 16 years
intended to safeguard society by legalizing prior marriages. old with a girl 15 years old (Act No. 3613, section 2),
Public policy should aid acts intended to validate neither of whom was included in any of the exceptions
marriages and should retard acts intended to invalidate mentioned in section 28 of the same Act; nor in those
marriages. This as for public policy, the courts can properly stated in section 29 thereof for the reason that the alleged
incline the scales of their decision in favor of that solution prior Chinese marriage has not been established.
which will most effectively promote the public policy. That
is the true construction which will best carry legislative If the supposed prior Chinese marriage had been
intention into effect. sufficiently proven, then in order that the subsequent
(FOR PERSONS) Sec. IV of the Marriage law provides that Philippine marriage could be valid, it would have been
all marriages contracted outside the islands, which would necessary either (a) that the Chinese marriage should
be valid by the laws of the country in which the same were have been previously annulled or dissolved: or (b) that
contracted, are valid in these islands. To establish a valid the first wife of respondent should have been absent for
foreign marriage pursuant to this comity provision, it is 7 consecutive years at the time of the second marriage
first necessary to prove before the courts ofthe Islands without the respondent having news of the absentee
the existence of the foreign law as a question of fact, and being alive; or (c) that the absentee should have been
it is then necessary to prove the alleged foreign marriage

Private International Law Page 31


generally considered as dead and believed to be so by Facts:
respondent at the time of contracting the subsequent
marriage, in either of which last two cases the Yap Siong died intestate. During the distribution of his
subsequent marriage will be valid until declared null and estate, Maria Lao and Jose Lao appeared claiming to be the
void by a competent court, while in the first it will be legitimate spouse and son of the deceased. Maria claims
valid without this limitation. (Act No. 3613, section 29 [a] that they had been married in the Philippines on June 24,
and [b]; section 30 [b].) But as already adverted to, the 1903. On the other hand, Dee Tim claims to be the
complete absence of proof of the supposed former legitimate widow of Yap Siong; that she and Yap Siong
Chinese marriage makes sections 29 and 30 of the were joined in holy matrimony on the 14th day of
Marriage Law inapplicable. September, 1893, in accordance with the laws of China.

Maria Ching having been validly married on June 21,


1946, she became emancipated on that same date (arts. Maria Lao presented marriage certificates as proof. Dee
314 [1] and 315, Civil Code). This emancipation brought Tim likewise presented a certificate of marriage and that it
about the loss by the father of the parental authority that was positive proof of her marriage and that it complied
he claims. On the other hand, by article 48 of Chapter V of with the custom and practice in China with reference to
the Spanish Marriage Law of 1870, whose articles 44 to 78 marriage ceremonies. To support her contention she
were, and are now partly, in force in the Philippines presented a number of witnesses. Jan Peng, a Chinaman of
(Benedicto vs. De la Rama, 3 Phil., 34), the wife has the 52 years of age, swore that he knew the forms of
duty, among others, of living in her husband's company ceremonies of marriage in China, Dee Tim also presented a
and of following him to wherever he transfer his domicile witness, Ty Cong Ting, a Chinaman, 32 years of age and a
or residence. (Yaez de Barbuevo vs. Fuster, 29 Phil., 606, lawyer, who testified concerning the laws and customs in
612.) China with reference to the forms of marriage ceremony.
He testified that he knew and was well acquainted with
the customs and practices of Chinamen in China with
reference to marriages and the manner and form in which
they were celebrated, and the form of proof issued for the
PEOPLE V. DUMPO
purpose of proving that a marriage ceremony had been
62 Phil 247
performed. Mr. Ty Cong Ting was, at the time he testified
as a witness, the legal attorney of the Chinese Consul
Facts:
General in the City of Manila.
Moro Hassan and Mora Dupo have been legally married
according to the rites and practice of the Mohammedan Issue:
religion. Without this marriage being dissolved, it is
alleged that Dumpo contracted another marriage with Who is the legal wife?
Moro Sabdapal after which the two lived together as
husband and wife. Dumpo was prosecuted for and Held:
convicted of the crime of bigamy.
It is perhaps true that Yap Siong did on various occasions,
The accused interposed an appeal. It has been established
depending upon his interest and convenience at the
by the defense, without the prosecution having presented
particular time, state that Maria Lao was his querida and
any objection nor evidence to the contrary, that the
not his wife. It is also perhaps true, for the same reason,
alleged second marriage of the accused is null and void
that he stated that Dee Tim was not his wife but his
according to Mohammedan rites on the ground that her
querida. Evidently he was attempting to keep the
father had not given his consent thereto.
information, which he was quite able to do, until he had
passed to that bourn from which none returns, and until
Held:
a distribution of his large accumulated earnings among
his heirs became necessary.
Marriage among Moslems is a fact of which no judicial
notice may be taken and must be subject to proof in
every particular case. In the case at bar we have the Based on a preponderance of the evidence the Court was
uncontradicted testimony of Tahari, an Imam or convinced that both Dee Tim and Maria Lao were legally
Mohammedan priest authorized to solemnize marriages married to Yap Siong in good faith, believing that each
between Mohammedans, to the effect that the consent of was his sole and separate wife, living in absolute ignorance
the bride's father or in the absence thereof, that of the of the fact of his double marriage. They were each
chief of the tribe to which she belongs is an indispensable married in good faith and in ignorance of the existence of
requisite for the validity of such contracts. the other marriage. Yap Siong up to the time of his death
seems to have been successful in keeping each of his two
It is an essential element of the crime of bigamy that the wives ignorant of the fact that he was married to the other.
alleged second marriage, having all the essential
requisites, would be valid were it not for the subsistence Under the Leyes de Partidas (Law 1, title 13, partida 4),
of the first marriage. However, accuseds subsequent where two women innocently and in good faith are
marriage was void for lack of requisites necessary under legally united in holy matrimony to the same man, their
Moslem law, she must be acquitted. children born will be regarded as legitimate children and
each family will be entitled to one-half of the estate of
the husband upon distribution of his estate. That
provision of the Leyes de Partidas is a very humane and
LAO AND LAO V. DEE TIM wise law. It justly protects those who innocently have
45 Phil 739 (1924) entered into the solemn relation of marriage and their
descendants. The good faith of all the parties will be

Private International Law Page 32


presumed until the contrary is positively proved. (Article custom, the marriage between Yao Kee and Sy Kiat
69, Civil Code; Las Leyes de Matrimonio, section 96; Gaines cannot be recognized in this jurisdiction.
vs. Hennen, 65 U.S., 553.)c However, as petitioners failed to establish the
marriage of Yao Kee with Sy Kiat according to the laws
YAO KEE V. GONZALES of China, they cannot be accorded the status of
167 SCRA 736 legitimate children but only that of acknowledged
natural children. Petitioners are natural children, it
FACTS: appearing that at the time of their conception Yao
Kee and Sy Kiat were not disqualified by any
impediment to marry one another. [See Art. 269, Civil
Code] And they are acknowledged children of the
1. Sy Kiat, a Chinese national, died in Calooocan deceased because of Sy Kiats recognition of Sze Sook
City where he was then residing leaving behind Wah and its extension to Sze Lai Cho and Sy Chun Yen
real and personal properties here in the who are her sisters of the full blood.
Philippines. Private respondents on the other hand are also the
2. Private respondents (Aida Sy-Gonzales et al.,) deceaseds acknowledged natural children with
filed a petition for the grant of letters or Asuncion Gillego , a Filipina with whom he lived for 25
administration alleging that they were the years without the benefit of marriage. They have in
children of the deceased with Asuncion Gillego. their favor their fathers acknowledgment, evidence
3. Petition was opposed by herein petitioners (Yao by a compromise agreement entered into by and
Kee et al.,) alleging that they were the legitimate between their parents and approved by the CFI
family. wherein Sy Kiat not only acknowledged them as his
4. The probate court found that Sy Kiat was legally children by Asuncion Gillego but likewise made
married to Yao Kee and that their 3 offsprings provisions for their support and future inheritance.
were the legitimate children.
5. The court likewise ruled that respondents are the REPUBLIC V. ORBECIDO
acknowledged illegitimate offspring of Sy Kiat GR NO. 154380, October 5, 2005
with Asuncion Gillego.
6. On appeal, the lower courts decision was set
aside declaring petitioners as the acknowledge
natural children of Sy Kiat and Asuncion Gillego. Facts:
7. Oppostiors were declared the acknowelged
natural children of the deceased since the On May 24, 1981, Cipriano Orbecido III married Lady
legality of the alleged marriage of Sy Kiat and Yao Myros M. Villanueva in the Philippines in Lam-an, Ozamis
Kee in China had not been proven to be valid to City. Their marriage was blessed with a son and a
the laws of China. daughter.

ISSUE: In 1986, Ciprianos wife left for the United States bringing
along their son Kristoffer. A few years later, Cipriano
Was the fact of marriage of Sy Kiat and Yao Kee in discovered that his wife had been naturalized as an
China proven as a custom? American citizen.

HELD: Sometime in 2000, Cipriano learned from his son that his
wife had obtained a divorce decree and then married a
Custom is defined as a rule of conduct formed by certain Innocent Stanley. She, Stanley and her child by him
repetition of acts, uniformly observed (practiced) as currently live in San Gabriel, California.
a social rule, legally binding and obligatory. The law
requires that a custom must be proved as a fact, Cipriano thereafter filed with the trial court a petition for
according to the rules of evidence. [Article 12, Civil authority to remarry invoking Paragraph 2 of Article 26 of
Code] On this score the Court had occasion to state the Family Code. No opposition was filed. Finding merit in
that a local custom as a source of right cannot be the petition, the court granted the same. The Republic,
considered by a court of justice unless such custom is herein petitioner, through the Office of the Solicitor
properly established by competent evidence like any General (OSG), sought reconsideration but it was denied.
other fact. The same evidence, if not one of a higher
degree, should be required of a foreign custom.
The OSG contends that Paragraph 2 of Article 26 of the
Construing this provision of law the Court has held
Family Code is not applicable to the instant case because
that to establish a valid foreign marriage two things
it only applies to a valid mixed marriage; that is, a
must be proven, namely 1) the existence of the
marriage celebrated between a Filipino citizen and an
foreign law as a question of fact; and 2) the alleged
alien. Furthermore, the OSG argues there is no law that
foreign marriage by convincing evidence.
governs respondents situation. The OSG posits that this is
In the case at bar petitioners did not present any
a matter of legislation and not of judicial determination.
competent evidence relative to the law and custom
of China on marriage. The testimonies of Yao and Gan
Ching (brother) cannot be considered as proof of Held:
Chinas law or custom on marriage not only because
they are self serving evidence, but more importantly, Taking into consideration the legislative intent and
there is no showing that they are competent to applying the rule of reason, we hold that Paragraph 2 of
testify on the subject matter. For failure to prove the Article 26 should be interpreted to include cases involving
foreign law or custom, and consequently, the validity parties who, at the time of the celebration of the
of the marriage in accordance with said law or marriage were Filipino citizens, but later on, one of them

Private International Law Page 33


becomes naturalized as a foreign citizen and obtains a In case of disagreement, the court shall
divorce decree. The Filipino spouse should likewise be decide whether or not:
allowed to remarry as if the other party were a foreigner
at the time of the solemnization of the marriage. To rule 1. The objection is proper; and
otherwise would be to sanction absurdity and injustice. 2. Benefit has occurred to the family
Where the interpretation of a statute according to its exact prior to the objection or
and literal import would lead to mischievous results or thereafter. If the benefit accrued
contravene the clear purpose of the legislature, it should prior to the objection, the
be construed according to its spirit and reason, resulting obligation shall be
disregarding as far as necessary the letter of the law. A enforced against the separate
statute may therefore be extended to cases not within the property of the spouse who has
literal meaning of its terms, so long as they come within its not obtained consent.
spirit or intent.
The foregoing provisions shall not
In view of the foregoing, we state the twin elements for prejudice the rights of creditors who
the application of Paragraph 2 of Article 26 as follows: acted in good faith. (117a)

1. There is a valid marriage that has been Muslim Code


celebrated between a Filipino citizen and a
foreigner; and
2. A valid divorce is obtained abroad by the alien Art. 34. Mutual rights and obligations.
spouse capacitating him or her to remarry.
1. The husband and the wife are
The reckoning point is not the citizenship of the parties at obliged to live together, observe
the time of the celebration of the marriage, but their mutual respect and fidelity, and
citizenship at the time a valid divorce is obtained abroad render mutual help and support in
by the alien spouse capacitating the latter to remarry. accordance with this Code.
2. When one of the spouses neglects
VI. CONSEQUENCES OF MARRIAGE his or her duties to the conjugal
union or brings danger, dishonor
1. PERSONAL RELATIONS or material injury upon the other,
the injured party may petition the
court for relief. The court may
Civil Code Art. 15, supra
counsel the offender to comply
with his or her duties, and take
Family Code such measures as may be proper.

Art. 69. The husband and wife shall fix the 3. The husband and the wife shall
family domicile. In case of inherit from each other in
disagreement, the court shall decide. accordance with this Code.

4. The husband and the wife shall


The court may exempt one spouse
have the right to divorce in
from living with the other if the latter
accordance with this Code.
should live abroad or there are other
valid and compelling reasons for the
exemption. However, such exemption Art. 35. Rights and obligations of the husband.
shall not apply if the same is not The husband shall fix the residence
compatible with the solidarity of the of the family. The court may exempt
family. (110a) the wife from living with her husband
on any of the following grounds:
Art. 70. The spouses are jointly responsible for
the support of the family. The expenses a. Her dower is not satisfied in
for such support and other conjugal accordance with the stipulations;
obligations shall be paid from the or
community property and, in the b. The conjugal dwelling is not in
absence thereof, from the income or keeping with her social standing
fruits of their separate properties. In or is, for any reason, not safe for
case of insufficiency or absence of said the members of the family or her
income or fruits, such obligations shall property.
be satisfied from the separate
properties. (111a) Art. 36. Rights and obligations of the wife.

Art. 73. Either spouse may exercise any 1. The wife shall dutifully manage the
legitimate profession, occupation, affairs of the household. She may
business or activity without the purchase things necessary for the
consent of the other. The latter may maintenance of the family, and the
object only on valid, serious, and moral husband shall be bound to reimburse
grounds. the expenses, if he has not delivered
the proper sum.

Private International Law Page 34


2. The wife cannot, without the or abroad, the following rules shall
husband's consent, acquire any prevail:
property by gratuitous title, except
from her relatives who are within the 1. If the husband is a citizen of the
prohibited degrees in marriage. Philippines while the wife is a
foreigner, the provisions of this
3. The wife may, with her husband's Code shall govern their relations;
consent, exercise any profession or 2. If the husband is a foreigner and
occupation or engage in lawful the wife is a citizen of the
business which is in keeping with Philippines, the laws of the
Islamic modesty and virtue. However, if husband's country shall be
the husband refuses to give his consent followed, without prejudice to the
on the ground that his income is provisions of this Code with
sufficient for the family according to its regard to immovable property.
social standing or his opposition is (1325a)
based on serious and valid grounds,
the matter shall be referred to the
Agama Arbitration Council. Art. 144. When a man and a woman live
together as husband and wife, but they
4. The wife shall have the right to are not married, or their marriage is
demand the satisfaction of her mahr. void from the beginning, the property
acquired by either or both of them
5. Unless otherwise stipulated in the through their work or industry or their
marriage settlements, the wife retain wages and salaries shall be governed
ownership and administration of her by the rules on co-ownership. (n)
exclusive property.
Family Code:
6. The wife shall be entitled to an equal
and just treatment by the husband.
Art. 74. A marriage in articulo mortis may also
be solemnized by the captain of a ship
2. PROPERTY RELATIONS or chief of an airplane during a voyage,
or by the commanding officer of a
Civil Code: military unit, in the absence of a
chaplain, during war. The duties
Art. 15, supra mentioned in the two preceding
articles shall be complied with by the
ship captain, airplane chief or
Art. 117. The wife may exercise any profession commanding officer. (n)
or occupation or engage in business.
However, the husband may object,
provided: Art. 75. Marriages between Filipino citizens
abroad may be solemnized by consuls
and vice-consuls of the Republic of the
1. His income is sufficient for Philippines. The duties of the local civil
the family, according to its registrar and of a judge or justice of the
social standing, and peace or mayor with regard to the
2. His opposition is founded on celebration of marriage shall be
serious and valid grounds. performed by such consuls and vice-
consuls. (n)
In case of disagreement on this
question, the parents and Art. 76. No marriage license shall be necessary
grandparents as well as the family when a man and a woman who have
council, if any, shall be consulted. If no attained the age of majority and who,
agreement is still arrived at, the court being unmarried, have lived together
will decide whatever may be proper as husband and wife for at least five
and in the best interest of the family. years, desire to marry each other. The
(n) contracting parties shall state the
foregoing facts in an affidavit before
Art. 118. The property relations between any person authorized by law to
husband and wife shall be governed in administer oaths. The official, priest or
the following order: minister who solemnized the marriage
a. By contract executed before the shall also state in an affidavit that he
marriage; took steps to ascertain the ages and
b. By the provisions of this Code; other qualifications of the contracting
and parties and that he found no legal
impediment to the marriage. (n)
c. By custom. (1315a)

Art. 77. In case two persons married in


Art. 124. If the marriage is between a citizen of
accordance with law desire to ratify
the Philippines and a foreigner,
their union in conformity with the
whether celebrated in the Philippines
regulations, rites, or practices of any

Private International Law Page 35


church, sect, or religion it shall no after the termination of their
longer be necessary to comply with the cohabitation.
requirements of Chapter 1 of this Title
and any ratification made shall merely When only one of the parties to a void
be considered as a purely religious marriage is in good faith, the share of
ceremony. (23) the party in bad faith in the co-
ownership shall be forfeited in favor of
Art. 80. The following marriages shall be void their common children. In case of
from the beginning: default of or waiver by any or all of the
1. Those contracted under the ages common children or their descendants,
of sixteen and fourteen years by each vacant share shall belong to the
the male and female respectively, respective surviving descendants. In
even with the consent of the the absence of descendants, such
parents; share shall belong to the innocent
2. Those solemnized by any person party. In all cases, the forfeiture shall
not legally authorized to perform take place upon termination of the
marriages; cohabitation. (144a)

3. Those solemnized without a Art. 148. In cases of cohabitation not falling


marriage license, save marriages under the preceding Article, only the
of exceptional character; properties acquired by both of the
parties through their actual joint
4. Bigamous or polygamous
contribution of money, property, or
marriages not falling under Article
industry shall be owned by them in
83, Number 2;
common in proportion to their
respective contributions. In the
5. Incestuous marriages mentioned
absence of proof to the contrary, their
in Article 81;
contributions and corresponding
6. Those where one or both shares are presumed to be equal. The
contracting parties have been same rule and presumption shall apply
found guilty of the killing of the to joint deposits of money and
spouse of either of them; evidences of credit.

7. Those between stepbrothers and If one of the parties is validly married


stepsisters and other marriages to another, his or her share in the co-
specified in Article 82. (n) ownership shall accrue to the absolute
community or conjugal partnership
Art. 147. When a man and a woman who are existing in such valid marriage. If the
capacitated to marry each other, live party who acted in bad faith is not
exclusively with each other as husband validly married to another, his or her
and wife without the benefit of shall be forfeited in the manner
marriage or under a void marriage, provided in the last paragraph of the
their wages and salaries shall be preceding Article.
owned by them in equal shares and
the property acquired by both of them The foregoing rules on forfeiture shall
through their work or industry shall be likewise apply even if both parties are
governed by the rules on co- in bad faith. (144a)
ownership.
Muslim Code:
In the absence of proof to the contrary,
properties acquired while they lived Art. 37. How governed. The property
together shall be presumed to have relations between husband and wife
been obtained by their joint efforts, shall be governed in the following
work or industry, and shall be owned order:
by them in equal shares. For purposes
of this Article, a party who did not
a. By contract before or at the time
participate in the acquisition by the
of the celebration of marriage;
other party of any property shall be
b. By the provisions of this Code;
deemed to have contributed jointly in
and
the acquisition thereof if the former's
efforts consisted in the care and
c. By custom.
maintenance of the family and of the
household.
Art. 38. Regime of property relations. The
property relations between the
Neither party can encumber or dispose
spouses, in the absence of any
by acts inter vivos of his or her share in
stipulation to the contrary in the
the property acquired during
marriage settlements or any other
cohabitation and owned in common,
contract, shall be governed by the
without the consent of the other, until
regime of complete separation of

Private International Law Page 36


property in accordance with this Code thereafter freely cohabited with
and, in a suppletory manner, by the the other as husband and wife;
general principles of Islamic law and
the Civil Code of the Philippines. 5. That either party was physically
incapable of consummating the
VII. Dissolution of Marriage marriage with the other, and such
incapacity continues and appears
to be incurable; or
1. Annulment
6. That either party was afflicted
Civil Code: with a sexually-transmissible
disease found to be serious and
Art. 66. When either or both of the contracting appears to be incurable. (85a)
parties are citizens or subjects of a
foreign country, it shall be necessary, Art. 46. Any of the following circumstances
before a marriage license can be shall constitute fraud referred to in
obtained, to provide themselves with a Number 3 of the preceding Article:
certificate of legal capacity to contract
marriage, to be issued by their 1. Non-disclosure of a previous
respective diplomatic or consular conviction by final judgment of
officials. (13a) the other party of a crime
involving moral turpitude;
Art. 71. All marriages performed outside the 2. Concealment by the wife of the
Philippines in accordance with the laws fact that at the time of the
in force in the country where they marriage, she was pregnant by a
were performed, and valid there as man other than her husband;
such, shall also be valid in this country,
except bigamous, polygamous, or 3. Concealment of sexually
incestuous marriages as determined by transmissible disease, regardless
Philippine law. (19a) of its nature, existing at the time
of the marriage; or

Family Code 4. Concealment of drug addiction,


habitual alcoholism or
Art. 45. A marriage may be annulled for any of homosexuality or lesbianism
the following causes, existing at the existing at the time of the
time of the marriage: marriage.

1. That the party in whose behalf it No other misrepresentation or deceit


is sought to have the marriage as to character, health, rank, fortune or
annulled was eighteen years of chastity shall constitute such fraud as
age or over but below twenty- will give grounds for action for the
one, and the marriage was annulment of marriage. (86a)
solemnized without the consent
of the parents, guardian or person Art. 47. The action for annulment of marriage
having substitute parental must be filed by the following persons
authority over the party, in that and within the periods indicated
order, unless after attaining the herein:
age of twenty-one, such party
freely cohabited with the other
1. For causes mentioned in number
and both lived together as
1 of Article 45 by the party whose
husband and wife;
parent or guardian did not give his
2. That either party was of unsound
or her consent, within five years
mind, unless such party after
after attaining the age of twenty-
coming to reason, freely
one, or by the parent or guardian
cohabited with the other as
or person having legal charge of
husband and wife;
the minor, at any time before such
3. That the consent of either party party has reached the age of
was obtained by fraud, unless twenty-one;
such party afterwards, with full 2. For causes mentioned in number
knowledge of the facts 2 of Article 45, by the same
constituting the fraud, freely spouse, who had no knowledge of
cohabited with the other as the other's insanity; or by any
husband and wife; relative or guardian or person
having legal charge of the insane,
4. That the consent of either party at any time before the death of
was obtained by force, either party, or by the insane
intimidation or undue influence, spouse during a lucid interval or
unless the same having after regaining sanity;
disappeared or ceased, such party

Private International Law Page 37


3. For causes mentioned in number In the partition, the conjugal dwelling
3 of Article 45, by the injured and the lot on which it is situated, shall
party, within five years after the be adjudicated in accordance with the
discovery of the fraud; provisions of Articles 102 and 129.

4. For causes mentioned in number Art. 51. In said partition, the value of the
4 of Article 45, by the injured presumptive legitimes of all common
party, within five years from the children, computed as of the date of
time the force, intimidation or the final judgment of the trial court,
undue influence disappeared or shall be delivered in cash, property or
ceased; sound securities, unless the parties, by
mutual agreement judicially approved,
5. For causes mentioned in number
had already provided for such matters.
5 and 6 of Article 45, by the
injured party, within five years
after the marriage. (87a) The children or their guardian or the
trustee of their property may ask for
the enforcement of the judgment.
Art. 48. In all cases of annulment or declaration
of absolute nullity of marriage, the
Court shall order the prosecuting The delivery of the presumptive
attorney or fiscal assigned to it to legitimes herein prescribed shall in no
appear on behalf of the State to take way prejudice the ultimate
steps to prevent collusion between the successional rights of the children
parties and to take care that evidence accruing upon the death of either of
is not fabricated or suppressed. both of the parents; but the value of
the properties already received under
the decree of annulment or absolute
In the cases referred to in the
nullity shall be considered as advances
preceding paragraph, no judgment
on their legitime. (n)
shall be based upon a stipulation of
facts or confession of judgment. (88a)
Art. 52. The judgment of annulment or of
absolute nullity of the marriage, the
Art. 49. During the pendency of the action and
partition and distribution of the
in the absence of adequate provisions
properties of the spouses and the
in a written agreement between the
delivery of the children's presumptive
spouses, the Court shall provide for the
legitimes shall be recorded in the
support of the spouses and the
appropriate civil registry and registries
custody and support of their common
of property; otherwise, the same shall
children. The Court shall give
not affect third persons. (n)
paramount consideration to the moral
and material welfare of said children
and their choice of the parent with Art. 53. Either of the former spouses may
whom they wish to remain as provided marry again after compliance with the
to in Title IX. It shall also provide for requirements of the immediately
appropriate visitation rights of the preceding Article; otherwise, the
other parent. (n) subsequent marriage shall be null and
void.chan robles virtual law library
Art. 50. The effects provided for by paragraphs
(2), (3), (4) and (5) of Article 43 and by Art. 54. Children conceived or born before the
Article 44 shall also apply in the proper judgment of annulment or absolute
cases to marriages which are declared nullity of the marriage under Article 36
ab initio or annulled by final judgment has become final and executory shall
under Articles 40 and 45. be considered legitimate. Children
conceived or born of the subsequent
marriage under Article 53 shall likewise
The final judgment in such cases shall
be legitimate.
provide for the liquidation, partition
and distribution of the properties of
the spouses, the custody and support Art. 55. A petition for legal separation may be
of the common children, and the filed on any of the following grounds:
delivery of third presumptive legitimes,
unless such matters had been 1. Repeated physical violence
adjudicated in previous judicial or grossly abusive conduct
proceedings. directed against the
petitioner, a common child,
or a child of the petitioner;
All creditors of the spouses as well as
2. Physical violence or moral
of the absolute community or the
pressure to compel the
conjugal partnership shall be notified
petitioner to change religious
of the proceedings for liquidation.
or political affiliation;

Private International Law Page 38


3. Attempt of respondent to union or brings danger, dishonor
corrupt or induce the or material injury upon the other,
petitioner, a common child, the injured party may petition the
or a child of the petitioner, to court for relief. The court may
engage in prostitution, or counsel the offender to comply
connivance in such with his or her duties, and take
corruption or inducement; such measures as may be
proper. .chan robles virtual law
4. Final judgment sentencing library
the respondent to
imprisonment of more than 3. The husband and the wife shall
six years, even if pardoned; inherit from each other in
accordance with this Code.
5. Drug addiction or habitual
alcoholism of the 4. The husband and the wife shall
respondent; have the right to divorce in
accordance with this Code.
6. Lesbianism or homosexuality
of the respondent;

7. Contracting by the
respondent of a subsequent ROEHR VS. RODRIGUEZ
bigamous marriage, whether
in the Philippines or abroad;
Facts:
8. Sexual infidelity or
perversion; Petitioner Wolfgang Roehr, a German citizen, married a
Filipina, Carmen Rodriguez in Germany. The marriage was
9. Attempt by the respondent ratified in Tayasan, Negros Oriental. Private respondent
against the life of the filed a petition for the declaration of nullity of marriage
petitioner; or before the RTC of Makati. Petitioner filed a motion to
dismiss but was denied by the trial court. The petitioner
10. Abandonment of petitioner obtained a decree of divorce from the Court of First
by respondent without Instance of Hamburg - Blankenese and granting the custody
justifiable cause for more of the children to the father.
than one year.
Issue:
For purposes of this Article, the term
"child" shall include a child by nature Whether or not the legal effects of a divorce obtained from
or by adoption. (9a) a foreign country such as support and custody of the
2. Absolute Divorce children can be determined in our courts?

Held:
Civil Code:
Yes. In order to take effect, a foreign judgement must
Art. 15, supra clearly show that the opposing party has been given
ample opportunity to do so under the Rules of Civil
Art. 17, Supra Procedure. Accordingly, the respondent was not given the
opportunity to challenge the judgement of the German
Muslim Code: Court, therefore, legal effects of divorce must be
determined in our courts. The court held that the trial court
has jurisdiction over the issue between the parties as to
45-55 supra who has the parental custody.

Art. 27. By a husband. Notwithstanding the rule of


Islamic law permitting a Muslim to RECTO VS. HARDEN (supra)
have more than one wife but one wife
unless he can deal with them with
equal companionship and just
treatment as enjoined by Islamic law GONZALES VS. GONZALES
and only in exceptional cases.

Art. 34. Mutual rights and obligations.


Facts:
1. The husband and the wife are
obliged to live together, observe Plaintiff and defendant are citizens of the Philippine Islands
mutual respect and fidelity, and and at present residents of the City of Manila. They were
render mutual help and support in married in the City of Manila on January 19, 1919, and
accordance with this Code. lived together as man and wife in the Philippine Islands
2. When one of the spouses neglects until the spring of 1926. They voluntarily separated and
his or her duties to the conjugal since that time have not lived together as man and wife. Of

Private International Law Page 39


this union four children were born. Negotiations between It is therefore a serious question whether any foreign
the parties, both being represented by attorneys, divorce relating to citizens of the Philippine Islands, will be
whereupon it was mutually agreed to allow the plaintiff for recognized in this jurisdiction, except it be for a cause, and
her support and that of her children, five hundred pesos under conditions for which the courts of Philippine Islands
(P500) monthly; this amount to be increased in case of would grant a divorce. The lower court in granting relief as
illness or necessity, and the title of certain properties to be prayed for frankly stated that the securing of the divorce,
put in her name. Shortly after this agreement the husband the contracting of another marriage and the bringing into
left the Islands, betook himself to Reno, Nevada, and the world of innocent children brings about such a
secured in that jurisdiction an absolute divorce on the condition that the court must grant relief. The hardships of
ground of desertion. Shortly thereafter the defendant the existing divorce laws of the Philippine Islands are well
moved to California and returned to these Islands in known to the members of the Legislature. It is of no
August 1928, where he has since remained. On the same moment in this litigation what he personal views of the
date that he secured a divorce in Nevada he went through writer on the subject of divorce may be. It is the duty of
the forms of marriage with another citizen of these Islands the courts to enforce the laws of divorce as written by the
and now has three children as a result of that marriage. Legislature if they are constitutional. Courts have no right
Defendant, after his departure from these Islands, reduced to say that such laws are too strict or too liberal.
the amount he had agreed to pay monthly for the support
of his wife and four minor children and has not made the TENCHAVEZ V. ESCANO 15 SCRA 355
payments fixed in the Reno divorce as alimony.
Facts:
Issue:

Whether or not that the divorced acquired in Nevada is Pastor Tenchavez married Vicenta Escano on Feb. 24,
valid here in the Philippines? 1948 in Cebu City. As of June 1948, the newly-weds
were already estranged. Vicenta left for the US and
Held filed a verified complaint for divorce against the
Pastor in the State of Nevada on the ground of
While the parties in this action are in dispute over financial "extreme cruelty, entirely mental in character."
matters they are in unity in trying to secure the courts of A decree of divorce was issued by the Nevada Court.
this jurisdiction to recognize and approve of the Reno Later on, Vicenta married an American, Russell Leo
divorce. On the record here presented this can not be Moran in Nevada. She now lives with him in California
done. The public policy in this jurisdiction on the question and has begotten children. She acquired American
of divorce is clearly set forth in Act No. 2710, and the citizenship on August 8, 1958. On July 30, 1955,
decisions of this court. Pastor filed a complaint for legal separation and
damages against Vicenta and her parents in the CFI-
Cebu.
The entire conduct of the parties from the time of their
separation until the case was submitted to this court, in HELD:
which they all prayed that the Reno divorce be ratified and
confirmed, clearly indicates a purpose to circumvent the At the time the divorce decree was issued, Vicenta,
laws of the Philippine Islands regarding divorce and to like her husband, was still a Filipino citizen. She was
secure for themselves a change of status for reasons and then subject to Philippine laws under Art. 15 of the
under conditions not authorized by our law. At all times New Civil Code. Philippine law, under the NCC then
the matrimonial domicile of this couple has been within now in force, does not admit absolute divorce but
the Philippine Islands and the residence acquired in the only provides for legal separation.
State of Nevada by the husband of the purpose of securing For Philippine courts to recognize foreign divorce
a divorce was not a bona fide residence and did not confer decrees between Filipino citizens would be a patent
jurisdiction upon the Court of that State to dissolve the violation of the declared policy of the State,
bonds if matrimony in which he had entered in 1919. especially in view of the 3rd par. of Art. 17, of the
While the decisions of this court heretofore in refusing to New Civil Code which reads: Prohibitive laws
recognize the validity of foreign divorce has usually been concerning persons, their acts or property, and those
expressed in the negative and have been based upon lack which have, for their object, public order, public
of matrimonial domicile or fraud or collusion, we have not policy and good customs shall not be rendered
overlooked the provisions of the Civil Code now in force in ineffective by laws or judgments promulgated, or by
these Islands. Article 9 thereof reads as follows: determinations or conventions agreed upon in a
foreign country. Moreover, recognition would give
The laws relating to family rights and duties, or rise to scandalous discrimination in favor of wealthy
to the status, condition and legal capacity or citizens to the detriment of those members of our
persons, are binding upon Spaniards even society whose means do not permit them to sojourn
though they reside in a foreign country. abroad and obtain absolute divorce outside the
Philippines.
And article 11, the last part of which reads:
Therefore, a foreign divorce between Filipino citizens,
. . . the prohibitive laws concerning persons, sought and decreed after the effectivity of the NCC, is
their acts and their property, and those not entitled to recognition as valid in this jurisdiction.
intended to promote public order and good
morals, shall nor be rendered without effect by SIKAT V. CANSON
any foreign laws or judgments or by anything 67 PHIL 207
done or any agreements entered into a foreign
country.

Private International Law Page 40


Hilaria Sikat and John Canson contracted marriage That our divorce law, Act No. 2710, is too strict or too
and lived together as husband and wife until 1911 liberal is not for this court to decide. (Barretto
when they separated. Gonzalez vs. Gonzalez, supra.)
During the same year the wife commenced divorce The allotment of powers between the different
proceedings against her spouses, but the case was governmental agencies restricts the judiciary within
dismissed without the court passing upon the merits the confines of interpretation, not of legislation. The
thereof. legislative policy on the matter of divorce in this
At the time of their marriage in 1904, John Canson jurisdiction is clearly set forth in Act No. 2710 and has
was an Italian citizen but on February 27, 1922, he been upheld by this court;
became a naturalized Filipino citizen.
In 1929, he went to Reno, Nevada, United States of ARCA V. JAVIER
America, and on October 8, of that year, he obtained 95 PHIL 579
an absolute decree of divorce on the ground of
desertion. Dissatisfied with the decision of the Court of First
Hilaria, in 1933, filed another action, civil case No. Instance of Cavite ordering him to give a monthly
5398 of the Court of First Instance of Rizal, wherein allowance of P60 to plaintiffs beginning March 31,
she sought to compel the defendant to pay her a 1953, and to pay them attorney's fees in the amount
monthly pension of P500 as alimony or support. of P150 defendant took the case directly to this Court
Canson interposed three defenses: (1) adultery on the attributing five errors to the court below. The facts are
part of the plaintiff: (2) absolute divorce obtained by not disputed.
the defendant as decreed by the court in Reno,
Nevada, United States of America; and (3) that the Javier and Arca got married in Manila. Javier, an
defendant did not have the means to pay the enlisted US Navy personnel left for the States 7 years
allowance sought. The lower court dismissed the after the birth of their first born. At such time, Arca
complaint and declined to accord validity to the lived with Javiers parents. However, due to strained
divorce obtained in Reno but found that Hilaria Sikat relations with the latter, she left and transferred to
had forfeited her right to support because she had her hometown. Thereafter, Javier filed a case for
committed adultery. divorce in Alabama against Arca alleging
This judgment was not appealed and it became final. abandonment. Arca made her reply claiming among
On June 1, 1934, the present action was instituted by other things that she never abandoned her husband
the plaintiff-appellant to obtain the liquidation of the and that their separation was due to a physical
conjugal partnership. The action is predicated on the impossibility which justifies her separation if the
existence of a final decree of absolute divorce husband moves to ultra-marine colonies. The divorce
rendered by the court of Reno, Nevada, since 1929. however was granted.
Javier got married the 2 nd time but was subsequently
Held: divorced. After a few years, he went back to the
Philippines, and believing that the first two divorces
It is not, however, the citizenship of the plaintiff for were valid, remarried the 3rd time.
divorce which confers jurisdiction upon a court, but
his legal residence within the State (Cousins Hix vs. Issue:
Fluemer, 55 Phil. 851). And assuming that John
Canson acquired legal residence in the State of Did the Circuit Court of Mobile County acquire jurisdiction
Nevada through the approval of his citizenship of both spouses and effectively rendered a judgment in
papers, this did not confer jurisdiction on the Nevada rem when it granted divorce to Javier?
court to grant a divorce that would be valid in this
jurisdiction nor jurisdiction that could determine their Held:
matrimonial status, because the wife was still
domiciled in the Philippines. The Nevada court never No, one of the essential conditions for the validity of
acquired jurisdiction over her person. (Gorayeb vs. a decree of divorce is that the court must have
Hashim, 50 Phil. 26, and Cousins Hix vs. Fluemer, jurisdiction over the subject matter and in order that
supra.) This was not a proceeding in rem to justify a this may be acquired, plaintiff must be domiciled in
court in entering a decree as to the res or marriage good faith in the State in which it is granted (Cousins
relation entitled to be enforced outside of the Hix vs. Fluemer, 55 Phil., 851, 856).
territorial jurisdiction of the court. While it is true that Salud R. Arca filed an answer in
Plaintiff-appellant had made her choice of two the divorce case instituted at the Mobile County in
inconsistent remedies afforded her by law: (1) to view of the summons served upon her in this
impugn the divorce and file an action for support, or jurisdiction, but this action cannot be interpreted as
(2) uphold the validity of the divorce and sue for a placing her under the jurisdiction of the court
liquidation of conjugal partnership. She chose the because its only purpose was to impugn the claim of
first remedy when she filed her action for support. appellant that his domicile or legal residence at that
She lost the case and should take the consequences. time was Mobile County, and to show that the
The courts in the Philippines can grant a divorce only ground of desertion imputed to her was baseless and
on the ground of "adultery on the part of the wife or false. Such answer should be considered as a special
concubinage on the part of the husband" as appearance the purpose of which is to impugn the
provided for under section 1 of Act No. 2710. The jurisdiction of the court over the case.
divorce decree in question was granted on the ground It is established by the great weight of authority that
of desertion, clearly not a cause for divorce under our the court of a country in which neither of the
laws. spouses is domiciled and to which one or both of
them may resort merely for the purpose of obtaining
a divorce has no jurisdiction to determine their
matrimonial status; and a divorce granted by such a

Private International Law Page 41


court is not entitled to recognition elsewhere. (See It is established by the great weight of authority that the
Note to Succession of Benton, 59 L. R. A., 143) The court of a country in which neither of the spouses is
voluntary appearance of the defendant before such a domiciled and to which one or both of them may resort
tribunal does not invest the court with jurisdiction. merely for the purpose of obtaining a divorce has no
(Andrews vs. Andrews, 188 U. S., 14; 47 L. ed., 366.) jurisdiction to determine their matrimonial status; and a
It follows that, to give a court jurisdiction on the divorce granted by such a court is not entitled to
ground of the plaintiff's residence in the State or recognition elsewhere.
country of the judicial forum, his residence must be
bona fide. If a spouse leaves the family domicile and It follows that, to give a court jurisdiction on the ground
goes to another State for the sole purpose of of the plaintiff's residence in the State or country of the
obtaining a divorce, and with no intention of judicial forum, his residence must be bona fide. If a
remaining, his residence there is not sufficient to spouse leaves the family domicile and goes to another
confer jurisdiction on the courts of the State. (Ramirez State for the sole purpose of obtaining a divorce, and with
vs. Gmur, 82 Phil., 855.) no intention of remaining, his residence there is not
But even if his residence had been taken up is good sufficient to confer jurisdiction on the courts of that State.
faith, and the court had acquired jurisdiction to take This is especially true where the cause of divorce is one
cognizance of the divorce suit, the decree issued in his not recognized by the laws of the State of his own
favor is not binding upon the appellant; for the domicile.
matrimonial domicile of the spouses being the City of
Manila;

Ramirez v. Gmur
42 Phil 855 MANILA SURETY & FIDELITY VS. TEODORO

Leona Castro was the natural daughter of decedent Samuel FACTS:


Bischoff. Whereas Ana Ramirez was the latter's widow to
which they had no children. Leona was married to Jose Corominas, Jr. and Sonia Lizares were married in Iloilo
Kauffman. They had 3 children, Elena, Federico, and on January 5, 1935. On November 29,1954, a decree of
Ernesto. Later, Kauffman brought Leona to Switzerland to divorce was granted by the Court of the State of Nevada
recuperate her health. A few years later, she fell for a Dr. dissolving the bonds of matrimony between Sonia Lizares
Mory to whom she had a child, Leontina. She informed and Jose Corominas, Jr. . . .
Kauffman that she no longer wished to stay with him to
which the latter obtained a divorce in France where Leona
Trinidad Teodoro met Jose Corominas, Jr. in Hongkong on
was in default. Leona, after the divorce got married in
October 30, 1955. . . . On March 26, 1956, they went
London, and after which bore two children from which the
through a Buddhist wedding ceremony in Hongkong. Upon
last childbirth caused Leona her life. The heirs of Leona
their return to the Philippines they took up residence in a
from the first marriage and the second now claims the
rented house at No. 2305 Agno Street . . . Manila. On
Estate of Samuel to which Ana Ramirez opposed since
September 5, 1961, plaintiff and Jose Corominas, Jr. were
Leona is not a recognized natural child.
married for a second time on Washoe County, Nevada.
U.S.A.
Held:

The status of Leona Castro as recognized natural daughter Additional Pertinent facts, also mentioned in the decision
of Samuel Bischoff is fully and satisfactorily shown. under review and controverted by the parties, are that
Sonia Lizares is still living and that the conjugal partnership
With reference to the right of the von Kauffman children, formed by her marriage to Corominas was dissolved by the
it is enough to say that they are legitimate children, born Juvenile and Domestic Relations Court of Manila upon
to their parents in lawful wedlock; and they are therefore their joint petition, the decree of dissolution having been
entitled to participate in the inheritance which would have issued on October 21, 1957. Trinidad questioned the levy
devolved upon their mother, if she had survived the on the property since the property in question was her
testator. paraphernal property.

The Court is of the opinion that the decree of divorce ISSUE:


upon which reliance is placed by the representation of
the Mory children cannot be recognized as valid in the Whether or not the properties in question are conjugal?
courts of the Philippine Islands. The French tribunal has
no jurisdiction to entertain an action for the dissolution of
RULING:
a marriage contracted in these Islands by person domiciled
here, such marriage being indissoluble under the laws then
There is no doubt that the decree of divorce granted by
prevailing in this country.
the Court of Nevada in 1954 is not valid under Philippine
law, which has outlawed divorce altogether; that the
The evidence shows that the decree was entered against
matrimonial bonds between Jose Corominas, Jr. and Sonia
the defendant in default, for failure to answer, and there is
Lizares have not been dissolved, although their conjugal
nothing to show that she had acquired, or had attempted
partnership was terminated in 1957; and that the former's
to acquire, a permanent domicile in the City of Paris. It is
subsequent marriage in Hongkong to Trinidad Teodoro is
evident of course that the presence of both the spouses in
bigamous and void.
that city was due merely to the mutual desire to procure a
divorce from each other.
In the present case, however, we find no need to pass on
this question. The particular properties involved here

Private International Law Page 42


which were admittedly acquired by respondent Teodoro, discriminated against in her own country if the ends
cannot be deemed to belong to such co-ownership of justice are to be observed.
because, as found by the trial court and confirmed by the
Court of Appeals, the funds used in acquiring said
properties were fruits of respondent's paraphernal
investments which accrued before her "marriage" to PILAPIL V. IBAY-SOMERA
Corominas. In other words they were not acquired by 174 SCRA 653
either or both of the partners in the void marriage through
their work or industry or their wages and salaries, and
Petitioner Imelda Manalaysay Pilapil, a Filipino citizen,
hence cannot be the subject of co-ownership under Article
and private respondent Erich Ekkehard Geiling, a
144. They remain respondent's exclusive properties,
German national, were married in the Federal
beyond the reach of execution to satisfy the judgment
Republic of Germany. The marriage started
debt of Corominas.
auspiciously enough, and the couple lived together
for some time in Malate, Manila where their only
VAN DORN V. ROMILLO
child, Isabella Pilapil Geiling, was born on April 20,
139 SCRA 139
1980.
Thereafter, marital discord set in, with mutual
Facts: recriminations between the spouses, followed by a
separation de facto between them.
Petitioner Alice Reyes Van Dorn is a citizen of the
Philippines while private respondent Richard Upton is After about three and a half years of marriage, private
a US citizen; they were married in Hong Kong in 1972; respondent initiated a divorce proceeding against
after the marriage, they established their residence in petitioner in Germany. He claimed that there was
the Philippines and begot 2 children; Alicia filed for failure of their marriage and that they had been living
divorce in Nevada; the parties were divorced in apart since April, 1982. Petitioner, on the other hand,
Nevada, US, in 1982; and petitioner has remarried filed an action for legal separation, support and
also in Nevada, this time to Theodore Van Dorn. separation of property before the Regional Trial Court
On June 18, 1983 Upton filed a suit against petitioner of Manila.
in the RTC-Pasay, stating that petitioner's business in
Ermita, Manila (the Galleon Shop), is conjugal
property and asking that petitioner be ordered to Thereafter a decree of divorce was promulgated. The
render an accounting of that business, and that Upton records show that under German law said court was
be declared as having the right to manage the locally and internationally competent for the divorce
conjugal property. proceeding and that the dissolution of said marriage
was legally founded on and authorized by the
applicable law of that foreign jurisdiction.
Held:
More than five months after the issuance of the
Owing to the nationality principle embodied in Art. divorce decree, private respondent filed two
13, NCC, only Philippine nationals are covered by the complaints for adultery before the City Fiscal of
policy against absolute divorce the same being Manila alleging that, while still married to said
considered contrary to our concept of public policy respondent, petitioner "had an affair with a certain
and morality. However, aliens may obtain divorce William Chua as early as 1982 and with yet another
abroad, which may be recognized in the Philippines man named Jesus Chua sometime in 1983".
provided they are valid according to their national
law.
Issue:
In this case, the divorce in Nevada released private
respondents from the marriage from the standards
of American law, under which divorce dissolves the WON the adultery case be sustained even though there
marriage. has already been a finality of a divorce decree.

Court said that Ours is not only a court of law but Held:
also a court of equity. The Court could not turn its
back on its citizen when the foreign national itself The law specifically provides that in prosecutions for
benefited from such divorce decree; adultery and concubinage the person who can legally
file the complaint should be the offended spouse, and
Thus, pursuant to his national law, Upton is no nobody else.
longer the husband of petitioner. He would have no Corollary to such exclusive grant of power to the
standing to sue in the case below as petitioner's offended spouse to institute the action, it necessarily
husband who is entitled to exercise control over follows that such initiator must have the status,
conjugal assets. capacity or legal representation to do so at the time
of the filing of the criminal action. This is a familiar
and express rule in civil actions; in fact, lack of legal
To maintain, as Upton does, that under our laws,
capacity to sue, as a ground for a motion to dismiss in
petitioner has to be considered still married to him
civil cases, is determined as of the filing of the
and still subject to a wife's obligations under the NCC
complaint or petition. In these cases, therefore, it is
cannot be just. Petitioner should not be obliged to live
indispensable that the status and capacity of the
together with, observe respect and fidelity, and
complainant to commence the action be definitely
render support to private respondent. The latter
established and, as already demonstrated, such status
should not continue to be one of her heirs w/ possible
rights to conjugal properties. She should not be

Private International Law Page 43


or capacity must indubitably exist as of the time he in the United States. Each State of the union has its own
initiates the action. law applicable to its citizens and in force only within the
State. It can therefore refer to no other than the law of
In the present case, the fact that private respondent the State of which the decedent was a resident. Second,
obtained a valid divorce in his country, the Federal there is no showing that the application of the renvoi
Republic of Germany, it is deemed admitted. Said doctrine is called for or required by New York State law.
divorce and its legal effects may be recognized in the
Philippines insofar as private respondent is concerned However, intestate and testamentary succession, both
in view of the nationality principle in our civil law on with respect to the order of succession and to the amount
the matter of status of persons. of successional rights and to the intrinsic validity of
testamentary provisions, shall be regulated by the
Therefore, private respondent, being no longer the national law of the person whose succession is under
husband of petitioner, had no legal standing to consideration, whatever may be the nature of the
commence the adultery case under the imposture property and regardless of the country wherein said
that he was the offended spouse at the time he filed property may be found. (emphasis ours)
suit.

Likewise, Lorenzo Llorente was already an American


citizen when he divorced Paula. Such was also the
LLORENTE V. COURT OF APPEALS situation when he married Alicia and executed his will. As
GR No. 124371, November 23, 2000 stated in Article 15 of the civil code, aliens may obtain
divorces abroad, provided that they are valid in their
National Law. Thus the divorce obtained by Llorente is
FACTS: valid because the law that governs him is not Philippine
Law but his National Law since the divorce was contracted
after he became an American citizen. Furthermore, his
Lorenzo Llorente and petitioner Paula Llorente were National Law allowed divorce.
married in 1937 in the Philippines. Lorenzo was an
enlisted serviceman of the US Navy. Soon after, he left for
the US where through naturalization, he became a US The case was remanded to the court of origin for
Citizen. Upon his visit to his wife, he discovered that she determination of the intrinsic validity of Lorenzo Llorentes
was living with his brother and a child was born. The child will and determination of the parties successional rights
was registered as illegitimate but the name of the father allowing proof of foreign law.
was left blank. Llorente filed a divorce in California in
which Paula was represented by counsel, John Riley, and GARCIA V. RECIO
actively participated in the proceedings, which later on October 2, 2001
became final. He married Alicia and they lived together for
25 years bringing 3 children. He made his last will and
Rederick Recio, a Filipino, married Editha Samson, an
testament stating that all his properties will be given to his
Australian in Malabon Rizal. However, on 1989, they
second marriage. He filed a petition of probate that made
got divorced in an Australian family court.
or appointed Alicia his special administrator of his estate.
On 1992, Rederick became an Australian Citizen. He
Before the proceeding could be terminated, Lorenzo died.
later married Petitioner in 1994 in Cabanatuan City.
Paula filed a letter of administration over Llorentes estate.
Thereafter, the two separated and petitioner filed a
The trial granted the letter and denied the motion for
complaint for Declaration of Nullity of Marriage on
reconsideration. An appeal was made to the Court of
the ground of bigamy.
Appeals, which affirmed and modified the judgment of the
While the suit was pending, Rederick was able to
Trial Court that she be declared co-owner of whatever
obtain a divorce decree in Australia. Trial Court
properties, she and the deceased, may have acquired in
declared the marriage dissolved based on the
their 25 years of cohabitation.
subsequent divorce decree obtained by the
respondent.
ISSUE:
Issues:

Whether or not national law shall apply? Whether the divorce between respondent and Editha
Samson was proven;
RULING:
Whether respondent was proven to be legally capacitated
to marry petitioner;
Art. 15. Laws relating to family rights and duties, or to the
status, condition and legal capacity of persons are binding
Held:
upon citizens of the Philippines, even though living
abroad.
A divorce obtained abroad by an alien may be recognized
in our jurisdiction, provided such decree is valid according
Art. 16. Real property as well as personal property is to the national law of the foreigner. However, the divorce
subject to the law of the country where it is situated. decree and the governing personal law of the alien
spouse who obtained the divorce must be proven. Our
First, there is no such thing as one American law. The courts do not take judicial notice of foreign laws and
"national law" indicated in Article 16 of the Civil Code judgment; hence, like any other facts, both the divorce
cannot possibly apply to general American law. There is no decree and the national law of the alien must be alleged
such law governing the validity of testamentary provisions and proven according to our law on evidence.

Private International Law Page 44


vinculo matrimonii and (2) limited divorce or a mensa et
Was the first divorce validly obtained and binding? thoro. The first kind terminates the marriage, while the
second suspends it and leaves the bond in full force. There
At the outset, the Court lays the following basic legal is no showing in the case at bar which type of divorce was
principles; Philippine law does not provide for absolute procured by respondent.
divorce; hence, Philippine courts cannot grant it. A
marriage between two Filipinos cannot be dissolved even Respondent presented a decree nisi or an interlocutory
by a divorce obtained abroad, because of Articles 1522 and decree a conditional or provisional judgment of divorce.
1723 of the Civil Code. In mixed marriages involving a It is in effect the same as a separation from bed and
Filipino and a foreigner, Article 26 of the Family Code board, although an absolute divorce may follow after the
allows the former to contract a subsequent marriage in lapse of the prescribed period during which no
case the divorce is "validly obtained abroad by the alien reconciliation is effected.
spouse capacitating him or her to remarry." A divorce
obtained abroad by a couple, who are both aliens, may be On its face, the herein Australian divorce decree contains a
recognized in the Philippines, provided it is consistent with restriction that reads:
their respective national laws.
"1. A party to a marriage who marries again before this
Before a foreign divorce decree can be recognized by our decree becomes absolute (unless the other party has
courts, the party pleading it must prove the divorce as a died) commits the offence of bigamy."
fact and demonstrate its conformity to the foreign law
allowing it. Presentation solely of the divorce decree is This quotation bolsters the Courts contention that the
insufficient. In the case at bar, Respondent only presented divorce obtained by respondent may have been restricted.
the divorce decree; It did not absolutely establish his legal capacity to remarry
according to his national law.
Likewise, before a foreign judgment is given presumptive
evidentiary value, the document must first be presented Significance of the Certificate of Legal Capacity
and admitted in evidence. A divorce obtained abroad is
proven by the divorce decree itself. Indeed the best Legal capacity to contract marriage is determined by the
evidence of a judgment is the judgment itself. The decree national law of the party concerned. The certificate
purports to be a written act or record of an act of an mentioned in Article 21 of the Family Code would have
officially body or tribunal of a foreign country. been sufficient to establish the legal capacity of
respondent, had he duly presented it in court. A duly
However, under Sections 24 and 25 of Rule 132, a writing authenticated and admitted certificate is prima facie
or document may be proven as a public or official record of evidence of legal capacity to marry on the part of the
a foreign country by either (1) an official publication or (2) alien applicant for a marriage license.
a copy thereof attested by the officer having legal custody
of the document. If the record is not kept in the In the case at bar, there is absolutely no evidence that
Philippines, such copy must be (a) accompanied by a proves respondent's legal capacity to marry petitioner.
certificate issued by the proper diplomatic or consular
officer in the Philippine Foreign Service stationed in the Based on the above records, the Court cannot conclude
foreign country in which the record is kept and (b) that respondent, who was then a naturalized Australian
authenticated by the seal of his office. citizen, was legally capacitated to marry petitioner on
January 12, 1994. The court a quo erred in finding that the
The divorce decree between respondent and Editha divorce decree ipso facto clothed respondent with the
Samson appears to be an authentic one issued by an legal capacity to remarry without requiring him to adduce
Australian family court. However, appearance is not sufficient evidence to show the Australian personal law
sufficient; compliance with the aforementioned rules on governing his status; or at the very least, to prove his legal
evidence must be demonstrated. capacity to contract the second marriage.

Fortunately for respondent, this matter was not objected Neither can the Court grant petitioner's prayer to declare
to by the petitioner, thus by virtue of such waiver, is her marriage to respondent null and void on the ground
deemed admitted as evidence. of bigamy. After all, it may turn out that under Australian
law, he was really capacitated to marry petitioner as a
Who has the burden of proving a foreign law? direct result of the divorce decree.

Respondent has the burden of proof; The burden of proof Hence, case was remanded to the court a quo for further
lies with "the party who alleges the existence of a fact or determination of legal capacity and to receive evidence to
thing necessary in the prosecution or defense of an determine if bigamy has been committed;
action." It is a well-settled that courts cannot take judicial
notice of foreign laws. Like any other facts, they must be REPUBLIC V. ORBECIDO
alleged and proved. Australian marital laws are not among GR NO. 154380, October 5, 2005
those matters that judges are supposed to know by reason
of their judicial function. The power of judicial notice must Facts:
be exercised with caution, and every reasonable doubt
upon the subject should be resolved in the negative.
On May 24, 1981, Cipriano Orbecido III married Lady
nd Myros M. Villanueva in the Philippines in Lam-an, Ozamis
2 Issue: Is Respondent legally capacitated to remarry?
City. Their marriage was blessed with a son and a
daughter.
Divorce means the legal dissolution of a lawful union for a
cause arising after marriage. But divorces are of different
types. The two basic ones are (1) absolute divorce or a

Private International Law Page 45


In 1986, Ciprianos wife left for the United States bringing 1. For adultery on the part of the
along their son Kristoffer. A few years later, Cipriano wife and for concubinage on the
discovered that his wife had been naturalized as an part of the husband as defined in
American citizen. the Penal Code; or

Sometime in 2000, Cipriano learned from his son that his 2. An attempt by one spouse against
wife had obtained a divorce decree and then married a the life of the other. (n)
certain Innocent Stanley. She, Stanley and her child by him
currently live in San Gabriel, California. Art. 99. No person shall be entitled to a legal
separation who has not resided in the
Philippines for one year prior to the
Cipriano thereafter filed with the trial court a petition for
filing of the petition, unless the cause
authority to remarry invoking Paragraph 2 of Article 26 of
for the legal separation has taken place
the Family Code. No opposition was filed. Finding merit in
within the territory of this Republic.
the petition, the court granted the same. The Republic,
(Sec. 2a, Act No. 2710)
herein petitioner, through the Office of the Solicitor
General (OSG), sought reconsideration but it was denied.
TITLE II

The OSG contends that Paragraph 2 of Article 26 of the LEGAL SEPARATION


Family Code is not applicable to the instant case because
it only applies to a valid mixed marriage; that is, a Art. 55. A petition for legal separation may be filed on any
marriage celebrated between a Filipino citizen and an of the following grounds:
alien. Furthermore, the OSG argues there is no law that (1) Repeated physical violence or grossly abusive
governs respondents situation. The OSG posits that this is conduct directed against the petitioner, a
a matter of legislation and not of judicial determination. common child, or a child of the petitioner;

Held: (2) Physical violence or moral pressure to compel


the petitioner to change religious or political
Taking into consideration the legislative intent and affiliation;
applying the rule of reason, we hold that Paragraph 2 of
Article 26 should be interpreted to include cases involving (3) Attempt of respondent to corrupt or induce
parties who, at the time of the celebration of the the petitioner, a common child, or a child of the
marriage were Filipino citizens, but later on, one of them petitioner, to engage in prostitution, or
becomes naturalized as a foreign citizen and obtains a connivance in such corruption or inducement;
divorce decree. The Filipino spouse should likewise be
allowed to remarry as if the other party were a foreigner
(4) Final judgment sentencing the respondent to
at the time of the solemnization of the marriage. To rule
imprisonment of more than six years, even if
otherwise would be to sanction absurdity and injustice.
pardoned;
Where the interpretation of a statute according to its exact
and literal import would lead to mischievous results or
contravene the clear purpose of the legislature, it should (5) Drug addiction or habitual alcoholism of the
be construed according to its spirit and reason, respondent;
disregarding as far as necessary the letter of the law. A
statute may therefore be extended to cases not within the (6) Lesbianism or homosexuality of the
literal meaning of its terms, so long as they come within its respondent;
spirit or intent.
(7) Contracting by the respondent of a
In view of the foregoing, we state the twin elements for subsequent bigamous marriage, whether in the
the application of Paragraph 2 of Article 26 as follows: Philippines or abroad;

3. There is a valid marriage that has been (8) Sexual infidelity or perversion;
celebrated between a Filipino citizen and a
foreigner; and (9) Attempt by the respondent against the life of
4. A valid divorce is obtained abroad by the alien the petitioner; or
spouse capacitating him or her to remarry.
(10) Abandonment of petitioner by respondent
The reckoning point is not the citizenship of the parties at without justifiable cause for more than one year.
the time of the celebration of the marriage, but their
citizenship at the time a valid divorce is obtained abroad
For purposes of this Article, the term "child" shall include a
by the alien spouse capacitating the latter to remarry.
child by nature or by adoption. (9a)

5. Legal Separation
Art. 56. The petition for legal separation shall be denied on
any of the following grounds:
Civil Code:

Art. 97. A petition for legal separation may be (1) Where the aggrieved party has condoned the
filed: offense or act complained of;

Private International Law Page 46


(2) Where the aggrieved party has consented to shall be forfeited in accordance with the
the commission of the offense or act complained provisions of Article 43(2);
of;
(3) The custody of the minor children shall be
(3) Where there is connivance between the awarded to the innocent spouse, subject to the
parties in the commission of the offense or act provisions of Article 213 of this Code; and
constituting the ground for legal separation;
(4) The offending spouse shall be disqualified
(4) Where both parties have given ground for from inheriting from the innocent spouse by
legal separation; intestate succession. Moreover, provisions in
favor of the offending spouse made in the will of
(5) Where there is collusion between the parties the innocent spouse shall be revoked by
to obtain decree of legal separation; or operation of law. (106a)

(6) Where the action is barred by prescription. Art. 64. After the finality of the decree of legal separation,
(100a) the innocent spouse may revoke the donations made by
him or by her in favor of the offending spouse, as well as
the designation of the latter as beneficiary in any insurance
Art. 57. An action for legal separation shall be filed within
policy, even if such designation be stipulated as
five years from the time of the occurrence of the cause.
irrevocable. The revocation of the donations shall be
(102)
recorded in the registries of property in the places where
the properties are located. Alienations, liens and
Art. 58. An action for legal separation shall in no case be encumbrances registered in good faith before the
tried before six months shall have elapsed since the filing recording of the complaint for revocation in the registries
of the petition. (103) of property shall be respected. The revocation of or change
in the designation of the insurance beneficiary shall take
Art. 59. No legal separation may be decreed unless the effect upon written notification thereof to the insured.
Court has taken steps toward the reconciliation of the
spouses and is fully satisfied, despite such efforts, that The action to revoke the donation under this Article must
reconciliation is highly improbable. (n) be brought within five years from the time the decree of
legal separation become final. (107a)
Art. 60. No decree of legal separation shall be based upon
a stipulation of facts or a confession of judgment. Art. 65. If the spouses should reconcile, a corresponding
joint manifestation under oath duly signed by them shall
In any case, the Court shall order the prosecuting attorney be filed with the court in the same proceeding for legal
or fiscal assigned to it to take steps to prevent collusion separation. (n)
between the parties and to take care that the evidence is
not fabricated or suppressed. (101a) Art. 66. The reconciliation referred to in the preceding
Articles shall have the following consequences:
Art. 61. After the filing of the petition for legal separation,
the spouses shall be entitled to live separately from each (1) The legal separation proceedings, if still
other. pending, shall thereby be terminated at
whatever stage; and
The court, in the absence of a written agreement between
the spouses, shall designate either of them or a third (2) The final decree of legal separation shall be
person to administer the absolute community or conjugal set aside, but the separation of property and any
partnership property. The administrator appointed by the forfeiture of the share of the guilty spouse
court shall have the same powers and duties as those of a already effected shall subsist, unless the spouses
guardian under the Rules of Court. (104a) agree to revive their former property regime.

Art. 62. During the pendency of the action for legal The court's order containing the foregoing shall be
separation, the provisions of Article 49 shall likewise apply recorded in the proper civil registries. (108a)
to the support of the spouses and the custody and support
of the common children. (105a)
Art. 67. The agreement to revive the former property
regime referred to in the preceding Article shall be
Art. 63. The decree of legal separation shall have the executed under oath and shall specify:
following effects:
(1) The properties to be contributed anew to the
(1) The spouses shall be entitled to live restored regime;
separately from each other, but the marriage
bonds shall not be severed;
(2) Those to be retained as separated properties
of each spouse; and
(2) The absolute community or the conjugal
partnership shall be dissolved and liquidated but
(3) The names of all their known creditors, their
the offending spouse shall have no right to any
addresses and the amounts owing to each.
share of the net profits earned by the absolute
community or the conjugal partnership, which

Private International Law Page 47


The agreement of revival and the motion for its approval 5. Resident aliens with whose
shall be filed with the court in the same proceeding for government the Republic of the
legal separation, with copies of both furnished to the Philippines has broken diplomatic
creditors named therein. After due hearing, the court shall, relations;
in its order, take measure to protect the interest of 6. Any person who has been
creditors and such order shall be recorded in the proper convicted of a crime involving
registries of properties. moral turpitude, when the
penalty imposed was six months'
The recording of the ordering in the registries of property imprisonment or more. (174a)
shall not prejudice any creditor not listed or not notified,
unless the debtor-spouse has sufficient separate properties Art. 339. The following cannot be adopted:
to satisfy the creditor's claim. (195a, 108a)
1. A married person, without the
written consent of the other
Capacity to Remarry
spouse;
2. An alien with whose government
Art. 26. All marriages solemnized outside the
the Republic of the Philippines has
Philippines, in accordance with the
broken diplomatic relations;
laws in force in the country where they
3. A person who has already been
were solemnized, and valid there as
adopted. (n)
such, shall also be valid in this country,
except those prohibited under Articles
Muslim Code:
35 (1), (4), (5) and (6), 3637 and 38.
(17a)
TITLE V
Parental Authority
Where a marriage between a Filipino
citizen and a foreigner is validly
CHAPTER I
celebrated and a divorce is thereafter
Nature and Effects
validly obtained abroad by the alien
spouse capacitating him or her to
Art. 71. Who exercises.
remarry, the Filipino spouse shall have
capacity to remarry under Philippine
(1) The father and the mother shall
law. (As amended by Executive Order
jointly exercise just and
227)
reasonable parental authority and
fulfill their responsibility over
Nota bene:
their legitimate and
acknowledged children. In case of
Formal validity law of the place of celebration; lex loci
disagreement, the father's
contractus rule
decision shall prevail unless there
Substantial validity governed by:
is a judicial order to the contrary.
1. Personal law: status/legal capacity National law
(2) The mother shall exercise parental
of the parties
authority over her children born
2. lex loci celebrationis:
out of wedlock, but the court may,
exceptions:
when the best interests of the
Family Code Art. 26, paragraph 1
children so require, appoint a
Consular Marriages consul granted by general guardian. .chan robles
the accepting country where such virtual law library
consular office was located to
solemnize marriage Art. 72. Duty to parents.

(1) Children shall respect, revere, and


VIII. Parents and Children (Parental Relationship) obey their parents always unless
the latter cast them into disbelief.
Civil Code:
(2) Grandparents are likewise entitled
Art. 15, supra to respect and reverence, and
shall be consulted whenever
Art. 335. The following cannot adopt: practicable by all members of the
family on all important questions.
1. Those who have legitimate,
legitimated, acknowledged Art. 73. Duty to children. Every parent and
natural children, or natural every person exercising parental
children by legal fiction; authority shall see to it that the rights
2. The guardian, with respect to the of the children are respected, and their
ward, before the final approval of duties complied with, and shall
his accounts; particularly by precept and example,
3. A married person, without the imbue them with religious and civic
consent of the other spouse; attachment to the ideal of permanent
4. Non-resident aliens; world peace.

Private International Law Page 48


Art. 74. Effects upon person of children. The minor above seven years of age
parents have, with respect to their but below the age of puberty may
unemancipated children: choose the parent with whom he
wants to stay.
(a) The duty to support them, have
them in their company, educate (2) The unmarried daughter who has
and instruct them in keeping with reached the age of puberty shall
their means and represent them stay with the father; the son,
in all actions which shall redound under the same circumstances,
to their benefits; and shall stay with the mother.

(b) The power to correct, discipline, Art. 79. Guardian for marriage (wali). The
and punish them moderately. following persons shall have authority
to act as guardian for marriage (wali) in
Art. 75. Effects upon property of children. the order of precedence:

(1) The father, or in his absence the (a) Father


mother, shall be the legal (b) Paternal grandfather;
administrator of the property of (c) Brother and other paternal
the child under parental authority. relatives;
If the property is worth more than (d) Paternal grandfather's executor or
five thousand pesos, the father or nominee; or .chan robles virtual
the mother shall give a bond to be law library
approved by the court. (e) The court.

(2) The court may appoint a guardian Art. 80. Guardian of minor's property. The
(wasi) in the absence of one who following persons shall exercise
is natural or testamentary. guardianship over the property of
minors in the order of precedence:
Art. 76. Parental authority non-transferable.
Parental authority can neither be (a) Father;
renounced nor transferred except as (b) Father's executor or nominee;
otherwise provided in this Code and (c) Paternal grandfather;
the general principles of Islamic law. (d) Paternal grandfather's nominee;
or
Art. 77. Extinguishment of parental authority. (e) The court.

(1) Parental authority terminates


upon the death of the parents or Art. 163. The filiation of children may be by nature or by
the child, or upon emancipation. adoption. Natural filiation may be legitimate or
(2) Subject to Article 78, the widowed illegitimate. (n)
mother who contracts a
subsequent marriage shall lose Art. 164. Children conceived or born during the marriage
parental authority and custody of the parents are legitimate.
over all children by the deceased
husband, unless the second
Children conceived as a result of artificial insemination of
husband is related to them within
the wife with the sperm of the husband or that of a donor
the prohibited degrees of
or both are likewise legitimate children of the husband and
consanguinity.
his wife, provided, that both of them authorized or ratified
(3) The court may deprive a person of
such insemination in a written instrument executed and
parental authority or suspend the
signed by them before the birth of the child. The
exercise thereof if he treats his
instrument shall be recorded in the civil registry together
children with excessive harshness,
with the birth certificate of the child. (55a, 258a)
gives then corrupting or immoral
orders and counsel, or abandons
them. Art. 165. Children conceived and born outside a valid
marriage are illegitimate, unless otherwise provided in this
CHAPTER II Code. (n)
Custody and Guardianship
Art. 166. Legitimacy of a child may be impugned only on
Art. 78. Care and custody. the following grounds:

(1) The care and custody of children (1) That it was physically impossible for the
below seven years of age whose husband to have sexual intercourse with his wife
parents are divorced shall belong within the first 120 days of the 300 days which
to the mother or, in her absence, immediately preceded the birth of the child
to the maternal grandmother, the because of:
paternal grandmother, the sister
and aunts. In their default, it shall
devolve upon the father and the
nearest paternal relatives. The

Private International Law Page 49


(a) the physical incapacity of the birth of the child or of the fact of registration of said birth,
husband to have sexual intercourse whichever is earlier. (263a)
with his wife;
Art. 171. The heirs of the husband may impugn the filiation
(b) the fact that the husband and wife of the child within the period prescribed in the preceding
were living separately in such a way article only in the following cases:
that sexual intercourse was not
possible; or (1) If the husband should died before the
expiration of the period fixed for bringing his
(c) serious illness of the husband, action;
which absolutely prevented sexual
intercourse; (2) If he should die after the filing of the
complaint without having desisted therefrom; or
(2) That it is proved that for biological or other
scientific reasons, the child could not have been (3) If the child was born after the death of the
that of the husband, except in the instance husband. (262a)
provided in the second paragraph of Article 164;
or
Chapter 2. Proof of Filiation

(3) That in case of children conceived through Art. 172. The filiation of legitimate children is established
artificial insemination, the written authorization by any of the following:
or ratification of either parent was obtained (1) The record of birth appearing in the civil
through mistake, fraud, violence, intimidation, or register or a final judgment; or
undue influence. (255a)
(2) An admission of legitimate filiation in a public
Art. 167. The child shall be considered legitimate although document or a private handwritten instrument
the mother may have declared against its legitimacy or and signed by the parent concerned.
may have been sentenced as an adulteress. (256a)
In the absence of the foregoing evidence, the legitimate
Art. 168. If the marriage is terminated and the mother filiation shall be
contracted another marriage within three hundred days
after such termination of the former marriage, these rules
proved by:
shall govern in the absence of proof to the contrary:

(1) The open and continuous possession of the


(1) A child born before one hundred eighty days
status of a legitimate child; or
after the solemnization of the subsequent
marriage is considered to have been conceived
during the former marriage, provided it be born (2) Any other means allowed by the Rules of
within three hundred days after the termination Court and special laws. (265a, 266a, 267a)
of the former marriage;
Art. 173. The action to claim legitimacy may be brought by
(2) A child born after one hundred eighty days the child during his or her lifetime and shall be transmitted
following the celebration of the subsequent to the heirs should the child die during minority or in a
marriage is considered to have been conceived state of insanity. In these cases, the heirs shall have a
during such marriage, even though it be born period of five years within which to institute the action.
within the three hundred days after the
termination of the former marriage. (259a) Art. 174. Legitimate children shall have the right:

Art. 169. The legitimacy or illegitimacy of a child born after (1) To bear the surnames of the father and the
three hundred days following the termination of the mother, in conformity with the provisions of the
marriage shall be proved by whoever alleges such Civil Code on Surnames;
legitimacy or illegitimacy. (261a)
(2) To receive support from their parents, their
Art. 170. The action to impugn the legitimacy of the child ascendants, and in proper cases, their brothers
shall be brought within one year from the knowledge of and sisters, in conformity with the provisions of
the birth or its recording in the civil register, if the husband this Code on Support; and
or, in a proper case, any of his heirs, should reside in the
city or municipality where the birth took place or was (3) To be entitled to the legitimate and other
recorded. successional rights granted to them by the Civil
Code. (264a)
If the husband or, in his default, all of his heirs do not
reside at the place of birth as defined in the first paragraph Chapter 3. Illegitimate Children
or where it was recorded, the period shall be two years if
they should reside in the Philippines; and three years if Art. 175. Illegitimate children may establish their
abroad. If the birth of the child has been concealed from illegitimate filiation in the same way and on the same
or was unknown to the husband or his heirs, the period evidence as legitimate children.
shall be counted from the discovery or knowledge of the

Private International Law Page 50


The action must be brought within the same period Art. 213. In case of separation of the parents, parental
specified in Article 173, except when the action is based on authority shall be exercised by the parent designated by
the second paragraph of Article 172, in which case the the Court. The Court shall take into account all relevant
action may be brought during the lifetime of the alleged considerations, especially the choice of the child over
parent. (289a) seven years of age, unless the parent chosen is unfit. (n)

Art. 176. Illegitimate children shall use the surname and Art. 214. In case of death, absence or unsuitability of the
shall be under the parental authority of their mother, and parents, substitute parental authority shall be exercised by
shall be entitled to support in conformity with this Code. the surviving grandparent. In case several survive, the one
The legitime of each illegitimate child shall consist of one- designated by the court, taking into account the same
half of the legitime of a legitimate child. Except for this consideration mentioned in the preceding article, shall
modification, all other provisions in the Civil Code exercise the authority. (355a)
governing successional rights shall remain in force. (287a)
Art. 215. No descendant shall be compelled, in a criminal
case, to testify against his parents and grandparents,
Chapter 4. Legitimated Children except when such testimony is indispensable in a crime
against the descendant or by one parent against the other.
Art. 177. Only children conceived and born outside of (315a)
wedlock of parents who, at the time of the conception of
the former, were not disqualified by any impediment to
marry each other may be legitimated. (269a) Chapter 2. Substitute and Special Parental Authority

Art. 178. Legitimation shall take place by a subsequent Art. 216. In default of parents or a judicially appointed
valid marriage between parents. The annulment of a guardian, the following person shall exercise substitute
voidable marriage shall not affect the legitimation. parental authority over the child in the order indicated:
(270a)chan robles virtual law library (1) The surviving grandparent, as provided in Art.
214;
Art. 179. Legitimated children shall enjoy the same rights
as legitimate children. (272a) (2) The oldest brother or sister, over twenty-one
years of age, unless unfit or disqualified; and
Art. 180. The effects of legitimation shall retroact to the
time of the child's birth. (273a) (3) The child's actual custodian, over twenty-one
years of age, unless unfit or disqualified.
Art. 181. The legitimation of children who died before the
celebration of the marriage shall benefit their Whenever the appointment or a judicial guardian over the
descendants. (274) property of the child becomes necessary, the same order
of preference shall be observed. (349a, 351a, 354a)
Art. 182. Legitimation may be impugned only by those who
are prejudiced in their rights, within five years from the Art. 217. In case of foundlings, abandoned neglected or
time their cause of action accrues. (275a) abused children and other children similarly situated,
parental authority shall be entrusted in summary judicial
Art. 209. Pursuant to the natural right and duty of parents proceedings to heads of children's homes, orphanages and
over the person and property of their unemancipated similar institutions duly accredited by the proper
children, parental authority and responsibility shall include government agency. (314a)
the caring for and rearing them for civic consciousness and
efficiency and the development of their moral, mental and Art. 218. The school, its administrators and teachers, or
physical character and well-being. (n) the individual, entity or institution engaged in child are
shall have special parental authority and responsibility
Art. 210. Parental authority and responsibility may not be over the minor child while under their supervision,
renounced or transferred except in the cases authorized by instruction or custody.
law. (313a)
Authority and responsibility shall apply to all authorized
Art. 211. The father and the mother shall jointly exercise activities whether inside or outside the premises of the
parental authority over the persons of their common school, entity or institution. (349a)
children. In case of disagreement, the father's decision
shall prevail, unless there is a judicial order to the contrary. Art. 129. Those given the authority and responsibility
under the preceding Article shall be principally and
Children shall always observe respect and reverence solidarily liable for damages caused by the acts or
towards their parents and are obliged to obey them as omissions of the unemancipated minor. The parents,
long as the children are under parental authority. (311a) judicial guardians or the persons exercising substitute
chan robles virtual law library parental authority over said minor shall be subsidiarily
liable.
Art. 212. In case of absence or death of either parent, the
parent present shall continue exercising parental authority. The respective liabilities of those referred to in the
The remarriage of the surviving parent shall not affect the preceding paragraph shall not apply if it is proved that they
parental authority over the children, unless the court exercised the proper diligence required under the
appoints another person to be the guardian of the person particular circumstances.
or property of the children. (n)

Private International Law Page 51


All other cases not covered by this and the preceding However, if in the same proceeding the court finds the
articles shall be governed by the provisions of the Civil petitioner at fault, irrespective of the merits of the
Code on quasi-delicts. (n) petition, or when the circumstances so warrant, the court
may also order the deprivation or suspension of parental
authority or adopt such other measures as it may deem
Chapter 3. Effect of Parental Authority just and proper. (318a)
Upon the Persons of the Children
Art. 224. The measures referred to in the preceding article
Art. 220. The parents and those exercising parental may include the commitment of the child for not more
authority shall have with the respect to their than thirty days in entities or institutions engaged in child
unemancipated children on wards the following rights and care or in children's homes duly accredited by the proper
duties: government agency.
(1) To keep them in their company, to support,
educate and instruct them by right precept and The parent exercising parental authority shall not interfere
good example, and to provide for their with the care of the child whenever committed but shall
upbringing in keeping with their means; provide for his support. Upon proper petition or at its own
instance, the court may terminate the commitment of the
(2) To give them love and affection, advice and child whenever just and proper. (391a)
counsel, companionship and understanding;

(3) To provide them with moral and spiritual Chapter 4. Effect of Parental Authority Upon
guidance, inculcate in them honesty, integrity, the Property of the Children
self-discipline, self-reliance, industry and thrift,
stimulate their interest in civic affairs, and inspire Art. 225. The father and the mother shall jointly exercise
in them compliance with the duties of legal guardianship over the property of the unemancipated
citizenship; common child without the necessity of a court
appointment. In case of disagreement, the father's
(4) To furnish them with good and wholesome decision shall prevail, unless there is a judicial order to the
educational materials, supervise their activities, contrary.
recreation and association with others, protect
them from bad company, and prevent them from Where the market value of the property or the annual
acquiring habits detrimental to their health, income of the child exceeds P50,000, the parent
studies and morals; concerned shall be required to furnish a bond in such
amount as the court may determine, but not less than ten
(5) To represent them in all matters affecting per centum (10%) of the value of the property or annual
their interests; income, to guarantee the performance of the obligations
prescribed for general guardians.
(6) To demand from them respect and
obedience; A verified petition for approval of the bond shall be filed in
the proper court of the place where the child resides, or, if
the child resides in a foreign country, in the proper court of
(7) To impose discipline on them as may be
the place where the property or any part thereof is
required under the circumstances; and
situated.

(8) To perform such other duties as are imposed


The petition shall be docketed as a summary special
by law upon parents and guardians. (316a)
proceeding in which all incidents and issues regarding the
performance of the obligations referred to in the second
Art. 221. Parents and other persons exercising parental paragraph of this Article shall be heard and resolved.
authority shall be civilly liable for the injuries and damages
caused by the acts or omissions of their unemancipated
The ordinary rules on guardianship shall be merely
children living in their company and under their parental
suppletory except when the child is under substitute
authority subject to the appropriate defenses provided by
parental authority, or the guardian is a stranger, or a
law. (2180(2)a and (4)a )
parent has remarried, in which case the ordinary rules on
guardianship shall apply. (320a)
Art. 222. The courts may appoint a guardian of the child's
property or a guardian ad litem when the best interests of
Art. 226. The property of the unemancipated child earned
the child so requires. (317)
or acquired with his work or industry or by onerous or
gratuitous title shall belong to the child in ownership and
Art. 223. The parents or, in their absence or incapacity, the shall be devoted exclusively to the latter's support and
individual, entity or institution exercising parental education, unless the title or transfer provides otherwise.
authority, may petition the proper court of the place
where the child resides, for an order providing for
The right of the parents over the fruits and income of the
disciplinary measures over the child. The child shall be
child's property shall be limited primarily to the child's
entitled to the assistance of counsel, either of his choice or
support and secondarily to the collective daily needs of the
appointed by the court, and a summary hearing shall be
family. (321a, 323a)
conducted wherein the petitioner and the child shall be
heard.
Art. 227. If the parents entrust the management or
administration of any of their properties to an

Private International Law Page 52


unemancipated child, the net proceeds of such property The suspension or deprivation may be revoked and the
shall belong to the owner. The child shall be given a parental authority revived in a case filed for the purpose or
reasonable monthly allowance in an amount not less than in the same proceeding if the court finds that the cause
that which the owner would have paid if the administrator therefor has ceased and will not be repeated. (33a)
were a stranger, unless the owner, grants the entire
proceeds to the child. In any case, the proceeds thus give Art. 232. If the person exercising parental authority has
in whole or in part shall not be charged to the child's subjected the child or allowed him to be subjected to
legitime. (322a) sexual abuse, such person shall be permanently deprived
by the court of such authority. (n)

Chapter 5. Suspension or Termination of Parental Authority Art. 233. The person exercising substitute parental
authority shall have the same authority over the person of
Art. 228. Parental authority terminates permanently: the child as the parents.
(1) Upon the death of the parents;
In no case shall the school administrator, teacher of
(2) Upon the death of the child; or individual engaged in child care exercising special parental
authority inflict corporal punishment upon the child. (n)
(3) Upon emancipation of the child. (327a)
Ching Leng v. Galang
Art. 229. Unless subsequently revived by a final judgment, 57 OG 2312
parental authority also terminates:
(1) Upon adoption of the child;
In Ching Leng the provision in the 1935 Constitution
(2) Upon appointment of a general guardian; stating "those whose fathers are citizens of the
Philippines" refers only to legitimate children. When the
(3) Upon judicial declaration of abandonment of 1973 and 1987 Constitutions were drafted, the framers did
the child in a case filed for the purpose; not attempt to change the intent of this provision, even as
they were presumably aware of the Ching Leng doctrine.
(4) Upon final judgment of a competent court
divesting the party concerned of parental Nevertheless, I believe that it is now time to abandon the
authority; or Ching Leng doctrine. (Tecson V. Comelec)

(5) Upon judicial declaration of absence or The prevailing doctrine today is that an illegitimate child of
incapacity of the person exercising parental a Filipino father and an alien mother follows the citizenship
authority. (327a) of the alien mother as the only legally known parent. The
illegitimate child, even if acknowledged and legally
adopted by the Filipino father, cannot acquire the
Art. 230. Parental authority is suspended upon conviction
citizenship of the father. The Court made this definitive
of the parent or the person exercising the same of a crime
doctrinal ruling in Ching Leng v. Galang which involved the
which carries with it the penalty of civil interdiction. The
illegitimate minor children of a naturalized Filipino of
authority is automatically reinstated upon service of the
Chinese descent with a Chinese woman, Sy An. The
penalty or upon pardon or amnesty of the offender. (330a)
illegitimate children were later on jointly adopted by the
naturalized Filipino and his legal wife, So Buan Ty.
Art. 231. The court in an action filed for the purpose in a
related case may also suspend parental authority if the
The facts in Ching Leng as quoted by the Court from the
parent or the person exercising the same:
trial courts decision are as follows:

(1) Treats the child with excessive harshness or


After the petitioner Ching Leng Alias Ching Ban Lee
cruelty;
obtained judgment in this Court dated May 2, 1950
granting his petition for naturalization, he together with
(2) Gives the child corrupting orders, counsel or his wife So Buan Ty filed another petition also in this Court
example; in Special Proc. No. 1216 for the adoption of Ching Tiong
Seng, Ching Liong Ding, Victoria Ching Liong Yam, Sydney
(3) Compels the child to beg; or Ching and Ching Tiong An, all minors and admittedly the
illegitimate children of petitioner Ching Leng with one Sy
(4) Subjects the child or allows him to be An, a Chinese citizen. Finding the petition for adoption
subjected to acts of lasciviousness. proper, this Court granted the same in a decision dated
September 12, 1950, declaring the said minors free from
all legal obligations of obedience and maintenance with
The grounds enumerated above are deemed to include respect to their mother Sy An and to all legal intents and
cases which have resulted from culpable negligence of the purposes the children of the adopter Ching Leng alias
parent or the person exercising parental authority. Ching Ban Lee and So Buan Ty with all the legal rights and
obligations provided by law.
If the degree of seriousness so warrants, or the welfare of
the child so demands, the court shall deprive the guilty On September 29, 1955, Ching Leng took his oath of
party of parental authority or adopt such other measures allegiance and became therefore a full pledge (sic)
as may be proper under the circumstances. Filipino citizen. Believing now that his adopted illegitimate
children became Filipino citizens by virtue of his

Private International Law Page 53


naturalization, petitioner Ching Leng addressed a Marcosa S. Dy Jiongco, a Filipina born of a Filipina
communication to the respondent Commissioner of mother and a Chinese father was married to a
Immigration requesting that the alien certificate of Chinaman by the name of (Filipino name) Juan Uy
registration of the said minors be cancelled. (Bold Tue, (Chinese name) Ng Chion Tue.
underscoring supplied) That Juan Uy Tue (Ng Chion Tue), before his marriage
with Marcosa S. Dy Jiongco, had been married to a
In Ching Leng, the Court made a definitive ruling on the Chinese woman with whom he had some children,
meaning of "minor child or children" in Section 15 of the the petitioner herein and also one called Ng Guan.
Naturalization Law, as well as the meaning of children It appears that Ng Guan was residing in the Philippine
"whose parents are citizens of the Philippines" under the Islands at the time of the presentation of the present
Constitution. The Court categorically ruled that these petition;
children refer to legitimate children only, and not to The Chinese wife of Juan Uy Tue died while the
illegitimate children. Thus, the Court held: petitioner herein, Ng Hian, was a very small child;
Juan Uy Tue, after the death of his Chinese wife,
married Marcosa S. Dy Jiongco.
It is claimed that the phrases "minor children" and "minor
child", used in these provisions, include adopted children. Ng Tio was the daughter of the brother of the said
The argument is predicated upon the theory that an Juan Uy Tue, born of a Chinese father and mother;
adopted child is, for all intents and purposes, a legitimate that the father of the little girl had given her to
child. Whenever, the word "children" or "child" is used in Marcosa S. Dy Jiongco;
statutes, it is generally understood, however, to refer to Marcosa S. Dy Jiongco, being the stepmother of the
legitimate children, unless the context of the law and its said Ng Hian, adopted him and was bringing him to
spirit indicate clearly the contrary. Thus, for instance, the Philippine Islands to study.
when the Constitution provides that "those whose parents The Board of Special Inquiry refused the right of each
are citizens of the Philippines, "and "those whose mothers child to enter the Philippine Islands;
are citizens of the Philippines," who shall elect Philippine However, after a rehearing, Ng Tio was admitted but
citizenship "upon reaching the age of majority", are Ng Hian was not.
citizens of the Philippines (Article IV, Section 1, Petitioner petitioned for habeas corpus in the CFI, CFI
subdivisions 3 and 4), our fundamental law clearly refers to granted entry to petitioner, Collector of Customs
legitimate children (Chiong Bian vs. De Leon, 46 Off. Gaz., appealed; Thus the present petition;
3652-3654; Serra v. Republic, L-4223, May 12, 1952).
Issue:
Similarly, the children alluded to in said section 15 are
Whether or not the minor children of a deceased resident
those begotten in lawful wedlock, when the adopter, at
Chinese merchant have a right to enter the territory of the
least is the father. In fact, illegitimate children are under
Philippine Islands;
the parental authority of the mother and follow her
nationality, not that of the illegitimate father (U.S. vs. Ong
Held:
Tianse, 29 Phil. 332, 335-336; Santos Co vs. Govt of the
Philippines, 52 Phil. 543, 544; Serra v. Republic, supra;
It is true that the petitioner, Ng Hian, had never been in
Gallofin v. Ordoez, 70 Phil. 287; Quimsuan vs. Republic, L-
the Philippine Islands before. It is also true that the said
4693, Feb. 16, 1953). Although, adoption gives "to the
Marcosa S. Dy Jingco was his stepmother. She swore
adopted person the same rights and duties as if he were a
positively that she had adopted him. That fact is not
legitimate child of the adopter", pursuant to said Article
denied of record. Until the fact is denied we must accept it.
341 of our Civil Code, we have already seen that the
There is nothing in the record which shows or tends to
rights therein alluded to are merely those enumerated in
show that she had not adopted him in good faith. The
Article 264, and do not include the acquisition of the
question whether or not Marcosa S. Dy Jiongco could bring
nationality of the adopter.
Ng Hian into the territory of the Philippine Islands as her
adopted son has been discussed by the Federal Courts of
Moreover, as used in said section 15 of the Naturalization the United States. In the case of Ex parte Fong Yim (134
Law, the term "children" could not possibly refer to those Fed. Rep., 938), the court held that:
whose relation to the naturalized person is one created
by legal fiction, as, for instance, by adoption, for, A Chinese merchant domiciled in the United States has
otherwise, the place and time of birth of the child would the right to bring into this country with his wife minor
be immaterial. The fact that the adopted persons involved children legally adopted by him in China, where it is
in the case at bar are illegitimate children of appellant shown that the adoption was bona fide, and that the
Ching Leng does not affect substantially the legal situation children have lived as members of his family and have
before us, for, by legal fiction, they are now being sought been supported by him for several years.
to be given the status of legitimate children of said
appellant, despite the circumstance that the Civil Code of The court further said:
the Philippine does not permit their legitimation.
Of course, the question whether the adoption is a genuine
Ng Hian v. Collector one is a question of fact, open to investigation . . . . The
34 Phil 248 evidence shows that the practice of adopting children in
China is very common, that it takes place substantially
On 30th of October, 1915 the steamship Tian arrived without legal formalities, but that the rights and
at the port of Manila. obligations of children adopted and recognized as such
A woman, Marcosa S. Dy Jiongco, together with two are similar to those of natural children. Under these
children, Ng Tio a female of the age of 9 years, and Ng circumstances I can see no difference between the legal
Hian a boy of 16 years of age (the petitioner herein) status of adopted children and of natural children. The
were on board. Supreme Court (of the United States) having decided that a
Chinese merchant domiciled in this country has the right

Private International Law Page 54


to bring into it his natural children, I think that the same parties opposed to the judgment had been given ample
decision is authority for the proposition that he has the opportunity to do so on grounds allowed under Rule 39,
right to introduce his adopted children; Section 50 of the Rules of Court (now Rule 39, Section 48,
1997 Rules of Civil Procedure), to wit:
ROEHR VS. RODRIGUEZ
SEC. 50. Effect of foreign judgments. - The effect
of a judgment of a tribunal of a foreign country,
Facts: having jurisdiction to pronounce the judgment is
as follows:
Petitioner Wolfgang Roehr, a German citizen, married a
Filipina, Respondent Carmen Rodriguez in Germany. The
(a) In case of a judgment upon a specific thing,
marriage was ratified in Tayasan, Negros Oriental. They had
the judgment is conclusive upon the title to the
two children. Private respondent filed a petition for the
thing;
declaration of nullity of marriage before the RTC of Makati.
Petitioner filed a motion to dismiss but was denied by the
trial court. However, The petitioner obtained a decree of (b) In case of a judgment against a person, the
divorce from the Court of First Instance of Hamburg - judgment is presumptive evidence of a right as
Blankenese with the custody of the children granted to the between the parties and their successors in
father. interest by a subsequent title; but the judgment
may be repelled by evidence of a want of
Issue: jurisdiction, want of notice to the party,
collusion, fraud, or clear mistake of law or fact.
Whether or not the legal effects of a divorce obtained from
a foreign country such as support and custody of the It is essential that there should be an opportunity to
children can be determined in our courts? challenge the foreign judgment, in order for the court in
this jurisdiction to properly determine its efficacy. In this
Held: jurisdiction, our Rules of Court clearly provide that with
respect to actions in personam, as distinguished from
Yes. In order to take effect, a foreign judgement must actions in rem, a foreign judgment merely constitutes
clearly show that the opposing party has been given prima facie evidence of the justness of the claim of a
ample opportunity to do so under the Rules of Civil party and, as such, is subject to proof to the contrary.
Procedure. Accordingly, the respondent was not given the
opportunity to challenge the judgment of the German In the present case, it cannot be said that private
Court, therefore, legal effects of divorce must be respondent was given the opportunity to challenge the
determined in our courts. The court held that the trial court judgment of the German court so that there is basis for
has jurisdiction over the issue between the parties as to declaring that judgment as res judicata with regard to the
who has the parental custody. rights of petitioner to have parental custody of their two
children. The proceedings in the German court were
summary.
In Garcia v. Recio,19 Van Dorn v. Romillo, Jr.,20 and Llorente
v. Court of Appeals, the Court consistently held that a Exam is on Wednesday
divorce obtained abroad by an alien may be recognized in 630 to 830
our jurisdiction, provided such decree is valid according Coverage is from start to parents and children
to the national law of the foreigner. Relevant to the
present case is Pilapil v. Ibay-Somera, where the Court
specifically recognized the validity of a divorce obtained by
a German citizen in his country, the Federal Republic of
Germany. The Court held in Pilapil that a foreign divorce
and its legal effects may be recognized in the Philippines
insofar as respondent is concerned in view of the
nationality principle in our civil law on the status of
persons.

In this case, the divorce decree issued by the German


court dated December 16, 1997 has not been challenged
by either of the parties. In fact, save for the issue of
parental custody, even the trial court recognized said
decree to be valid and binding, thereby endowing private
respondent the capacity to remarry. Thus, the present
controversy mainly relates to the award of the custody of
their two children, Carolynne and Alexandra Kristine, to
petitioner.

As a general rule, divorce decrees obtained by foreigners


in other countries are recognizable in our jurisdiction, but
the legal effects thereof, e.g. on custody, care and
support of the children, must still be determined by our
courts. Before our courts can give the effect of res judicata
to a foreign judgment, such as the award of custody to
petitioner by the German court, it must be shown that the

Private International Law Page 55

You might also like